You are on page 1of 236

(SAE08OS.

6) Figures 2a and 2b show the radiograph and MRI scan of a 56-year-old


woman who has low back pain and right leg pain. She has grade 3/5 toe and ankle
dorsiflexion strength on the right side. Nonsurgical management has failed to provide
relief; therefore, surgery should include Review Topic

1 L5 pars repair.
2 L5 laminectomy alone.
3 L5 laminectomy and fusion.
4 stand-alone anterior lumbar interbody fusion.
5 L5-S1 total disk replacement.

PREFERRED RESPONSE 3

The lateral radiograph and MRI scan demonstrate a grade 2 isthmic spondylolisthesis
of L5 on S1. The radiograph shows a pars defect of L5. Isthmic spondylolistheses are
most common at L5-S1. Degenerative spondylolistheses rarely progress beyond a
grade 1 slip. The patient has frank neurologic weakness on the right side and
nonsurgical management has failed to provide relief. In patients with significant
motor weakness, neurologic decompression is indicated. An L5 pars repair is not
recommended in patients with more than a grade 1 slip. Laminectomy alone can
destabilize the spine and lead to further slippage and thus it is recommended to fuse
the segment. A stand-alone anterior lumbar interbody fusion has a high failure rate
with isthmic spondylolisthesis. Isthmic spondylolisthesis is a contraindication for
lumbar total disk replacement. While there is some literature that supports fusion
without laminectomy or decompression for patients with isthmic slips and radicular
pain without neurologic deficit, this patient does not fulfill these criteria.

(SAE11OS.87) Based on the current available best-evidence, what postoperative


activities should be recommended for patients undergoing first-time lumbar
diskectomy for disk herniation? Review Topic

1 Bed rest
2 Avoid exercise for 6 to 8 weeks
3 Early return to low-intensity exercise
4 Early return to high-intensity exercise
5 Gradual return to low-intensity exercise after 6 weeks

PREFERRED RESPONSE 4

Recent evidence supports return to high-intensity exercise at 4 weeks for patients


undergoing first-time lumbar diskectomy for disk herniation.

Ostelo et al., in a Cochrane review update, demonstrated that exercise programs


starting 4 to 6 weeks after surgery in patients undergoing first-time lumbar
diskectomy for disk herniation lead to a faster decrease in pain and disability than no
rehabilitation. Additionally, high-intensity exercise programs seem to lead to a faster
decrease in pain and disability than low-intensity programs.

Carragee et al., in a prospective review of 50 consecutive patients undergoing first-


time lumbar diskectomy for disk herniation, demonstrated that lifting of postoperative
activity restrictions after limited diskectomy allowed shortened sick leave without
increased complications. He concluded that postoperative precautions in these patients
may not be necessary.

(SAE12SN.63) Figures 63a and 63b show the radiographs of a 38-year-old man who
reports low back and bilateral lower extremity pain. The spondylolisthesis is best
classified as which of the following? Review Topic

1 Pathologic
2 Isthmic
3 Acquired
4 Degenerative
5 Dysplastic
PREFERRED RESPONSE 2

Spondylolisthesis can be classified into five types. Type I, dysplastic, occurs at the
lumbosacral junction as a result of congenital abnormalities of the upper sacrum
and/or the arch of L5. Type II, isthmic, refers to those involving a lesion in the pars
interarticularis. Type IIA, lytic, represents fatigue fractures of the pars. Type IIB
describes those with elongated, but intact pars. Type IIC describes those that are a
result of an acute fracture of the pars. Type III, degenerative spondylolisthesis, results
from long-standing intersegmental disease. Type IV, traumatic, refers to those
resulting from fractures in regions other than the pars, such as the pedicles. Type V,
pathologic, refers to spondylolisthesis resulting from generalized or local bone
disease. The radiographs demonstrate type II, isthmic spondylolisthesis.

(SAE09SN.97) What is the most common nonanesthetic-related reversible cause of


changes in intraoperative neurophysiologic monitoring data? Review Topic

1 Pedicle screw misplacement


2 Patient positioning
3 Spinal cord ischemia
4 Retractor placement
5 Hypotension

PREFERRED RESPONSE 2

Patient positioning that results in local nerve compression, plexus traction, or


improper neck alignment is the most common nonanesthetic-related cause of changes
in intraoperative neurophysiologic monitoring data during spinal surgery.

(OBQ13.192) Which of following is pathognomonic of intervertebral disk


degeneration? Review Topic

1 Increased water content in the nucleus pulposus


2 Loss of Type X collagen in the nucleus pulposus
3 Decreased thickness of the inner annulus fibers
4 Degradation of large proteoglycans molecules in the nucleus pulposus
5 Annular tears and fissuring occurring most frequently in the central disc

PREFERRED RESPONSE 4

Degradation of large proteoglycan molecules in the nucleus pulposus is


pathognomonic of intervertebral disk (IVD) degeneration.

Degeneration of the intervertebral disk (IVD) is a major pathological process


implicated in low back pain and is often considered a prerequisite for intervertebral
disc herniation. While the pathophysiologic causes of IVD degeneration at the
molecular level are not fully known, there are many physical and molecular changes
that are known to contribute to the disease process. The most significant is loss of
large proteoglycan molecules and decreased water content.

An et al. showed that large proteoglycans (PGs), such as aggrecan and versican,
decrease in patients with intervertebral disk (IVD) degeneration.

Kepler et al. reviewed IVD degeneration. They report that degeneration leads to
changes in the expression of matrix proteins, cytokines, and proteinases. They suggest
treatment with gene therapy, such as Growth and Differentiation Factor-5 (GDF-5),
may help to promote the healing of degenerated intervertebral disks.

Illustration A shows a cadaveric image of normal disk anatomy (left) and IVD
degeneration (right)

Incorrect Answers:
Answer 1: Water content in the nucleus pulposus is decreased in intervertebral disk
(IVD) degeneration.
Answer 2: Type X collagen is one of the key molecules in endochondral bone growth
and development. Type X collagen has been shown to increase in IVD.
Answer 3: The inner layers of annulus will thicken with degenerative disease. As a
result, the fibre bundles within the layers become more irregularly distributed with
increased interbundle spaces. This renders them susceptible to injury due to the
inability to support sustained loads.
Answer 5: Annular tears and fissures occur most frequently in the posterolateral
location of the annulus fibrosis.

(SAE12SN.49) Halo treatment for preadolescent children typically requires the use of
which of the following? Review Topic

1 4 to 6 pins with an insertional torque of 1 to 5 in-lb


2 4 to 6 pins with an insertional torque of 4 to 6 in-lb
3 4 to 6 pins with an insertional torque of 6 to 8 in-lb
4 8 to 12 pins with an insertional torque of 1 to 5 in-lb
5 8 to 12 pins with an insertional torque of 4 to 6 in-lb

PREFERRED RESPONSE 4

The complication rate with halo vest treatment in children is reported to be as high as
68% in contrast to a 36% complication rate in adults. These complications include not
only pin tract infections, but also skull penetration. Multiple pins allow for the early
removal of pins without fixation consequences should pin site infections begin to
develop. Moreover, there is significant variability in the insertional torque applied by
a variety halo pin torque wrenches, including those from the same manufacturer.
Consequently, the use of a large number of pins (8 to 12) placed a very low
insertional torque (1 to 5 in-lb) in children is recommended. A CT scan of the head
should also be considered to assess for the thickest areas of the skull suitable for pin
application.

(OBQ14.77) A 58-year-old male presents after a motor vehicle accident with severe
pain and point tenderness over his lumbar spine. He is hemodynamically stable and
full neurologic examination reveals no deficits. Radiographs showed no evidence of
fracture. A CT was performed and is shown in Figure A. What is the most appropriate
treatment of his injury. Review Topic

1 Anti-inflammatory medication and gradual return to activity


2 Admit for pain control and repeat neurologic examination
3 Thoracolumbosacral orthosis
4 Multi-level posterior spinal instrumentation and fusion
5 Anterior corpectomy and fusion
PREFERRED RESPONSE 4

The clinical presentation is consistent with a minimally displaced fracture that extends
through all three columns on the spine in a patient with ankylosing spondylitis. The
most appropriate treatment is posterior spinal instrumentation and fusion.

In patients with ankylosing spondylitis (AS), the rigid spine creates a long lever arm
that makes even minimally displaced fractures potentially unstable. Thus, despite
being minimally displaced in a neurologically intact patient, most fractures in AS
warrant a posterior instrumentation and fusion, typically of three levels above and
three levels below the fracture.

Caron et al. present a retrospective review of patients with ankylosed spines (due to
AS or DISH) and characterize their fractures and outcomes of treatment. Cervical
fractures were most common (55%) and rates of occurrence decreased as they
progressed down the spine, with lumbar fractures seen only 8% of the time. They
found the most common successful surgical intervention was multilevel posterior
instrumentation and fusion (with decompression when necessary for neurologic
compromise).

Wang et al. reviewed 12 cases of patients with AS who presented with traumatic
spinal injuries. They reviewed clinical histories and available imaging. They found
that MRI was the most sensitive test for identifying occult fractures of the spine, and
recommend using MRI to rule out occult fractures as well as better characterize
fractures seen on radiographs or CT imaging for patients with AS.

Werner et al. present a review of spinal fractures in patients with AS. They note that
non-operative treatment of these fractures is reserved only for patients who have an
unacceptably high risk of undergoing surgery. For three column thoracolumbar
fractures they recommend surgical stabilization with a long posterior construct.

Figure A is a sagittal CT image of the lumbar spine with marginal syndesmophytes


consistent with AS. The white arrow highlights a minimally displaced fracture that
extends through all three columns on the spine. Illustration A are radiographs
comparing DISH (on the left) with non-marginal syndesmophytes, and AS (on the
right) with marginal syndesmophytes.

Incorrect Answers:
Answer 1: In an ankylosed spine, this is an unstable fracture and requires surgical
stabilization.
Answer 2: Despite being neurologically intact, this fracture requires stabilization
whether or not the neurologic status changes. Thus waiting for repeat examination
prior to surgery is not required.
Answer 3: In an ankylosed spine, this is an unstable fracture and requires surgical
stabilization.
Answer 5: This fracture is best stabilized via posterior instrumentation and fusion.
(SAE13PE.45) What is the minimum hours per day of wear that has been correlated
with the effectiveness of bracing on curve progression in idiopathic scoliosis? Review
Topic

1 Prescribed brace wear 23 hours/day


2 Prescribed brace wear 16 hours/day
3 Actual brace wear more than 12 hours/day
4 Actual brace wear 6 hours/day

PREFERRED RESPONSE 3

The efficacy of brace treatment for patients with adolescent idiopathic scoliosis is
controversial because its effectiveness remains unproven. One of the challenges is
patient noncompliance with prescribed bracing regimens. A recent study investigated
curve progression based on actual brace wear using a temperature sensor to accurately
assess brace wear. The total hours of brace wear correlated with lack of curve
progression with a dose-response effect noted. Curves did not progress in 82% of
patients who actually wore the brace more than 12 hours per day. For those who wore
the brace for fewer than 7 hours per day, curves progressed in 69%. Prescribed
bracing regimens (eg, 16 hours/day or 23 hours/day) had no effect on actual brace
wear or curve progression.

(SAE09SN.17) A 36-year-old woman is brought to the emergency department


intubated and sedated following a motor vehicle accident. She is moving her upper
and lower extremities spontaneously. She cannot follow commands. CT scans are
shown in Figures 7a through 7c. The initial survey does not reveal any other injuries.
Initial management of the cervical injury should consist of immediate Review Topic

1 immobilization with a halo ring and vest with reduction when medically stable.
2 closed traction reduction using Gardner-Wells tongs.
3 posterior open reduction, stabilization, and fusion.
4 cervical MRI followed by reduction.
5 anterior open reduction, stabilization, and fusion.

PREFERRED RESPONSE 4

The patient has a bilateral facet dislocation of C6-C7 with preservation of at least
some neurologic function. Urgent reduction is necessary. However, because she is
sedated and unable to follow commands, an MRI scan is necessary before any closed
or open posterior reduction to look for an associated disk herniation. If a disk
herniation is present, it must be removed prior to any reduction maneuver to prevent
iatrogenic neurologic injury. It is very unlikely that this injury can be reduced with an
open anterior procedure alone.

(SAE07PE.71) A 9-year-old child sustained a fracture-dislocation of C-5 and C-6


with a complete spinal cord injury. What is the likelihood that scoliosis will develop
during the remaining years of his growth? Review Topic

1 10%
2 20%
3 50%
4 70%
5 100%

PREFERRED RESPONSE 5

The incidence of late spinal deformity after complete spinal cord injury in children
depends on the level of the spinal cord injury and the age of the patient at the time of
injury. If a cervical level injury occurs before age 10 years, paralytic scoliosis will
develop in virtually 100% of patients.

(SAE09SN.53) A 29-year-old man reports a 2-week history of severe neck pain after
being struck sharply on the back of the head and neck while moving a refrigerator
down a flight of stairs. Initial evaluation in the emergency department revealed no
obvious fracture and he was discharged in a soft collar. Neurologic examination is
within normal limits, and radiographs taken in the office are shown in Figures 21a
through 21c. Subsequent MRI scans show intra-substance rupture of the transverse
atlantal ligament. What is the most appropriate treatment option at this time? Review
Topic
1 Discontinue use of the soft collar and encourage range of motion
2 Semi-rigid collar immobilization for 6 to 8 weeks
3 Surgical stabilization
4 Halo skeletal fixation
5 Outpatient physical therapy with isometric neck exercises

PREFERRED RESPONSE 3

Dickman and associates classified injuries of the transverse atlantal ligament into two
categories. Type I injuries are disruptions through the substance of the ligament itself.
Type II injuries render the transverse ligament physiologically incompetent through
fractures and avulsions involving the tubercle of insertion of the transverse ligament
on the C1 lateral mass. Type I injuries are incapable of healing without supplemental
internal fixation. Type II injuries can be treated with a rigid cervical orthosis with a
success rate of 74%. Surgery may be required for type II injures that fail to heal with
3 to 4 months of nonsurgical management.

(SBQ13PE.82) A renal ultrasound should be obtained in a patient with which of the


following diagnoses? Review Topic

1 Congenital pseudoarthrosis of the clavicle


2 Accessory navicular
3 Congenital scoliosis
4 Charcot-Marie-Tooth Disease
5 Congenital curly toe
PREFERRED RESPONSE 3

A renal ultrasound should be obtained in a patient with congenital scoliosis.

Congenitial scoliosis has a prevalence that is estimated at 1% to 4% in general


population. It is associated with systemic anomalies in up to 61% of cases. The most
common anomalies are cardiac defects, genitourinary defects and spinal cord
malformations. As such, additional imaging of these systems should be considered in
the initial work-up of these patients.

Ruf et al. retrospectively investigated 56 consecutive operative cases of congenital


scoliosis. The average age at surgery was 9.9 years (1.5–17 years). There was one
wound complication, 2 hardware failures, 1 revision and no neurological
complications. They concluded that one-stage posterior hemivertebra resection and
instrumentation was safe in this population.

Xu et al. examined the efficacy of selective partial hemivertebra excision via


posterior-only approach in 17 adolescent patients with congenital kyphoscoliosis.
There were no postoperative infections and no neurological complications. They
concluded that this approach may be most successful in patients aged 9 to 14 years
old, with the Risser sign grades from 0–3 and Cobb angles <60°.

Illustration A from Erol et al (UPOJ, Vol 15, 2002;37-42) shows a diagram of 'types'
of congenital scoliosis based on the morphology of the vertebrae.

Incorrect Answers:
Answer 1,2,5: These diagnoses do not seem to have systemic associations.
Answer 4: Charcot-Marie-Tooth Disease does not commonly manifest in the kidney

(SAE10PE.9) A 13-year-old girl with adolescent idiopathic scoliosis is otherwise


healthy with a normal neurologic examination and she began her menstrual cycle 3
months ago. Standing radiographs show a high left thoracic curve from T1-T6 that
measures 29 degrees, a right thoracic curve from T7-L1 that measures 65 degrees, and
a left lumbar curve from L1-L5 that measures 31 degrees, correcting to 12, 37, and 10
degrees, respectively, on bending films. Her Risser sign is 1. What is the most
appropriate management? Review Topic

1 Bracing
2 Posterior spinal fusion of only the right thoracic curve
3 Posterior spinal fusion from T2-L4
4 Vertebral body stapling to halt progression of the curve
5 Anterior and posterior spinal fusion
PREFERRED RESPONSE 2

The patient has typical adolescent idiopathic scoliosis with a right thoracic curve. This
represents a Lenke-1B curve pattern; therefore, only treatment of the thoracic curve is
required. The proximal thoracic and thoracolumbar curves are very flexible. The
patient is Risser 1 and has just started her menstrual cycles; therefore, she is at
significant risk for further curve progression. Bracing is not appropriate for a curve of
this magnitude and will not halt the progression of this curve, nor will vertebral body
stapling stop this curve. Vertebral body stapling is sometimes useful in small thoracic
curves of less than 35 degrees and skeletally immature patients. Anterior and posterior
spinal fusion is not required because the patient has no other risk factors, such as
neurofibromatosis nor is she at risk for crankshaft. Anterior fusion is an option, but it
is not listed.

(SAE11AN.28) The radiographic findings shown at the C5-6 disk above the C6-7
fusion in Figure 28 are most commonly associated with what part of the surgical
technique? Review Topic

1 A needle placed there for radiographic confirmation of the level


2 Excessive periosteal stripping of the upper fused level
3 Dissection of the longus colli extending cephalad to that disk
4 Screw penetration of the upper end plate
5 The upper end of the plate being in close proximity to the adjacent disk

PREFERRED RESPONSE 5

When the upper border of the plate is located in close proximity to the cephalad
adjacent disk, there is a higher incidence of osteophyte formation. The clinical
implications of this are not yet understood. Screw penetration or needle puncture may
influence the degenerative process at that disk, but this would manifest itself more as
narrowing and end plate changes as opposed to an osteophyte forming along the
anterior annulus. The role of the longus colli and periosteal dissection are not fully
elucidated but are less commonly associated with this finding.
(SAE12SN.47) A 75-year-old woman reports persistent severe low back pain after
lifting a bag of groceries 8 weeks ago. She denies the presence of any lower extremity
pain, weakness, or other symptoms. AP and lateral lumbar spine radiographs are
shown in Figures 47a and 47b. For further evaluation, what imaging study should be
obtained next? Review Topic

1 Lumbar spine flexion and extension radiographs


2 Lumbar spine CT
3 Lumbar spine MRI
4 Whole body bone scan
5 Tagged white blood cell scan

PREFERRED RESPONSE 3

The radiographs show a L1 vertebral compression fracture. MRI scans of the lumbar
spine are indicated to help determine the age of the fracture via evaluation of the fat-
suppressed T2-weighted images. If those images show edema (bright signal) within
the fracture, it can be assumed to be an acute or subacute fracture. Also, MRI scans
may help further characterize the fracture as a benign osteoporosis-related fracture
versus a pathologic fracture. Lastly, MRI scans allow for evaluation of the absence,
presence, and degree of spinal stenosis. Although a lumbar spine CT scan is also a
good option as a next step, given that the patient has a known fracture as seen on the
radiographs, MRI scans will likely provide more relevant information in this case. A
CT scan may assist in the evaluation of the posterior vertebral body wall and posterior
elements in patients with burst fractures. A three-phase bone scan is a good alternative
for the evaluation of the acuity of vertebral compression fractures in patients who
have a contraindication to MRI scans. Flexion-extension radiographs are most
commonly used to evaluate for instability and will not help determine the age of this
fracture. Tagged white blood cell scans are performed to evaluate for infection, which
is highly unlikely in this patient.
(SAE12SN.87) Metal-on-metal lumbar disk arthroplasty devices may generate cobalt
and chromium ions into the serum of patients after implantation into the lumbar spine.
Which of the following statements best represents the levels of the serum ion levels in
these patients? Review Topic

1 The serum ion levels are not measureable in these patients.


2 The serum ion levels are measureable, but are of negligible value in these patients.
3 The serum ion levels measured equal the values measured in the local tissues in
total hip arthroplasty metal-on-metal prostheses.
4 The serum ion levels measured are much lower in terms of their level to the values
measured in total hip arthroplasty metal-on-metal prostheses.
5 The serum ion levels measured are similar in terms of their level to the values
measured in total hip arthroplasty metal-on-metal prostheses.

PREFERRED RESPONSE 5

According to two studies looking at patients with a cobalt-chrome metal-on-metal


lumbar disk arthroplasty, serum ion levels in these patients were similar to values
measured in patients with total hip arthroplasty metal-on-metal prostheses.

(SAE12SN.64) Which of the flowing trajectories is preferred for placement of C1


lateral mass screws? Review Topic

1 7.5 degrees lateral and 5 degrees cephalad


2 7.5 degrees medial and 10 degrees cephalad
3 10 degrees medial and 22 degrees cephalad
4 10 degrees lateral and 22 degrees cephalad
5 20 degrees medial and 22 degrees caudal

PREFERRED RESPONSE 3

The C1 lateral mass can safely accommodate screw fixation. Trajectory of 10 degrees
medial and 22 degrees cephalad was safely applied in a series of 50 patients.
Postoperative CT scans confirmed the safe trajectory. The benefit of lateral mass
screws is that they can be safely placed despite the existence of an anomalous
vertebral artery that could preclude the safe placement of transarticular screws.
(SAE11OS.128) Figures 128a and 128b show the radiograph and MRI scan of a 74-
year-old woman with severe neck pain and upper extremity numbness, tingling, and
clumsiness. She also reports that she has balance problems and sustained a distal
radius fracture in a fall 6 months ago. Examination reveals hyperreflexia in bilateral
quadriceps and Achilles reflexes, bilateral Hoffman's signs, and eight beats of clonus
in both lower extremities. What is the best treatment option? Review Topic

1 Posterior laminectomy
2 Posterior laminoplasty
3 Posterior laminectomy and fusion
4 Cervical collar and observation
5 Combined anteroposterior decompression and fusion

PREFERRED RESPONSE 5

The patient has cervical spondylosis and symptomatic myelopathy. The radiograph
reveals multilevel spinal cord compression and, most importantly, a fixed kyphosis of
the cervical spine. In the setting of cord compression and kyphotic deformity, a
combined anteroposterior approach allows for ventral and dorsal decompression,
kyphosis correction, and stabilization. Observation in the setting of severe myelopathy
will likely lead to further disease progression. In the setting of cervical kyphosis,
posterior-only treatment options will not adequately address cord deformation and,
therefore, not improve symptoms as reliably.

(SAE12SN.72) What is the most common presenting problem in patients with cauda
equina syndrome? Review Topic

1 Urinary retention
2 Urinary incontinence
3 Saddle numbness
4 Lower extremity numbness and weakness
5 Back and leg pain

PREFERRED RESPONSE 5

In one recent retrospective cohort study of 42 patients with cauda equina syndrome,
83% had low back pain at presentation, 90% had radicular lower extremity pain, 60%
had urinary retention, and 55% had urinary incontinence. Objective findings at
presentation included 55% with leg weakness, 62% with sensory deficit, 62% with
absent ankle jerk reflexes, 76% with perianal sensory deficit, and 50% with decreased
rectal tone.

(SAE09SN.11) A 73-year-old woman reports a 4-month history of severe left-sided


posterior buttock pain and left leg pain. The leg pain radiates into the left lateral thigh
and posterior calf with cramping. Examination reveals mild difficulty with a single-
leg toe raise on the left side and a diminished ankle reflex. There is also a significant
straight leg raise test at 45 degrees which exacerbates symptoms. An MRI scan is
shown in Figure 4. What is the most appropriate treatment at this time? Review Topic

1 Lumbar laminectomy with synovial cyst excision


2 Repeat epidural steroid injection
3 Microdiskectomy at L4-5
4 Nonsteroidal medication and outpatient physical therapy
5 Left-sided facet blocks at L4-5 and L5-S1

PREFERRED RESPONSE 1

Lumbar spinal stenosis with lumbar radiculopathy can be commonly caused by a


synovial cyst arising from the facet joints. Lyons and associates reported on the
surgical treatment of synovial cysts in 194 patients. Of the 147 with follow-up data,
91% reported good pain relief and 82% had improvement of their motor deficits.
Epstein reported a 58% to 63% incidence of good/excellent results and a 38 to 42
point improvement on the SF-36 Physical Function Scale. It was also suggested that
since the presence of a synovial cyst indicates facet pathology, possible fusion should
be considered in these patients, especially those with underlying spondylolisthesis.

(SBQ12SP.20) Amphotericin B is most appropriate for the treatment of which type of


spine infection? Review Topic

1 Fungal osteomyelitis
2 Bacterial osteomyelitis with a gram-positive organism
3 Bacterial osteomyelitis with a gram-negative organism
4 Tuberculous osteomyelitis
5 Viral meningomyelitis

PREFERRED RESPONSE 1

Amphotericin B would be most appropriate for the treatment of fungal infections of


the spine.

Amphotericin B is a broad-spectrum anti-fungal medication. It is commonly used as


the first-line agent for treatment of fungal infections of the spine. The most common
fungi involving the spine include cryptococcus, candida, and aspergillus. The
indications for débridement and stabilization with spinal fusion, includes resistance to
antibiotic therapy, spinal instability, and/or neurologic deficits.

Kim et al. reviewed fungal infections of the spine. They comment that fungus
infections are most commonly spread by hematogenous or direct spread. Access to the
vascular system may include intravenous lines, during implantation of prosthetic
devices, or during surgery.

Frazier et al. retrospectively reviewed 11 patients with fungal osteomyelitis of the


spine. Nine of the patients were immunocompromised secondary to diabetes mellitus,
corticosteroid use, chemotherapy for a tumor, or malnutrition. All were treated with
anti-fungal medication. 10 of 11 patients were also treated with surgical debridement.
Paralysis secondary to the spine infection developed in eight patients. After an
average of 6.3 years of follow-up, the infection had resolved in all nine surviving
patients.

Illustration A shows the mechanism of action of Amphotericin. Illustration B shows


T1- (Image A and B) and T2-weighted (Image C) images of the lower thoracic and
lumbar spine. There are hypointense signals within the T12 and L1 vertebral bodies
(Images A and B) indicative of fungal osteomyelitis.

IncorrectAnswers:
Answer 2: Cefazolin, vancomycin may be used for the treatment of bacterial
osteomyelitis with gram-positive organisms.
Answer 3: Fluoroquinolone (e.g., ciprofloxacin) may be used for the treatment of
bacterial osteomyelitis with gram-negative organisms.
Answer 4: Combination treatment with isoniazid, ethambutol, rifampin, and
pyrazimide is used for the treatment of tuberculosis osteomyelitis.
Answer 5: Treatment of viral meningomyelitis will be dependent of the viral
organism.

(SAE08OS.75) Which of the following is a relative contraindication to performing


laminoplasty in a patient with cervical myelopathy? Review Topic

1 Ossification of the posterior longitudinal ligament (OPLL)


2 Cervical kyphosis measuring 20 degrees
3 30 degrees of sagittal mobility on flexion-extension views
4 Multilevel canal stenosis
5 Patient age older than 55 years

PREFERRED RESPONSE 2

Laminoplasty is one of the surgical options for decompressing the spinal cord in
patients with cervical myelopathy. An ideal candidate is a patient with preserved
cervical lordosis, as expansion of the canal relies on posterior drift of the spinal cord
to achieve decompression. Posterior decompression, such as laminectomy and
laminoplasty, are ideal for multilevel canal stenosis. Developed in Japan,
laminoplasty is commonly performed in patients with OPLL as it avoids the dangers
of working around the ossified posterior ligament. There is no age criterion for this
procedure. As it is a motion-preserving procedure, 30 degrees of flexion-extension is
not considered a contraindication.

(SAE09SN.90) Which of the following lumbar disk components has the highest
tensile modulus to resist torsional, axial, and tensile loads? Review Topic

1 Nucleus pulposus
2 Cartilaginous end plate
3 Anterior longitudinal ligament
4 Annulus fibrosis
5 Cellular matrix

PREFERRED RESPONSE 4

The annulus fibrosis has a multilayer lamellar architecture mode of type I collagen
fibers. Each successive layer is oriented at 30 degrees to the horizontal in the opposite
direction, leading to a “criss-cross” type pattern. This composition allows the annulus,
which has the highest tensile modulus, to resist torsional, axial, and tensile loads.

(SAE10BS.20) Following spinal cord injury (SCI), methylprednisolone (a bolus of 30


mg/kg plus 5.4 mg/kg per hour over 24 hours) initiated within 8 hours of injury has
been associated with which of the following MRI findings? Review Topic

1 Decreased extent of cord edema only


2 Decreased extent of cord hemorrhage only
3 Decreased extent of both cord edema and hemorrhage
4 Increased extent of cord hemorrhage but decreased edema
5 No difference in extent of hemorrhage and edema

PREFERRED RESPONSE 2

MRI findings following SCI treated with high dose steroids have demonstrated that
the steroids are associated with a lower extent of cord hemorrhage. No difference in
cord edema was found. The decreased cord hemorrhage seen with use of high dose
steroid adminstration in this setting has not correlated with improved clinical
outcomes.

(SAE12SN.36) What is the most common non-anesthetic-related reversible cause of


sustained changes in intraoperative neurophysiologic monitoring signals during spinal
surgery? Review Topic

1 Pedicle screw misplacement


2 Patient positioning
3 Spinal cord ischemia
4 Retractor placement
5 Hypertension
PREFERRED RESPONSE 2

Patient positioning that results in local nerve compression, plexus traction, or


improper neck alignment is the most common non-anesthetic-related cause of changes
in intraoperative neurophysiologic monitoring data during spinal surgery. Pedicle
screw malpositioning, spinal cord ischemia, and retractor placement are all less
common causes. Hypotension, not hypertension, can be a cause of intraoperative
neurophysiologic changes.

(SAE12SN.66) Which of the following is a significant risk factor for airway


complications after anterior cervical surgery? Review Topic

1 Smoking history
2 Pulmonary disease
3 Absence of drainage from surgical drains
4 Surgical time of more than 5 hours
5 Myelopathy

PREFERRED RESPONSE 4

In a study of 311 patients undergoing anterior cervical surgery only, a surgical time of
more than 5 hours and exposure of four or more vertebral bodies involving C4 or
higher were found to be risk factors for postoperative airway complications.
Surprisingly, preoperative pulmonary status, smoking history, absence of drainage,
and myelopathy were not associated with airway complications.

(SAE09SN.33) A 42-year-old woman is brought to the emergency department


following a motor vehicle accident. She has sustained multiple injuries, and she is
intubated and pharmacologically paralyzed. Sagittal cervical CT scans through the
right cervical facets, the left cervical facets, and the midline are shown in Figures 12a
through 12c, respectively. Definitive management of her cervical injury should
consist of Review Topic
1 anterior diskectomy and fusion at C4-C5.
2 immobilization in a Philadelphia collar and voluntary flexion and extension
radiographs when awake.
3 occipital-cervical fusion with instrumentation.
4 halo immobilization for 12 weeks.
5 left C6 superior facetectomy and posterior fusion at C6-C7 with instrumentation.

PREFERRED RESPONSE 3

The CT scans reveal an occipital-cervical dissociation with subluxation of the


occipitocervical joints bilaterally. Definitive management should consist of an
occipital-cervical fusion with instrumentation. Immobilization in a Philadelphia collar
is inadequate for this highly unstable injury, and halo immobilization, while affording
adequate temporary immobilization, is not appropriate definitive management for this
ligamentous injury. The patient does not have an injury at C4-C5 or C6-C7.

(SAE12SN.21) Which of the following indicates resolution of a postoperative wound


infection? Review Topic

1 C-reactive protein (CRP) has normalized and erythrocyte sedimentation rate (ESR)
is improving
2 CRP and ESR remain elevated
3 CRP and ESR are below normal
4 CRP has improved to the same degree as the ESR has improved
5 CRP remains elevated after the ESR has normalized

PREFERRED RESPONSE 1

Khan and associates in a retrospective review found that CRP was more responsive
and normalized with resolution of infection, whereas the ESR can remain elevated in
the presence of a normal CRP. Since the normal range of ESR and CRP usually
begins at 0, a level that is below normal is not likely to be found.
(SAE12SN.62) A 38-year-old man reports right upper extremity pain that radiates
from his neck to his anterior arm, dorsoradial forearm, and into the index finger.
Examination reveals weakness of the biceps muscle group and loss of his
brachioradialis reflex on that side. At which level is he most likely to have a right-
sided cervical disk protrusion on an MRI scan? Review Topic

1 C4-C5
2 C5-C6
3 C6-C7
4 C6 vertebral body
5 Far lateral C6-C7

PREFERRED RESPONSE 2

The patient has a typical right C6 radiculopathy based on his history and physical
examination. A posterolateral disk protrusion at the C5-C6 level is mostly likely to
cause a C6 radiculopathy because the C6 nerve roots exit just above the C6 pedicle
and therefore would be compressed by a right-sided C5-C6 disk protrusion. In
contrast to the lumbar spine, far lateral disk protrusions are not typically described in
the cervical spine.

(SAE12SN.90) A 78-year-old man is seen in the emergency room 3 hours after a fall
from a standing position. The patient sustained a mild scalp laceration and the injury
shown in Figure 90. He reports severe neck pain and is unable to move his hands and
legs. Examination reveals absent motor function in the wrist flexors, triceps, and
fingers. He cannot move his lower extremities during motor testing. The patient has
some sensation in the lower extremities. Bulbocavernosus reflex is absent. Based on
examination findings and the imaging findings, what is the most definitive treatment
option? Review Topic
1 Closed reduction and immobilization in a halo-thoracic vest
2 Halo application and cervical traction for 6 weeks, followed by 8 weeks of
immobilization in a halo-thoracic vest
3 Open reduction, decompression, and fusion with anterior-posterior stabilization
4 Open reduction, anterior decompression, and fusion
5 Uninstrumented posterior fusion spanning the injured segment

PREFERRED RESPONSE 3

The patient has a hyperostotic condition of the cervical spine, most likely ankylosing
spondylitis. Because of a rigid and osteoporotic spine, relatively minor falls can result
in unstable spinal injuries with significant instability and a high risk for neurologic
sequelae. The patient has an unstable injury at C6 with an incomplete spinal cord
injury, necessitating urgent decompression and stabilization. Studies have shown that,
in patients with ankylosing spondylitis, stand-alone anterior stabilization results in a
high failure rate. Halo-thoracic vests carry a high risk of septic and pulmonary issues,
especially in the elderly. Uninstrumented fusion will provide insufficient stability in
such patients.

(SBQ13PE.22) A 11-year-old male is referred for evaluation of scoliosis by his


primary care physician. He has a normal birth and development history and denies
any neurologic deficits or pain. On physical examination, he is neurologically intact
with normal reflexes and tone. A PA radiograph is shown in Figure A. What is the
next best step? Review Topic
1 Custom orthosis prescription to initiate bracing
2 Physical therapy referral and observation
3 Bending and lumbar oblique radiographs
4 Total spine CT
5 Total spine MRI

PREFERRED RESPONSE 5

A left thoracic curve is an abnormal finding and warrants further work-up with a total
axis MRI in order to rule out concomitant neurologic abnormalities such as a spinal
cord cyst and/or syrinx.

In adolescent idiopathic scoliosis (AIS), the most commonly occurring curve is a right
thoracic curve. Left thoracic curves are not as common, and warrants total axis MRI
in order to rule out concomitant central axis abnormalities. This is imperative not only
during initial work-up, but most importantly for operative planning.

Spiegel et al. performed a a retrospective radiographic review on 41 patients with


scoliosis associated with a Chiari I malformation and/or syringomyelia.
Approximately 50% of patients had an "atypical" pattern (left thoracic, double
thoracic, triple, long right thoracic). The authors recommend that MRI should be
considered in these patients.

Gillingham et al. provides a thorough review of early onset scoliosis and notes the
relatively high incidence of concurrent central axis abnormalities, even in patients
with normal neurologic exams. Rates have been reported upwards of 21.7%, with
malformations including Chiari Type 1, dural ectasias, syrinx, and spinal cord cysts.

Figure A exhibits a left thoracic curve. Further imaging in this patient revealed a
syrinx which required decompression.

Incorrect answers:
Answer 1: custom orthosis and non-operative treatment is not the best option, nor the
next best step.
Answer 2: Physical therapy will have no bearing on the treatment of this patients
thoracic curve.
Answer 3: Bending radiographs may be helpful further down the treatment line,
however, at this current time is not the next best step.
Answer 4: Total spine CT may help if any bony abnormalities or failure to formations
are noted, however, those are more typical in congenital or very early onset cases and
is not the next best step here.

(SAE11OS.65) Figures 65a and 65b show the MRI scans of a 33-year-old man with
severe left leg pain. He has had symptoms for 3 months with progressive worsening
pain and function. Examination reveals ankle plantar-flexor weakness and diminished
light touch sensation on the plantar surface of the foot. What treatment provides the
best outcome? Review Topic

1 Transforaminal epidural injection


2 Laminotomy and limited diskectomy
3 Laminotomy and complete diskectomy
4 Laminectomy and complete diskectomy
5 Laminotomy, diskectomy, and arthrodesis

PREFERRED RESPONSE 2

The patient's signs and symptoms are consistent with lumbar radiculopathy. Surgical
treatment for this condition has been shown to yield significantly improved outcomes
when compared with nonsurgical management. Surgical management is best
performed with a laminotomy and removal of the sequestered disk herniation
("limited diskectomy"). A complete (ie, subtotal) diskectomy may reduce the rate of
recurrence for disk herniation but has been shown to worsen back pain
postoperatively. A laminectomy may be necessary for larger herniations with severe
central stenosis; the patient does not meet those criteria and, as noted, a total
diskectomy is not indicated. Arthrodesis in the setting of primary lumbar disk
herniation is not indicated and is considered overly aggressive treatment.
(SAE11OS.78) Figures 78a and 78b show the CT scans of a 22-year-old man with
back pain after falling out of a tree. Examination reveals no palpable spinal step-offs,
posterior spinal pain, and normal neurologic function in the lower extremities. Normal
perineal sensation and normal rectal tone are present. What is the best management?
Review Topic

1 Bed rest
2 External orthosis
3 Anterior corpectomy and arthrodesis
4 Posterior instrumented arthrodesis
5 Posterior decompression and instrumented arthrodesis

PREFERRED RESPONSE 2

The patient has a stable L2 burst fracture. There is no evidence of neurologic injury or
disruption of the posterior ligamentous complex. According to the Thoracolumbar
Injury Classification System (TLICS), the severity score for this injury is 2 and
therefore nonsurgical management is recommended. The TLICS was developed to
define injury based on three clinical characteristics: injury morphology, integrity of
the posterior ligamentous complex, and neurologic status of the patient. Point values
are assigned to each major category based on injury severity. The sum of these points
represents the TLICS severity score, which may be used to guide treatment. The
injury scores are totaled to produce a management grade that is, in turn, used to guide
treatment. A score of >4 suggests the need for surgical treatment because of
significant instability, whereas a score of <4 suggests nonsurgical management. The
severity score offers prognostic information and is helpful in medical decision
making. An external orthosis provides enough support to obviate the need for bed rest
and avoid associated complications (deep venous thrombosis, pulmonary embolism,
pneumonia, skin ulceration). Surgical treatment, either through an anterior or
posterior approach, has been shown by Wood and associates to result in increased
pain and disability and is therefore not indicated in this setting. Additionally, there is
no need for decompression in the setting of a neurologically intact patient.
(SAE12SN.93) Figure 93 shows the axial T2-weighted MRI scan of the lumbar spine
of a 70-year-old man. The arrow points to which of the following structures? Review
Topic

1 Lamina
2 Facet joint
3 Lumbar synovial cyst
4 Ligamentum flavum
5 Epidural space

PREFERRED RESPONSE 4

The ligamenta flava (singular, ligamentum flavum, Latin for yellow ligament) are
ligaments that connect the laminae of adjacent vertebra, all the way from the axis to
the first segment of the sacrum. In T2-weighted sequencing, ligamentous structures
possess a low signal intensity. The ligamentum in this patient is markedly thickened,
resulting in severe spinal stenosis. The epidural space lies ventral and medial to the
ligamentum flavum and should possess a high signal intensity secondary to the
presence of cerebrospinal fluid. However, in the case of high-grade stenosis, there
may be little if any cerebrospinal fluid present, making the epidural space and central
canal difficult to identify. A lumbar synovial cyst should also have high signal
intensity because of the presence of synovial fluid.

(SBQ12SP.14) A 36-year-old male presents with acute onset of right buttock and leg
pain following lifting a heavy object. On physical exam he has weakness to knee
extension, numbness over the medial malleolus, and a decreased patellar reflex.
Which of the following would most likely explain this clinical presentation. Review
Topic
1 Lumbar arachnoiditis
2 L4/L5 paracentral disc herniation
3 L3/L4 far lateral (foraminal) disc herniation
4 L4/L5 far lateral (foraminal) disc herniation
5 L5/S1 far lateral (foraminal) disc herniation

PREFERRED RESPONSE 4

The clinical presentation is consistent with a L4 radiculopathy. A L4/L5 far lateral


(foraminal) disc herniation would compress the exiting root (L4) and cause these
symptoms.

The location of a prolapsed lumbar disc determines its symptoms. Central disc
herniations may give rise to back pain or cauda equina syndrome. Paracentral disc
herniations (90-95% of cases) affect the traversing nerve root. Far lateral disc
herniations (5-10%) affect the exiting nerve root.

Gregory et al. summarize physical signs in lumbar disc herniation. They state that the
straight-leg-raise is the most sensitive (73-98% sensitive) test and the crossed straight-
leg-raise is the most specific (88-98% specific) test for lumbar disc herniation. Other
specific tests include weak ankle dorsiflexion (89% specific), absent ankle reflex
(89% specific), and calf wasting (94% specific, but a late finding).

Illustration A shows how a paracentral L4/L5 disc herniation affects the traversing L5
root, but a far lateral L4/L5 disc herniation affects the L4 root. Illustration B shows
the dermatomal distribution of pain with root involvement from L3 to S1.

Incorrect Answers:
Answer 1: Lumbar arachnoiditis will not give rise to radiculopathy. This patient did
not have invasive spinal procedures that would put him at risk of this condition.
Symptoms of arachnoiditis include chronic pain, numbness and tingling of the
extremities, abnormal bowel, bladder and sexual function.
Answer 2: L4/L5 paracentral disc herniation would affect the traversing root (L5) and
give rise to L5 radiculopathy.
Answer 3: L3/L4 far lateral (foraminal) disc herniation would affect the exiting root
(L3) and give rise to L3 radiculopathy.
Answer 5: L5/S1 far lateral (foraminal) disc herniation would affect the exiting root
(L5) and give rise to L5 radiculopathy.

(OBQ06.17) In the treatment of thoracolumbar idiopathic scoliosis using an anterior


single rod technique with interbody cages, which of the following variables has been
associated with pseudoarthrosis. Review Topic
1 Thoracic curve coronal correction of > 40%
2 Thoracolumbar/lumbar curve coronal correction > 50%
3 Smaller adolescents (<50 kg)
4 Failure to maintain lumbar lordosis of > 45 degrees
5 Thoracic hyperkyphosis (>40 degrees )

PREFERRED RESPONSE 5

In select patients with thoracolumbar idiopathic scoliosis, an anterior approach with a


single rod and interbody cages may be indicated. Thoracic hyperkyphosis (>40
degrees ) is a risk factor for pseudoarthrosis in patients treated with this method.

In a prospective study, Sweet et al found anterior instrumented fusions using a single


solid rod had good radiographic and clinical outcomes. In their treatment group they
found common risk factors for pseudarthrosis were smoking, weight >70 kg, and T5-
T12 hyperkyphosis of > 40 degrees. They recommend consideration should be given
to alternate techniques in larger adolescents (>70 kg) with thoracic hyperkyphosis
(>40 degrees ). The average coronal correction of thoracic curves was from 55
degrees to 29 degrees (47%). The average correction of thoracolumbar/lumbar curves
was from 50 degrees to 15 degrees (70%). Neither of these variables were associated
with pseudoarthrosis. In the sagittal plane, lordosis was maintained in
thoracolumbar/lumbar fusions at -58 degrees (T12-sacrum). Improved maintenance of
lumbar lordosis is considered one of the advantages of an anterior approach.

In an additional study from the same group at Wash U, Hurford et al designed a study
to compare the results of anterior DUAL-rod instrumentation with their previous
experience using single-rod constructs. They found the two technique were
comparable in the amount of radiographic deformity correction obtained. However,
they report the absence of any pseudarthroses in the 60 patients with dual-rod is a
distinct advantage over the single rod technique.

(SAE09SN.32) A 38-year-old man reports a 2-week history of acute lower back pain
with radiation into the left lower extremity. There is no history of trauma and no
systemic signs are noted. Examination reveals a positive straight leg test at 35 degrees
on the left side and a contralateral straight leg raise on the right side. Motor testing
demonstrates mild weakness of the gluteus medius and weakness of the extensor
hallucis longus of 3+/5. Sensory examination demonstrates decreased sensation along
the lateral aspect of the calf and top of the foot. Knee and ankle reflexes are intact and
symmetrical. Radiographs demonstrate no obvious abnormality. MRI scans show a
posterolateral disk hernation. The diagnosis at this time is consistent with a herniated
nucleus pulposus at Review Topic

1 L1-2.
2 L2-3.
3 L3-4.
4 L4-5.
5 L5-S1.

PREFERRED RESPONSE 4

The patient’s history and physical examination findings are consistent with a lumbar
disk herniation at the L4-5 level. Weakness of the extensor hallucis longus and
gluteus medius are consistent with an L5 lumbar radiculopathy. Nerve root tension
signs are also consistent with sciatica from a lumbar disk herniation. The MRI scans
confirm a posterolateral disk herniation at L4-5, which typically affects the exiting L5
nerve root.

(SAE12SN.12) An 80-year-old man with a history of chronic obstructive pulmonary


disease (COPD) and dementia is involved in a fall from standing height, striking his
forehead. He is seen in the emergency department with predominantly mechanical
neck pain but no obvious neurologic deficits. Radiographs reveal a nondisplaced type
II odontoid fracture. What is the most appropriate treatment? Review Topic

1 Immobilization in a rigid cervical orthosis for 6 to 8 weeks


2 Posterior occipital-cervical fusion with iliac crest bone graft
3 Open reduction and internal fixation of the odontoid process with an anterior
odontoid screw
4 Resection of the odontoid process through a transoral approach
5 Halo skeletal fixation

PREFERRED RESPONSE 1

The treatment options for a type II odontoid fracture include halo immobilization,
odontoid screw fixation, and posterior atlantoaxial arthrodesis. However, surgical care
at this time without attempting nonsurgical management is not warranted; therefore,
the most appropriate management at this time is immobilization in a rigid cervical
orthosis for 6 to 8 weeks. Halo vest fixation can lead to high healing rates but is
generally contraindicated in elderly patients, especially one with COPD and dementia.
Posterior surgical fusion techniques provide high fusion rates, but do so at the expense
of loss of cervical rotation and surgical complications. Resection of a nondisplaced
odontoid fracture without cord compression via a transoral approach is not necessary.
(SAE09SN.9) A 60-year-old man is evaluated in the ICU after a rollover motor
vehicle accident 3 days ago. He has multiple upper and lower extremity trauma and
was found unresponsive at the accident scene. Surgery is planned for the extremity
trauma once the patient is medically stable. He remains intubated and the cervical
spine is immobilized in a semi-rigid collar. Examination reveals mild erythema in the
posterior occipital cervical region. Initial AP and lateral radiographs of the cervical
spine have not revealed any obvious fracture. What is the most appropriate treatment
option at this time? Review Topic

1 Continued semi-rigid immobilization until the extremity surgeries are completed


2 Halo skeletal fixation prior to the extremity surgery
3 Definitive clearance of the cervical spine with CT and/or MRI
4 Removal of the semi-rigid collar and physical examination when the patient is
responsive
5 Soft collar immobilization and local wound care

PREFERRED RESPONSE 3

Ackland and associates demonstrated that the failure to achieve early spinal clearance
in an unconscious blunt trauma patient predisposed the patient to increased morbidity
secondary to the prolonged used of cervical immobilization. They demonstrated that
the four significant predictors of collar-related ulcers were ICU admission,
mechanical ventilation, the necessity for cervical MRI, and the time to cervical spine
clearance and collar removal. The risk of pressure-related ulceration increased by
66% for every 1-day increase in Philadelphia collar time and this highlights the need
for definitive C-spine clearance.

(SAE08OS.69) Figures 21a and 21b show a transverse MRI scan at L4/5 and a lateral
radiograph of a 75-year-old man with a 2-year history of worsening low back and
bilateral leg pain that occurs with ambulation beyond approximately a half a block.
Management consisting of physical therapy, anti-inflammatory medications, and
muscle relaxants has failed to successfully resolve his symptoms. He has minimal
medical comorbidities. What is the best treatment approach for this patient? Review
Topic
1 Initiation of lumbar flexion and core strengthening exercises
2 Laminectomy and posterior spinal fusion of L4/5
3 Laminectomy of L4 without fusion
4 Percutaneous endoscopic diskectomy
5 A course of three epidural steroid injections

PREFERRED RESPONSE 2

The patient has spinal stenosis and degenerative spondylolisthesis. Laminectomy and
posterior spinal fusion have been demonstrated to be superior to both nonsurgical
management and laminectomy without fusion in prospective randomized studies. The
effect of instrumentation on improvement of clinical outcomes remains a matter of
debate. This patient has exhausted physical therapy approaches per the history.
Epidural steroid injections have not been shown to significantly alter the natural
history of patients with degenerative lumbar spinal stenosis and spondylolisthesis.
While minimal access decompression has been described, percutaneous endoscopic
diskectomy would be more appropriate for a disk herniation.

(SAE10BS.57) A 35-year-old man with a history of spine surgery 5 years ago reports
the recent development of frequent low back pain radiating to the legs. The patient
blames a low-energy fall that occurred 9 months ago for the recent symptoms.
Radiographs reveal previous interbody fusions of L4-L5 and L5-S1, with hardware
present. The vertebrae appear well-fused and stable. What is the most likely cause of
the low back pain? Review Topic

1 Increased range of motion and strains at L3-L4


2 A herniated disk due to the fall
3 A compression fracture
4 Development of pseudarthrosis due to failure of the previous fusions
5 Loosening of the hardware that was used for the previous fusions
PREFERRED RESPONSE 1

It is increasingly recognized in the spine literature that fusion of two or more


vertebrae produces increased range of motion in adjacent motion segments, resulting
in increased stresses and, with time, degeneration of the adjacent disks. Loss of disk
height would be an initial indication of disk degeneration. If the patient's fall had
caused a fracture, the symptoms would have been immediate.

(SAE12SN.71) What is the most common physical finding in myelopathic patients?


Review Topic

1 Babinski sign
2 Hoffman sign
3 Hyperreflexia
4 Clonus
5 Ataxic gait

PREFERRED RESPONSE 2

The Hoffman sign is the most common finding, occurring in 80% of myelopathic
patients, and it is more common with increasing severity of the myelopathy. The
prevalence of hyperreflexia has been shown to be no different from the prevalence in
individuals without myelopathy. Sustained clonus and Babinski signs have been
shown to occur in only one third of the patients with myelopathy.

(SAE09SN.40) In patients without spondylolisthesis or scoliosis undergoing


laminectomy for lumbar spinal stenosis, spinal fusion is generally recommended if
Review Topic

1 a dural tear is repaired.


2 more than one level requires decompression.
3 less than one half of each facet is removed bilaterally.
4 the pars interarticularis is fractured.
5 the patient is a smoker.
PREFERRED RESPONSE 4

With the notable exception of fusion for degenerative spondylolisthesis and scoliosis,
there is a paucity of evidence on the indications for spinal fusion in patients
undergoing laminectomy for spinal stenosis. However, it is generally recommended
that if the spine is destabilized (for example by removal of one complete facet joint or
by an iatrogenic pars fracture), spinal fusion should be considered. Although fusion
can be considered for a very long laminectomy, a two-level laminectomy does not
represent, by itself, a clear indication for the addition of a spinal fusion. The repair of
a dural tear and the use of nicotine by the patient play no role in the determination of
whether or not to add fusion to a laminectomy procedure.

(OBQ14.194) A 63-year-old male, with history of myocardial infarction, presents


with buttock and leg pain. He states the pain is worse when climbing stairs, and is
absent when walking down a hill. He reports when walking on a flat surface the pain
begins after roughly 50 meters, but if he stops walking and remains standing upright,
the pain resolves after a few minutes. He denies any leg pain when sitting and driving
a car. These symptoms are most consistent with: Review Topic

1 Neurogenic claudication
2 Vascular claudication
3 Lumbar radiculopathy
4 Cervical myelopathy
5 Central cord syndrome

PREFERRED RESPONSE 2

This clinical presentation is most consistent with vascular claudication.

Vascular claudication refers to the pain, aching or fatigue of the muscles of the
buttocks, thigh and/or calf that occurs with exertion, and is related to a failure to meet
muscular oxygen requirements, usually caused by peripheral vascular disease
impeding blood flow to the peripheral muscles.

Neurogenic claudication is the classic symptom caused by lumbar spinal stenosis.


Neurogenic claudication classically presents with bilateral buttock pain with upright
activities, but seems to improve by postural changes that flex the lumbar spine. These
posture changes are thought to increase the cross sectional area of the central canal,
which relieves pressure on the affected area.

Issack et al. reviewed degenerative lumbar spinal stenosis. They state that patients
with vascular claudication will have similar symptoms of leg cramping, whether
ambulating or riding a stationary bicycle. In comparison, patients with neurogenic
claudication have diminished symptoms of claudication while positioned seated.

Young et al. reviewed the use of lumbar epidural/transforaminal steroids for


managing spinal disease. They report that two thirds of acute low back pain episodes
resolve within 7 weeks, so the utility and practice patterns regarding the timing and
number of epidural/transforaminal steroid injections is usually based on expert
opinion, rather than high level research evidence.

Illustration A shows an angiogram of a patients with normal (left) vs abnormal (right)


arterial vasculature. Illustration B shows the typical MRI of a patient with spinal
stenosis.

Incorrect Answers:
Answer 1: Walking tolerance less than 2 blocks is consistent with both neurogenic
from vascular claudication. In neurogenic claudication, pain is improved with a flexed
forward trunk, which requires patients to sit for relief, not stand.
Answer 3: With lumbar radiculopathy, buttock and leg pain would be worse when
driving a car.
Answer 4: Patients would complain of an unsteady gait and hand weakness with
advanced cervical myelopathy.
Answer 5: Central cord syndrome patient would present with motor impairment that is
disproportionately greater in the upper extremities compared to the lower.

(SAE09SN.57) Figures 23a and 23b show the MRI scans of a 50-year-old woman
who has increasing gait disturbance. She reports three falls in the past week.
Examination reveals hyperreflexia, motor weakness in the biceps and triceps, and a
positive Hoffman’s sign. What is the most appropriate treatment plan? Review Topic

1 Observation
2 Physical therapy
3 Epidural steroid injections
4 Cervical laminectomy
5 Anterior cervical diskectomy and fusion
PREFERRED RESPONSE 5

The patient has obvious signs of progressive myelopathy. Based on her significant
physical examination findings, nonsurgical management will not significantly impact
her outcome. Cervical decompression alone is contraindicated in patients with
cervical kyphosis such as seen here. Anterior cervical fusion is the best option.

(SBQ12SP.77) A 68-year-old is undergoing lateral lumbar interbody fusion using the


tranpsoas approach. Which of the following statements is true regarding the safe
approach zone for this procedure as you move cranial to caudal in the lumbar spine?
Review Topic

1 Safe approach zone increases due to vessels moving more ventral


2 Safe approach zone increases due to lumbar plexus moving more dorsal
3 Safe approach zone decreases due to vessels moving more dorsal
4 Safe approach zone decreases due to lumbar plexus moving more ventral
5 Safe approach zone remains the same throughout the lumbar spine

PREFERRED RESPONSE 4

As you move cranial to caudal in the lumbar spine, the safe approach zone for the
lateral transpsoas approach decreases due to the more ventral position of the lumbar
plexus.

Lateral lumbar interbody fusion has become more common for degenerative spine
disorders and adjacent segment degeneration. This transpsoas approach is typically
useful for pathology from L1-L2 disc space to the L4-L5 disc space and places the
lumbar plexus at risk. Working at the more caudal disc spaces is especially difficult
given the more ventral position of the plexus, but the use of triggered EMG retractors
and probes can help prevent nerve injuries. Surgical approach can be especially
difficult in patients with rotational deformities.

Benglis et al. did a cadaver study with specimens placed lateral to trace the course of
the lumbar plexus. They found that the plexus move more ventral with respect to the
disc space moving more caudal in the lumbar spine.

Park et al. used 10 cadaver specimens to measure the distance of the lumbar nerve
roots from the center of the disc space in the lateral approach. While disc space access
was generally safe, there was less distance to the nerve root for more caudal disc
levels.

Regev et al. did a MRI study to evaluate the safe working corridor for the lateral
approach. The safe zone narrows considerably in the L4-L5 disc space due to more
ventral position of the nerve roots, and they recommend careful monitoring when
addressing this level.

Incorrect Answers:
Answers 1, 2, and 5: The safe approach zone decreases moving from cranial to caudal
in the lumbar spine
Answer 3: While the safe approach zone does decrease, the reason is due to a more
ventral position of the lumbar plexus. The position of the great vessels remains
relatively unchanged.

(OBQ08.178) A 66-year-old female presents to your clinic complaining of back pain,


difficulty standing-up straight, weakness in her legs, and neurogenic claudication. On
upright thoracolumbar radiographs, there is a 75 degree thoracolumbar curve with the
apex at L2, and the C7 plumb line falls 12 cm anterior to the posterosuperior corner of
S1. Aside from a decompression of the stenotic levels, which of the following choices
will lead to the MOST reliable decrease in overall disability? Review Topic

1 Ensuring the lumbar lordosis is within 15 degrees of the pelvic incidence


2 Decreasing the cobb angle to less than 25 degrees
3 Correcting the sagittal vertical axis to +3 cm from neutral
4 Increasing the pelvic tilt to greater than 20 degrees
5 Stopping the fusion at L5

PREFERRED RESPONSE 3

This patient has a spinal deformity in both the coronal and sagittal planes. Among the
options given, correction of the sagittal vertical axis (SVA) to +3 cm is the most
reliable predictor of clinical improvement.

Spinal malalignment in Adult Spinal Deformity (ASD) challenges balance


mechanisms used for maintenance of an upright posture to achieve the basic human
needs of preserving level visual gaze and retaining the head over the pelvis. Severe
malalignment can result in greater muscular effort and energy expenditure to maintain
the erect posture as well as use of compensatory mechanisms. As such, surgical
correction of these deformities are aimed at achieving proper spinopelvic alignment.

Glassman et al. performed a multi-center retrospective study of 298 adults with spinal
deformity. Regardless of operative (129 patients) or non-operative care (172 patients)
a positive sagittal balance was the found to be the most reliable predictor of clinical
symptoms in both patient groups.

Schwab et al. published a current concepts review on operative management for adult
spinal deformities and identified three major goals of surgery: (1) Correct the SVA to
within 5 cm of neutral, (2) Ensure the pelvic tilt is less than 20 degrees, (3) Ensure the
lumbar lordosis is within 9 degrees of the pelvic incidence.

Illustration A demonstrates how to measure the SVA. Illustration B depicts the


realignment objectives in the saggital plane as described by Schwab et al.

Incorrect Answers:
Answer 1: Ensuring that the lumbar lordosis is within 9 degrees of the pelvic
incidence has been shown to be a reliable predictor of clinical outcome.
Answer 2: The amount of coronal correction has not been shown to reliably affect
outcomes in adult spinal deformity patients.
Answer 4: Ensuring the pelvic tilt is less (not greater) than 20 degrees has been shown
to be a reliable predictor of clinical symptoms.
Answer 5: Stopping a large thoracolumabr fusion at L5 has not been shown to
decrease disability in adult deformity patients. Extending the fusion to the sacrum can
improve correction and maintenance of sagittal balance.

(SAE12SN.48) Posterior lumbar spine arthrodesis may be associated with adjacent


segment degeneration cephalad or caudad to the fusion segment. Which of the
following is the predicted rate of symptomatic degeneration at an adjacent segment
warranting either decompression and/or arthrodesis at 5 to 10 years after lumbar
fusion? Review Topic

1 5.5% at 5 years and 75.5% at 10 years


2 10.1% at 5 years and 65.2% at 10 years
3 16.5% at 5 years and 36.1% at 10 years
4 26.5% at 5 years and 56.1% at 10 years
5 49.2% at 5 years and 15.1% at 10 years

PREFERRED RESPONSE 3

Ghiselli and associates described a rate of symptomatic degeneration at an adjacent


segment warranting either decompression or arthrodesis to be 16.5% at 5 years and
36.1% at 10 years based on a Kaplan-Meier analysis.

(SAE09SN.87) Figure 31 shows the radiograph of a 64-year-old woman who is seen


in the emergency department following a motor vehicle accident. She has no
voluntary motor function in her distal upper extremities or lower extremities. She
does not have a bulbocavernosus reflex. She has a blood pressure of 80/50 mm Hg
with a pulse of 50/min. Her hypotension does not improve with initial fluid
resuscitation. Further treatment of her hypotension should consist of Review Topic

1 continued rapid fluid infusion.


2 administration of broad-spectrum antibiotics.
3 administration of 30/mg/kg methylprednisolone over 1 hour.
4 administration of pressors.
5 cardioversion and implantation of a pacemaker.

PREFERRED RESPONSE 4

The hallmark of neurogenic shock is hypotension without tachycardia. It is associated


most commonly with high cervical spinal cord injuries and results from loss of
function of the sympathetic nervous system. Because the peripheral vasculature is
dilated due to loss of its sympathetic tone, continued rapid administration of fluid
corrects the hypotension and can quickly lead to fluid overload and congestive heart
failure. Therefore, neurogenic shock is best treated by the use of pressors.
Cardioversion or administration of antibiotics or systemic steroids is not appropriate
treatment for this patient’s hypotension.

(OBQ13.61) The 'Risser sign' is one of the most commonly used markers for skeletal
maturation and growth potential in patients with adolescent idiopathic scoliosis. What
'Risser sign' has been shown to correlate with the greatest velocity of skeletal linear
growth? Review Topic

1 Risser 0
2 Risser I
3 Risser II
4 Risser III
5 Risser IV

PREFERRED RESPONSE 1

There are two stages of life where the velocity of postnatal skeletal growth is most
rapid. These are: (1) during the first year of life and (2) puberty. Both correlate with a
Risser sign of 0. Risser 0 covers the first 2/3rd of the pubertal growth spurt and
correlates with the greatest velocity of skeletal linear growth.

Risser grades range from 0 to V and are a measure of the progression of ossification
in the pelvis. The Risser sign is usually referenced in clinical decision-making
regarding adolescent idiopathic scoliosis.

Biondi et al. examined 111 patients to determine the relationship between the
accuracy of the Risser sign and bone age determinations. They found that the iliac
crest apophysis maturation correlated with skeletal age assessment. They suggest that
Risser sign is a reliable method for assessing skeletal bone age.

Illustration A shows an AP pelvis radiograph of a patient that is a Risser II based on


the iliac crest apophysis maturation scale. Illustration B shows a chart showing the
rate of growth correlated with the Risser staging system. Illustration C shows the
Risser staging system from 0 - V.

Risser I: is given when the ilium calcification measures 25%. As this stage the
velocity of linear skeletal growth is on a descending slope.

Risser II: the greater trochanteric apophysis unites with the femur and Ilium
calcification measures 50%. There is usually 3 cm of sitting growth remaining and no
further growth in the lower extremities.

Risser III: Ilium calcification measures 75%. There is usually 2 cm of sitting growth
remaining.

Risser IV: Ilium calcification is almost complete. There is usually 1 cm of sitting


growth remaining.

Risser V marks complete skeletal maturation.

Incorrect Answers:
Answers 2-5: During Risser 0, the velocity of linear skeletal growth has peaked.
Between Risser I through IV the rate of growth progressively declines until growth
stops in Risser V.
(SAE11AN.18) The most common neurologic injury following an anterior cervical
diskectomy and fusion (ACDF) is injury to which of the following structures? Review
Topic

1 Recurrent laryngeal nerve


2 Superior laryngeal nerve
3 C5 root
4 Spinal cord
5 Sympathetic chain

PREFERRED RESPONSE 1

The most common neurologic injury in ACDF is injury to the recurrent laryngeal
nerve. It is most vulnerable on the right because it crosses from lateral to midline
more cephalad in the incision after it passes under the subclavian artery; conversely,
on the left the course is more caudal because it passes under the aortic arch, a more
caudal structure. The superior laryngeal nerve runs along with the superior thyroid
artery in the upper cervical spine, putting it at risk during surgical procedures on the
upper cervical spine which are less commonly performed. A C5 root palsy more
commonly occurs as a result of multilevel posterior decompressive procedures,
possibly because of its short transverse take-off from the cord. The sympathetic chain
lies on top of the longus colli and can be injured if retractors are not placed under the
longus colli muscle.

(OBQ13.129) A 13-year-old premenarchal girl presents with back pain and scoliosis.
Figures A through D are the bone scan, SPECT scan, axial and coronal images
respectively. What is the most likely diagnosis? Review Topic
1 Metastasis
2 Aneurysmal bone cyst
3 Osteoblastoma
4 Osteoid osteoma
5 Eosinophilic granuloma

PREFERRED RESPONSE 4

This patient has osteoid osteoma (OO).

OO and osteoblastoma (OB) of the spine commonly present in the 2nd decade of life.
CT shows a low attenuation nidus with central mineralization and varying degrees of
perinidal sclerosis. NSAIDS are thought to inhibit PGE 2 and PGI 2 produced within
the nidus, and suppress perinidal edema arising from high levels of COX2 expression
in neoplastic nidal osteoblasts.

Burn et al. reviewed pediatric OO and OB of the spine. OB is more likely to have
neurological deficit because of spinal canal encroachment. Surgery involves complete
resection. They recommend onlay fusion when the facet and pedicle have been
resected, and instrumentation if resection of the vertebral body is carried out.

Jayakumar et al. discuss a case report of OO of the T6 vertebra that was treated
successfully with NSAIDS alone. OOs of the spine are the most common cause of
painful scoliosis in children and young adults. They caution against a non-operative
course in patients with an immature skeleton, significant skeletal deformity or with a
long delay before diagnosis.

Figures A and B are bone scan and SPECT scans respectively that demonstrate
intense tracer uptake in the left transverse process of L3 characteristic of an osteoid
osteoma. Dextroconvex scoliosis of the thoracic and levoconvex scoliosis of the
lumbar spine is noted. Figures C and D are axial and coronal reconstructed CT images
showing show a lesion of the left transverse process of L3 with a dense "nidus" with
surrounding lucency characteristic of osteoid osteoma. Illustration A is a table
showing the characteristics of pediatric epidural spinal tumors.

Incorrect Answers:
Answer 1: There is no suggestion of metastasis. While metastatic lesions may give
rise to a mixed blastic-lytic picture on CT, and may involve the posterior elements,
they are less likely to present with painful scoliosis than OO. They do not classically
have a sclerotic nidus.
Answer 2: Aneurysmal bone cysts are purely lytic without a sclerotic nidus, and are
unlikely to give rise to pain.
Answer 3: Osteoblastomas also occur in the posterior elements. Osteoblastomas are
larger, less ossified and are sometimes associated with secondary aneurysmal bone
cysts.
Answer 5: Eosinophilic granulomas arise from the vertebral body rather than posterior
elements and can produce vertebra plana. Lesions are purely lytic.

(SAE12SN.98) Thoracic disk herniations most typically occur at what level of the
thoracic spine? Review Topic

1 Upper third
2 Junction of upper third and middle third
3 Middle third
4 Junction of middle third and lower third
5 Lower third

PREFERRED RESPONSE 5

Most thoracic disk herniations occur in the lower (caudal) third of the thoracic spine.
This predilection may be related to the unique anatomic and biomechanical
environment of that region. The 11th and 12th ribs do not join the rib cage anteriorly
and do not form a true articulation with the transverse processes posteriorly.
Furthermore, flexion and torsional forces tend to concentrate between T10 and L1.

(OBQ15.72) Within the intervertebral disk, aggrecan is primarily responsible for:


Review Topic
1 Tensile strength of the nucleus pulposis
2 Tensile strength of the annulus fibrosis
3 Sensory transmission
4 Maintaining water content
5 Intermolecular cross-linking of collagen

PREFERRED RESPONSE 4

Aggrecan's primary function in the intervertebral disc is to maintain water content.

Aggrecan, the most abundant proteoglycan within the intervertebral disk, is a


hydrophillic molecule which helps attract and maintain water within the disc. This is
important to maintain compressive strength. Early in life aggrecan has a high
percentage of chondroitin sulfate chains. With aging this gradually changes to higher
percentage of keratin sulfate. Other proteoglycans include decorin and versican.
Decorin is believed to regulate cell cycles and influence fibrillogensis. Versican is a
large extra-cellular matrix proteoglycan involved in cell adhesion and migration.

Roughley et al. provide a review of biologic changes within the intervertebral disc
during development, maturation, and degeneration. They note that aggrecan works to
maintain water content within the disc, thus improving its compressive strength.
Aggrecan content decreases with age leading to a loss of compressive strength.

Illustration A shows the structure of aggrecan.

Incorrect answers:
Answer 1: Type II collagen is primarily responsible for tensile strength of the nucleus
pulposis.
Answer 2: Type I collagen is primarily responsible for tensile strength of the annulus
fibrosis.
Answer 3: Sensory transmission is thought to be conducted via neuropeptides such as
substance P.
Answer 5: Collagen within the intervertebral disc can under go cross-linking
enzymically (via molecules such as hydroxyallysin) or non-enzymically (via
molecules such as pentosadine).

(SAE11OS.163) Figures 163a through 163c show the radiograph and MRI scans of a
45-year-old woman with severe right arm pain. She has had symptoms for 6 months
without resolution despite multiple nonsurgical treatments. Examination reveals
weakness in the right triceps and wrist flexors with decreased sensation in the middle
finger and a positive Spurling's sign. What is the most appropriate treatment for the
patient's symptoms? Review Topic
1 Posterior laminoplasty
2 Posterior cervical foraminotomy
3 Anterior cervical foraminotomy
4 Anterior cervical diskectomy and arthrodesis
5 Anterior corpectomy and arthrodesis

PREFERRED RESPONSE 4

The patient has symptoms and signs of cervical radiculopathy despite a long course of
nonsurgical management. Therefore, surgical decompression is indicated and is best
performed through an anterior cervical diskectomy and arthrodesis. Single level
anterior cervical diskectomy and arthrodesis have been shown to produce significant
improvements in arm pain and neurologic function. Anterior cervical foraminotomy,
while reported, has insufficient data to support its use and it places the vertebral artery
at significant risk. Posterior cervical foraminotomy is contraindicated given the
ventral spinal cord compression; foraminotomy places the patient at risk for spinal
cord injury. The patient has one-level cervical disease, therefore a corpectomy is
unnecessary. Posterior laminoplasty is used to treat myelopathy, not radiculopathy.

(SAE12SN.57) A 28-year-old man has had a 2-week history of right posterior leg
pain, with numbness and tingling in the same distribution. He denies any problems
with bowel or bladder function. Examination shows intact motor strength in his
bilateral lower extremities, with numbness to light touch in the lateral border of his
right foot. Over the past 2 weeks, his leg pain has improved significantly. MRI scans
are shown in Figures 57a and 57b. What is the most appropriate course of
management? Review Topic
1 Referral to a pain clinic for epidural injections
2 Activity modification and anti-inflammatory medications
3 Urgent hospital admission followed by hemilaminotomy and diskectomy
4 Hemilaminotomy and diskectomy scheduled as an elective procedure
5 Posterior laminectomy and fusion

PREFERRED RESPONSE 2

The patient has an L5-S1 disk herniation, which has a favorable prognosis without
surgical intervention. Most acute lumbar disk herniations resolve with nonsurgical
management. The most appropriate course of initial treatment should be analgesics
and activity modification, followed by rehabilitation as the symptoms allow. Although
the MRI scan indicates a large disk herniation, he has no symptoms or signs that
would warrant urgent surgical decompression. Planned elective diskectomy should be
considered only if nonsurgical management fails to provide relief. Epidural injections
could be considered if the initial course of treatment fails to give the patient
significant relief. Posterior laminectomy and fusion is not indicated without the
presence of instability.

(SBQ12SP.9) A 62-year-old male underwent posterior spinal instrumented fusion for


degenerative lumbar spondylolithesis one year ago. He presents to office complaining
of persistent lower back pain. The pain initially improved but over the last 6 months
he has had recurring pain at the site of the surgery primarily with activity. He denies
back pain at rest or night pain. Physical examination reveals a well healed wound and
no physical abnormalities. He has no tenderness to palpation to the thoracic or lumbar
spine. He has no neurological deficits. His laboratory results show an erythrocyte
sedimentation rate (ESR) = 8 mm/h and C-reactive protein (CRP) = 3 mg/L at the last
visit which are both within normal limits. Figure A shows a series of radiographs
from his pre-operative, 3 month post-operative and 1 year post-operative clinic visits,
respectively. Which of the following investigations would best confirm the suspected
underlying diagnosis? Review Topic
1 MRI of lumbar spine
2 Repeat ESR/CRP and whole body bone scan
3 CT of lumbar spine
4 Dynamic flexion/extension plain film radiographs
5 Dynamic lateral bending plain film radiographs

PREFERRED RESPONSE 3

This patient has clinical and radiographic features of failed spinal arthrodesis. CT
scan have been shown to be the most relable method for assessing spinal fusion post-
operatively.

Reported rates of nonunion following posterior spinal fusion range from 0-70%. Rates
are variable due to the various operative techniques, underlying diagnoses as well as
asymptomatic patients. Non-union may lead to changes in alignment, spinal instability
and potential neurological injury. Static radiographs have long been used as a
practical method of fusion assessment, but these tend to significantly overestimate the
presence of a solid fusion. Lack of movement at a fused segment, on dynamic views,
does not confirm fusion. CT offers excellent bony resolution and are less affected by
metal artifact compared to MRI.

Patel et al. showed that tobacco use, malnutrition, oral anti-inflammatory use,
multilevel fusion, prior spine surgery, and sagittal imbalance are all risk factors for
spinal nonunion.

Mok et al. looked at erythrocyte sedimentation rate (ESR) and C-reactive protein
(CRP) levels after spinal surgery and compared their usefulness as predictors of
infectious complications in the early postoperative period. They found that CRP is
more applicable, predictable, and responsive in the early postoperative period
compared with ESR. Using a second rise or failure to decrease as expected for CRP is
sensitive for infection.

Shelby et al. reviewed the radiological assessment of spinal fusion. They comment
that fine-cut imaging, multiplanar reconstruction, and metal artifact reduction have
increased the ability to assess fusion on CT.

Figure A shows a series of lateral radiographs of the lumbar spine with posterior
spinal instrumentation. Illustration A shows a lateral CT scan image of multi-level
pseudoarthrosis after posterior spinal instrumented fusion. Illustration B shows an
intra-operative view of spinal pseudoarthrosis (white arrow).

Incorrect Answers:
Answer 1: MRI would appropriate for the investigation of epidural abscess, however
there is little clinical suspicion for this diagnosis. The amount of hardware present
would also cause significant metal artifact with MR imaging, making the study
difficult to interpret.
Answer 2: Although infection needs to remain in your differential diagnosis,
ESR/CRP and bone scan are not useful in the evaluation of pseudoarthrosis. Findings
of the bone scan are very non-specific and are often falsely-negative.
Answer 4: Although dynamic static radiographs are easy to obtain, they fail to
diagnose nonunion 60-80% of the time. Adjacent segment disease often occurs with
posterior fusion, however pain would be localized superior to the hardware in the
thoracic region.
Answer 5: Lateral bending radiographs are not indicated in the investigation of
nonunion.

(SBQ12SP.19) A 41-year-old male presents with acute onset of low back pain that
started when he was trying to lift a heavy box while helping his brother move
apartments two days ago. The pain has been severe enough to cause him to miss work
yesterday. He has no neurologic deficits. What are the chances he will return to work
within 6 weeks? Review Topic

1 80 to 90%
2 70 to 80%
3 60 to 70%
4 50 to 60%
5 40 to 50%

PREFERRED RESPONSE 3

Most adults (up to 80%) will experience an episode of low back pain in their lifetime.
In those whose pain is severe enough to cause them to miss work, 60 to 70% will
return by 6 weeks, and 80 to 90% will return by 12 weeks.

Low back pain is common in adults. Most patients experience resolution of symptoms
quickly and have no lasting loss of function. However, 5 to 10% develop chronic
pain. Recurrence of pain is common, and is part of the natural history, occurring in 20
to 72% of patients. After 12 weeks, return to work rates are slow.

Shen et al. review the nonoperative management of acute and chronic low back pain.
The authors note that none of the available interventions has been proven by high
quality large randomized controlled trials. The authors believe that low dose oral
steroids are safe in the short term and that injection therapy should not be used
without a reasonable presumptive diagnosis.

Madigan et al. review the management of lumbar degenerative disease. The authors
emphasize that the majority of patients improve within 6 weeks with or without
treatment. For patients that do not, the authors do not recommend epidural injection,
as there are no good studies to support their use in the treatment of discogenic back
pain. When surgical treatment is indicated they state that arthrodesis is the gold
standard, although the long term results of total disk arthroplasty are being elucidated.

Illustration A shows a table of available non-operative treatment modalities for back


pain. Illustration B shows a list of possible diagnoses associated with acute low back
pain, with those associated with neurogenic pain italicized. Illustration C shows a list
of 'red flag' symptoms indicating serious/emergent causes of low back pain.

Incorrect answers:
Answer 1: 80 to 90% will return by 12 weeks.
Answer 2: Longer than 6 weeks is required for 80% return
Answers 4, 5: These numbers are too low for return to work at 6 weeks.

(OBQ08.181) A 32-year-old male presents with left leg pain and weakness. An axial
image from his MRI is shown in Figure A. Which of the following physical exam
findings would be most consistent with this MRI finding. Review Topic

1 Numbness over dorsal aspect of the foot, weakness to gluteus medius


2 Numbness over plantar foot, weakness to his gastrocsoleus complex
3 Numbness over medial malleolus, and weakness to quadriceps
4 Numbness over medial calf, weakness in his EHL
5 Numbness over lateral malleolus, weakness to hip adduction

PREFERRED RESPONSE 1

The MRI demonstrates a left paracentral L4/5 disc protrusion which leads to
compression of the traversing (descending) left L5 nerve root. Numbness over the
dorsal aspect of the foot and weakness to gluteus medius is consistent with a L5
radiculopathy.

While nerve root innervation shows some variability by patient, L5 is


"characteristically" responsible for the sensation to the dorsal aspect of the foot, ankle
dorsiflexion (tibialis anterior - along with L4), great toe extension (EHL), and hip
abduction (gluteus medius).

Suri et al. reported on specific physical exam findings that significantly increased the
likelihood of nerve root impingement at specific lumbar levels. They found: L2 was
associated with decreased anterior thigh sensation. L3 was associated with a positive
femoral stretch test. L4 was associated with a blunted patellar reflex, decreased
medial ankle sensation or a positive crossed femoral stretch test. L5 was associated
with was associated with decreased hip abductor strength.

Luri et al. reported 8-year follow up on the patients in the spine patient outcomes
research trial who underwent surgical vs. conservative care for treatment of lumbar
herniated disc. They found that patients who underwent surgical treatment had
superior results that were maintained at 8 years compared to patients who underwent
conservative management.

Figure A is an axial MRI at the L4/5 disc space that shows a left paracentral disc
herniation compressing the descending L5 nerve root. Illustration A identifies the
structures in the MRI image. Illustration B demonstrates the dermatome, reflex and
motor function associated with the L4, L5 and S1 nerve root.

Incorrect Answers:
Answer 2: Numbness over plantar foot, weakness to his gastrocsoleus complex would
be consistent with an S1 radiculopathy.
Answer 3: Numbness over medial malleolus and weakness to quadriceps would be
consistent with an L4 radiculopathy.
Answer 4 and 5: Neither would be consistent with "characteristic" nerve root
innervation.

(SBQ13PE.102) An 26-year-old male presents to your office complaining of bilateral


hip and low back pain. On physical examination, he has 10 degree bilateral hip
flexion contractures. An AP pelvis radiograph is demonstrated in figure A. Which of
the following findings is consistent with this patient's presentation? Review Topic
1 A positive flexion, adduction, internal rotation (FADDIR) test
2 A history of untreated slipped capital femoral epiphysis (SCFE)
3 A thrombophilia
4 Normal serum ESR and CRP
5 Positive Human Leukocyte Antigen B27 (HLA-B27)

PREFERRED RESPONSE 5

The patient has large joint arthralgia and sacroiliac joint sclerosis on AP pelvis
radiograph, which is consistent with ankylosing spondylitis. Patients with ankylosing
spondylitis have positive Human Leukocyte Antigen B27 (HLA-B27).

Ankylosing spondylitis (AS) is a seronegative spondyloarthropathy that affects the


axial skeleton as well as large joints including the hips and knees. The most common
initial site of pain is the sacroiliac (SI) joint, and is demonstrated as sacroiliitis on
pelvic radiograph. Hip involvement is common, and typically manifests as hip pain
and flexion contracture. Serologic studies will be typically be negative for rheumatoid
factor, but positive for HLA-B27 in 90% of patients.

Kubiak et. al. review orthopaedic management of AS. Common orthopaedic


manifestations include SI joint pain, hip flexion contractures, and stiffness of the
cervical and lumbar spine. They report that laboratory analysis of patients with active
disease will typically demonstrate mild elevation of ESR, CRP, and WBC. Patients
with chronic AS may demonstrate a normocytic anemia. If HLA-B27 is negative, a
high clinical suspicion should still be maintained.

Gensler et al. review the different clinical conditions that compose of juvenile-onset
spondyloarthritis. They report on the different spondyloarthritides includes ankylosing
spondylitis, reactive arthritis, arthropathy associated with inflammatory bowel
disease, and that associated with psoriasis. They emphasize that the appearance of
sacroiliac joint and spinal disease in the form of ankylosing spondylitis usually takes
5–10 years after initial symptom presentation, and therefore, a definite diagnosis can
take several years leading to a delay in diagnosis.

Figure A is an AP pelvis radiograph of a skeletally mature individual demonstrating


sclerosis of the SI joint indicative of sacroiliitis. Illustration A shows a axial CT
image of the patient in the stem. Sclerosis and bone erosion can be seen in the
sacroiliac joint.

Incorrect Answers:
Answer 1: FADDIR Test refers to "flexion, adduction, and internal rotation" test for
femoroacetabular impingement which is unlikely to reproduce his symptoms. This
patient would likely have symptoms reproduced with flexion, abduction, and external
rotation (FABER or Patrick's Test).
Answer 2: Patients with a history of untreated SCFE may have persistent hip pain, but
would have radiographs most likely demonstrating pistol grip deformity and perhaps
osteonecrosis of the femoral epiphysis.
Answer 3: A thrombophilia may have osseous manifestations such as bone
infarctions, which are not evident on this image.
Answer 4: Patients with active AS will typically have mildly elevated inflammatory
markers.

(SAE12SN.37) Figure 37 shows the standing lateral radiograph of a 62-year-old


woman who reports lower back pain and the inability to stand upright. What
permanent anatomic pelvic parameter should be measured and considered when
determining the amount of lumbar lordosis correction that will be necessary to obtain
sagittal balance? Review Topic

1 Pelvic tilt
2 Pelvic incidence
3 Sacral slope
4 Acetabular version
5 Femoral version

PREFERRED RESPONSE 2

Pelvic incidence (PI) is the anatomic angle between the sacral end plate and a line
connecting the center of the femoral heads. Increased pelvic incidence has been found
to correlate with the incidence and severity of spondylolisthesis. Patients with
increased PI require increased lumbar lordosis to restore sagittal balance. Pelvic tilt
(PT) and sacral slope (SS) have also been found to correlate with lumbar lordosis;
however, both PT and SS can change depending on pelvic rotation. PI is the only
permanent pelvic parameter that is unaffected by pelvic rotation. Acetabular and
femoral version have not been found to be associated with lumbar lordosis.

(SAE11OS.127) A 38-year-old man sustained a complete thoracic spinal cord injury


at age 14. An MRI scan of his shoulder, when compared with studies from uninjured
controls, is more likely to show which of the following? Review Topic

1 Hypertrophied subscapular muscle


2 Rotator cuff tear
3 Posterior glenohumeral subluxation
4 Increased bone density
5 Supraspinatus nerve compression

PREFERRED RESPONSE 2

Children that sustain a spinal cord injury or otherwise use a wheelchair for mobility,
and thus often have more pain and a higher incidence of structural and functional
changes of the shoulder joint as an adult. MRI studies have shown a four-fold risk of
rotator cuff tears in people with long-term paraplegia when compared with age-
matched controls. An MRI scan would not show bone density changes. The other
answer choices have not been demonstrated in higher numbers on MRI in paraplegics.

(SAE11AN.46) A 17-year-old boy is shot in the left side of the neck at the C5-6 level
and sustains an incomplete spinal cord injury that is called a Brown-Sequard
syndrome. Which of the following best describes the expected deficits? Review Topic

1 Profound bilateral wrist extensor, finger flexor, and intrinsic weakness with good
preservation of lower extremity motor function
2 Severe bilateral upper and lower extremity weakness, pain and temperature sensory
deficit but preservation of deep pressure and proprioception
3 Weakness of the right upper and lower extremity with diminished pain and
temperature sensation on the left side of the body
4 Left wrist extensor weakness and numbness along the radial border of the left
forearm extending into the thumb and index finger
5 Weakness of the left upper and lower extremity with diminished pain and
temperature sensation on the right side of the body
PREFERRED RESPONSE 5

Brown-Sequard syndrome is an incomplete spinal cord injury that involves damage


unilaterally to the cord, most commonly from penetrating trauma. The motors fibers
of the cord decussate within the brainstem so the motor deficit is ipsilateral to the
injury; whereas, the pain and temperature fibers cross midline immediately on
entering the cord so that the sensory deficit is contralateral to the injury. This patient
was shot in the left side, thus he would have weakness of the left upper and lower
extremity with diminished pain and temperature sensation on the right side of the
body. Response 3 describes opposite symptoms that would result from a right-sided
injury. Response 1 describes a central syndrome with greater upper than lower
extremity involvement. Response 2 is an anterior cord syndrome with only
preservation of the posterior columns of the cord. Response 4 describes a C6 root
injury.

(OBQ13.271) A 45-year-old female returns to your clinic with 10-weeks of severe


pain that starts in her back and extends down her right leg to the top of her foot. On
physical exam she has decreased sensation on the dorsal aspect of her foot and 4/5
strength in her EHL. She has a positive straight leg raise on the right. The remaining
physical exam is unremarkable, including normal achilles and patellar reflexes
bilaterally, no clonus, and a down-going Babinski sign. Her pain has not been relieved
by NSAIDs, epidural steroids or physical therapy. Figure A is a sagittal MRI and
figure B is a axial MRI through the L4/5 disc space. What is the best treatment option
at this time? Review Topic

1 Continued oral anti-inflammatories


2 Right L4/5 microdiscectomy
3 Right L4/5 minimally invasive transforaminal interbody fusion
4 Referral for EMG and nerve conduction studies
5 L4/5 posterior decompression and instrumented fusion
PREFERRED RESPONSE 2

The patient has a herniated L4/5 disc leading to right L5 radicular pain. She continues
to have severe symptoms despite 10 weeks of nonoperative treatment, so the next step
is a right sided L4/5 microdiscectomy.

Patients with paracentral herniated lumbar discs present with radicular pain affecting
the traversing (caudal) nerve root. Unless the patient develops progressive neurologic
decline, patients with herniated lumbar discs should undergo no less than 6 weeks of
conservative treatment consisting of anti-inflammatory medications, rest and therapy.
Most patients improve with nonoperative modalities. If appropriate conservative care
fails, the correct surgical option is a unilateral microdiscectomy.

In the Spine Patient Outcomes Research Trial (SPORT) Weinstein et al. reported on
the results of 501 patients with herniated lumbar discs who had failed at least six
weeks of non-operative care. The patients were randomized to operative or non-
operative care, however there was a high amount of crossover between the two
groups. Because of this, there was no difference reported between the two groups at
final follow-up using an intent-to-treat analysis.

Weinstein et al., because of the flaws with the intent-to-treat analysis, also published
an observational study on 528 patients who received surgery and 191 who received
nonoperative care for a herniated lumbar disc. They reported that while both groups
had an improvement from baseline, at two years, patients who elected to undergo
surgery had significantly better outcomes than those who chose conservative care.

Lurie et al. reported the eight-year results from the observational group of the SPORT
data, and found that the patients who underwent surgery continued to have
statistically superior outcomes compared to those who underwent conservative care at
long-term follow-up.

Figure A is a T2 sagittal MRI of the lumbar spine demonstrating a right sided L4/5
disc herniation, and Figure B is an axial image again demonstrating a paracentral L4/5
disc herniation.

Incorrect answers:
Answer 1 is incorrect because the patient has had symptoms for 10 weeks and has
failed conservative management.
Answers 3 and 5 are incorrect because a fusion is not indicated for a herniated lumbar
disc.
Answer 4 is incorrect because no further diagnostic studies are indicated. Both the
patient's symptoms and imaging are concordant with a L4/5 disc herniation being the
cause of her symptoms.
(SAE12SN.82) A 22-year-old man has an acute spinal cord injury after a diving
accident. Preliminary radiographs reveal bilateral jumped facets at C6-7. Neurologic
examination shows an incomplete spinal cord injury consistent with an ASIA B
impairment grade. The patient is otherwise hemodynamically stable with no other
injuries. Attempts at closed high weight reduction with tong traction have so far been
unsuccessful. What is the most appropriate management at this time? Review Topic

1 Continue a high weight closed reduction of the fracture-dislocation


2 Urgent surgical intervention for reduction and decompression
3 High-dose steroids for 48 hours before surgical stabilization
4 Halo fixation
5 Closed reduction under general anesthesia

PREFERRED RESPONSE 2

Although there are no current standards for the timing of surgical intervention for
acute spinal cord injuries there is increasing data, including animal studies, suggesting
that early decompression and stabilization of an acute spinal cord injury can be
beneficial. Continuing attempts at closed reduction is not advised given the failure of
attempted high weight reduction. In light of the neurologic deficit, waiting 48 hours
with or without steroid treatment is not recommended. Likewise, halo fixation without
reduction of the dislocation should not be considered for definitive treatment. Closed
reductions should not be performed under general anesthesia.

(SAE12SN.85) A 69-year-old man has nonpainful weakness in the upper and lower
extremities. He also notes progressive instability in his gait and increasing difficulty
ambulating, as well as manipulating small objects with his hands. MRI scans of his
cervical spine are shown in Figures 85a and 85b. When would be the most appropriate
time to proceed with surgical treatment? Review Topic

1 When the patient is medically stable for surgery


2 When the MRI scans show multisegmental high-intensity intramedullary signal
changes on T2-weighted sequences
3 When he reaches a Nurick grade of IV for his preoperative neurologic function
4 When he reports neck and/or extremity pain that becomes intolerable or not
controlled by medication
5 When he develops bowel or bladder incontinence

PREFERRED RESPONSE 1

The natural history of cervical myelopathy is one of slow deterioration over time,
typically in a stepwise fashion with a variable period of stable neurologic function.
More recent studies suggest that surgery should be performed as soon as possible
when cervical spondylotic myelopathy has been diagnosed. Both anterior and
posterior are effective and there is no statistical difference between their outcomes.
Surgical outcome is related to the patient's age, disease course, the presence of
osseous spinal stenosis, preoperative comorbidities, the preoperative spinal cord
functional score, and the presence of high-signal abnormalities on T2-weighted
images. To improve the operative result, all the influencing factors should be
considered. Patients with focal high-intensity intramedullary signal changes on T2-
weighted images have better clinical outcomes following surgery than do patients
with demonstrable multisegmental high-intensity intramedullary signal changes on
T2-weighted sequences. The transverse area and shape of the spinal cord at the
involved segment may also be predictive of surgical outcome. With progressive
compression, the cross section of the spinal cord changes from a boomerang shape to
a teardrop shape to a triangular shape. In patients with a Nurick grade of I, there are
signs of cord involvement, but gait remains normal. With a Nurick grade of II, there
are mild gait abnormalities, not affecting the patient's employment status. With a
Nurick grade of III, gait abnormalities prevent employment, but the patient remains
able to ambulate without assistance. In Nurick grade IV, the patient is only able to
ambulate with assistance. In Nurick grade V, the patient is chair-bound or bedridden.
Clearly, it is desirable to operate when the patient is functioning with a Nurick grade
of I or II. Whereas many patients presenting with cervical spondylotic myelopathy
also report axial neck pain and radicular symptoms in the upper extremities, this is not
always the case. Surgical intervention will generally be effective in eliminating this
pain; however, the pain is not the determining factor for performing surgery. Surgery
is performed to preserve and restore function. Changes in bowel and bladder function
can occur in extremely severe cases of myelopathy, but this is quite rare.

(SAE09SN.96) A 35-year-old woman reports an 8-week history of neck pain radiating


to her right upper extremity. She denies any history of trauma or provocative event.
Examination reveals decreased pinprick sensation in her right middle finger,
otherwise sensation is intact bilaterally. Finger flexors and interossei demonstrate 5/5
motor strength bilaterally. Finger extensors are 4/5 on the right and 5/5 on the left.
The triceps reflex is 1+ on the right and 2+ on the left. The most likely diagnosis is a
herniated nucleus pulposus at what level? Review Topic

1 C3-4
2 C4-5
3 C5-6
4 C6-7
5 C7-T1

PREFERRED RESPONSE 4

The patient’s neurologic examination is consistent with a C7 radiculopathy on the


right side. In a patient with this symptom complex in the absence of trauma, a cervical
disk herniation is the most common etiology for a C7 radiculopathy. There are eight
cervical nerve roots and the C7 nerve exits at the C6-7 disk space and is most
frequently impinged by a disk herniation at this level.

(SAE07PE.42) A 14-year-old boy reports a 4-month history of increasing backache


with difficulty walking long distances. His parents state that he walks with his knees
slightly flexed and is unable to bend forward and get his hands to his knees. He denies
numbness, tingling, and weakness in his legs and denies loss of bladder and bowel
control. A lateral radiograph of the lumbosacral spine is shown in Figure 18. What is
the best surgical management for this condition? Review Topic

1 Vertebrectomy of L5
2 Posterior spinal fusion with or without instrumentation from L4 to S1
3 Posterior spinal fusion without instrumentation from L5 to S1
4 Anterior spinal fusion from L4 to L5
5 Direct repair of the spondylolysis defect
PREFERRED RESPONSE 2

The patient has a grade 4 spondylolisthesis. Optimal surgical management is posterior


spinal fusion from L4 to the sacrum. The use of instrumentation is controversial.
Vertebrectomy is typically reserved for spondylo-optosis (grade 5) cases. Spinal
fusion from L5 to S1 usually is not successful for a slip that is greater than 50%.
Isolated anterior spinal fusion has not been successful, and direct repair of the pars
defect is only useful for spondylolysis without spondylolisthesis.

(OBQ13.124) A 65 year-old female presents to your clinic with a chief complaint of


difficulty walking. She states that she has had low back pain and balance difficulties
for the last 2 years, but over the last few months new bilateral posterior thigh and
buttock pain has prevented her from walking more than 100 feet. She states the only
place she can walk comfortably is in the grocery store.

On physical exam she is unable to preform a tandem gait, and she has 5/5 strength
with hip flexion, knee flexion/extension, ankle dorsiflexion/plantar flexion and great
toe extension. Her sensation is intact in L2-S2, and she has equal and symmetric 3+
achilles and patellar reflexes. She has 8 beats of clonus, and a down-going Babinski
reflex bilaterally.

Radiographs of her lumbar spine are seen in figures A and B. What is the next step.
Review Topic

1 MRI of her lumbar spine


2 Six weeks of physical therapy and anti-inflammatory medication
3 Determine the patients ankle brachial index
4 Cervical Spine MRI
5 Lumbar Epidural Injection
PREFERRED RESPONSE 4

The patient has signs and symptoms of both lumbar spinal stenosis and myelopathy,
also known as tandem stenosis. The next most appropriate step is a cervical spine
MRI.

While 79% of patients with cervical myelopathy will have at least one sign on
physical exam, the absence of a one or more signs such as a Babinski reflex or a
Hoffman's sign does not rule out the diagnosis of myelopathy. Tandem cervical and
lumbar stenosis occurs in between 5 and 25% of patients with lumbar stenosis, and
because of the stepwise progressive nature of myelopathy, it is critical not to miss this
diagnosis.

Rhee et al. found that the sensitivity and specificity of specific physical exam findings
varies in patients with myelopathy. Overall 79% of patients will have at least one
physical exam sign of myelopathy, with biceps hyperreflexia (62%) and the Hoffman
sign (59%) being the most sensitive. Classic upper motor neuron findings in the lower
extremity such as an upward Babinski reflex (13%) and clonus (13%) are not
sensitive.

Lee et al. performed a cadaveric study of 440 specimens to identify the overall
prevalence of stenosis in the population. They found 5.4% of the specimens had
cervical stenosis and 5.9% had lumbar stenosis. A total of 0.9% had both cervical and
lumbar stenosis.

Bajwa et al. evaluated over 1,000 skeletal remains to determine if tandem stenosis
(concomitant lumbar and cervical stenosis) is due to an increased risk of disc
degeneration or a congenitally small vertebral canal, and they concluded that tandem
stenosis is likely due to a congenitally small vertebral canal in both the cervical and
lumbar spine.

Figure A and B demonstrate an AP and lateral radiograph of the lumbar spine


respectively. A degenerative L4/5 spondylolisthesis is present.

Incorrect answers:
Answers 1, 2, 3 and 5 are all incorrect because they fail to identify that the patient has
myelopathy.

(SAE11OS.51) A 17-year-old girl with a history of Scheuermann's kyphosis has a


fixed thoracic deformity of 80 degrees. There was no correction of her deformity on
supine hyperextension radiographs. What is the most appropriate treatment? Review
Topic

1 Posterior arthrodesis
2 Anterior interbody arthrodesis
3 Smith-Petersen osteotomies with posterior arthrodesis
4 Vertebral column resection with posterior arthrodesis
5 Pedicle subtraction osteotomy with posterior arthrodesis

PREFERRED RESPONSE 3

The Smith-Petersen osteotomy is most appropriate for long, sweeping, global


kyphosis, such as Scheuermann's kyphosis. It can achieve approximately 10 degrees
of correction in the sagittal plane at each spinal level at which it is performed. The
pedicle subtraction osteotomy is the preferred osteotomy for patients with ankylosing
spondylitis, who have a sagittal plane imbalance. It can achieve approximately 30
degrees to 40 degrees of correction in the sagittal plane at each spinal level at which it
is performed. Vertebral column resections are extensive procedures, thus they are
most appropriately applied to pathologies with sharp angular kyphosis, anterior
fusions, and when maximal visualization and decompression of the spinal cord is
required. Sagittal curves were reduced an average of 50 degrees, with a lumbosacral
deformity treated via vertebral column resection. Anterior arthrodesis alone will not
provide sufficient correction and stabilization of the deformity. Posterior arthrodesis
alone, while providing stabilization, will not correct the fixed deformity.

(SAE07PE.100) A 7-year-old boy has had low back pain for the past 3 weeks.
Radiographs reveal apparent disk space narrowing at L4-5. The patient is afebrile.
Laboratory studies show a WBC count of 9,000/mmP3P and a C-reactive protein
level of 10 mg/L. A lumbar MRI scan confirms the loss of disk height at L4-5 and
reveals a small perivertebral abscess at that level. To achieve the most rapid
improvement and to lessen the chances of recurrence, management should consist of
Review Topic

1 oral antibiotics.
2 IV antibiotics.
3 surgical drainage of the perivertebral abscess and IV antibiotics.
4 bed rest.
5 cast immobilization.

PREFERRED RESPONSE 2

The patient has diskitis. Administration of IV antibiotics speeds resolution and


minimizes recurrence. Bed rest and cast immobilization have been successfully used
to treat this disorder but can be associated with prolonged recovery and frequent
recurrence, even when oral antibiotics are administered. A perivertebral abscess seen
in association with this condition usually resolves without surgery.
(SBQ12SP.92) A 36-year-old man presents to the emergency department after being
involved in a motor vehicle collision. He is complaining of back pain and imaging
shows the findings in Figure A. On neurological examination, he does not have any
deficits. MRI shows approximately 25% canal encroachment and no evidence of
injury to the posterior ligamentous complex. Which of the following is the most
appropriate course in management? Review Topic

1 Strict bedrest for six weeks then progressive weightbearing


2 Ambulation as tolerated with or without a TLSO
3 Surgical decompression and anterior stabilization
4 Surgical decompression and posterior stabilization
5 Surgical decompression and combined anterior/posterior stabilization

PREFERRED RESPONSE 2

The patient has a L1 burst fracture with minimal retropulsion of bony fragments in the
spinal canal. In the absence of neurological deficits and injury to the PLC, the most
appropriate treatment is ambulation as tolerated with or without a
thoracolumbrosacral orthosis (TLSO).

Thoracolumbar burst fractures are typically caused by an axial load with flexion and
commonly found in this location due to increased motion at these segments. With an
intact posterior ligamentous complex (PLC) and no neural compromise, TLSO is the
mainstay of treatment. If there is evidence of neurological deficit and/or PLC injury,
decompression and fusion are indicated. The degree of acceptable kyphosis is
controversial. The choice of anterior versus posterior approach is based on ease of
decompression.

Vaccaro et al. introduced a new classification system for thoracolumbar injuries,


TLICS, based on morphological appearance, integrity of the posterior ligamentous
complex, and neurological status. They advocate use of the system for nonoperative
versus operative decision making and communication between surgeons.

Bailey et al. completed a randomized, nonblinded controlled trial to determine the


efficacy of bracing for AO type A0-A3 thoracolumbar burst fractures. Both groups
were encouraged to ambulate as tolerated and the no brace group had bending
restrictions for 8 weeks. They found no difference in the Roland Morris Disability
Questionnaire (RMDQ) score at 3 months after injury.

Figure A is sagittal CT scan of the lumbar spine showing a burst fracture of L1 with
minimal retropulsion. Illustration A is the TLICS classification with score of 4 being
the branch point for nonoperative versus operative management.

Incorrect Answers:
Answer 1: Bedrest is not required for management of burst fractures.
Answer 3-5: In the absence of neurological compromise or PLC injury, operative
management is not indicat

(SAE07PE.25) What risk factor is most associated with progression of idiopathic


scoliosis to a curve requiring surgery? Review Topic

1 Curve magnitude of more than 20 degrees at menarche


2 Curve magnitude of more than 30 degrees at the peak height velocity
3 Curve magnitude of more than 30 degrees at skeletal age 12 years
4 Curve magnitude of more than 30 degrees at Risser grade 2
5 Curve flexibility of less than 50% at Risser grade 2

PREFERRED RESPONSE 2

The magnitude of the curve at the time of the peak height velocity is the most
prognostic sign in relationship to surgery. More than 70% of curves that measure
more than 30 degrees at this time are likely to reach surgical range.

(OBQ15.68) A 65-year-old man undergoes L4-L5 laminectomy and instrumented


fusion for lumbar spinal stenosis. Post procedure radiograph is shown in figure A.
Postoperatively, he develops numbness and tingling along the lateral aspect of his
thigh with no associated foot symptoms or weakness. Which of the following is the
most likely cause of this finding? Review Topic
1 L4 nerve root injury during laminectomy
2 L5 nerve root injury during laminectomy
3 Patient positioning during surgery
4 Dural tear
5 Use of monoaxial pedicle screws

PREFERRED RESPONSE 3

During lumbar laminectomy and instrumented fusion, patients are in prone position
with padded rests near the iliac crest. The lateral femoral cutaenous nerve (LFCN) is
at risk of compression during surgery due to positioning.

Careful patient position during orthopaedic surgery is of the utmost importance to


prevent nerve injury. When patients are placed lateral for hip procedures, adequate
positioning and padding of bony prominences and nerves is necessary to prevent
complications. During prone position for spine surgery, the lateral femoral cutaneous
nerve can be compressed under the iliac crest padded rest leading to meralgia
parasthetica (MP). Numbness and tingling over the lateral thigh are usually
temporary. The ulnar nerve at the elbow can also be at risk due to pressure from the
positioning of the arms and the brachial plexus can be compromised by possible
stretch.

Cho et al. review the literature regarding development of paresthesia due to


compression of the LFCN after prone positioning for spinal surgery. They reported up
to 24% incidence of after surgery. Surgery longer than 3.5 hours and shorter distance
between the pelvic posts were implicated in development of MP. They recommened
the patient should be positioned symmetrically, with smaller bolsters under the ASIS
and adequate distance between the two pelvic posts.

Labrom et al. examined the use of SSEPs during scoliosis surgery to evaluate
positional-related brachial plexopathy. They found that 27 of 434 patients had 30
percent or greater loss of SSEP during surgery. There was a significant association
with prone versus supine positioning (p<0.01).

Schwartz et al. used neurological monitoring to identify the incidence and location of
position-induced nerve injury in anterior cervical spine surgery. 1.8% of 3806 patients
showed evidence of impending neurological injury. The majority of these cases were
at the brachial plexus during shoulder taping and application of counter traction.

Figure A is an AP lumbar spine radiograph showing intact hardware from L4-L5


laminectomy and fusion.

Incorrect Answers:
Answers 1,2: Injury to the L4 or L5 nerve root would be expected to present with
symptoms past the knee and/or weakness
Answer 4: Localized numbness on the lateral aspect of the thigh would not be
expected with a dural tear
Answer 5: Use of monoaxial pedicle screws is not associated with MP.

(OBQ15.152) A teenage female is referred to you for evaluation of curvature in her


back that was discovered on routine school screening. She is diagnosed with
adolescent idiopathic scoliosis. In which of the following scenarios is a bracing
program the most appropriate treatment? Review Topic

1 Left upper thoracic curve 16 degrees and right main thoracic curve 18 degress,
Risser stage in figure B
2 Right thoracolumbar curve 35 degrees, Risser stage in figure C
3 Left upper thoracic curve 15 degrees and right main thoracic curve 30 degrees,
Risser stage in figure B
4 Left upper thoracic curve 26 degrees and right main thoracic curve 51 degrees,
Risser stage in figure A
5 Left thoracolumbar curve 19 degrees, Risser stage in figure A

PREFERRED RESPONSE 3

Bracing is most appropriate for skeletally immature patients with a curve > 20/25
degrees. Thus the patient with a left upper thoracic curve of 15 degrees and right main
thoracic curve of 30 degrees in Risser stage 1 of growth meets criteria.

The treatment of adolescent idiopathic scoliosis (AIS) depends on the magnitude and
location of curve as well skeletal maturity of the patient. For curves less than 20
degrees, observation is appropriate until skeletal maturity, with closer intervals during
times of peak growth. Curves between 20-25 and 45 degrees in patients who are
Risser stage 0,1 or 2 are best treated with bracing to stop progression. Curves with an
apex at T7 or below are typically treated with a Boston brace. Curves over 50 degrees
generally warrant a discussion about surgery to prevent progression past maturity.

In a landmark study, Weinstein et al. evaluated both a randomized and preference


based cohort of bracing versus observation. The trial was stopped early due to
efficacy of bracing. The rate of treatment success was 72% after bracing and 48%
after observation. Treatment success was strongly correlated to time of brace wear.

Schlenzka et al. reviewed indications, treatment, and complications associated with


brace treatment of AIS. They state that further evidence is necessary to evaluate the
efficacy of bracing in AIS.

Figure 1, 2, and 3 are radiographs depicting Risser stage 0, 1, and 4 respectively.


Illustration B shows radiographs of all Risser stages. Illustration C is the Lenke
classification system for idiopathic scoliosis.

Incorrect Answers:
Answers 1,5: Curves less than 20 degress can be observed
Answer 2: A 35 degree curve is typically braced in patients who are Risser stage 0,1,
and 2. This patient is near skeletal maturity and will not benefit from bracing
Answer 4: Bracing is less effective in curves > 45 degrees.

(SAE09SN.100) A 45-year-old woman has idiopathic scoliosis. Surgery is to include


an anterior thoracic release through an open left thoracotomy. The thoracotomy will
have what effect on the patient’s pulmonary function postoperatively? Review Topic

1 Unaffected
2 Transiently reduced postoperatively but ultimately improves to greater than
preoperative function
3 Transiently reduced immediately postoperatively but then quickly returns to
preoperative levels
4 Improves postoperatively due to correction of the scoliosis and is maintained long
term
5 Reduced postoperatively and often remains reduced long term

PREFERRED RESPONSE 5

A thoracotomy in an adult with idiopathic scoliosis causes a reduction in pulmonary


function that often does not return to preoperative levels. What pulmonary function
that does recover, recovers over many months. Long-term improvement in pulmonary
function, compared to preoperative function, is rarely seen. This should be considered
in planning surgical intervention in adults with scoliosis.

(SAE09SN.92) A 42-year-old woman who has had an 18-month history of severe low
back pain is referred to your office for surgical evaluation. She reports that the pain
initially began with right lower extremity pain and management consisted of oral
analgesics, nonsteroidal anti-inflammatory drugs, and muscle relaxants. She has seen
a chiropractor as well as a pain management specialist and she is status-post epidural
steroid injections. She has also completed exhaustive physical therapy, as she is a
certified athletic trainer and runs a health fitness program at a community hospital.
Currently, she denies lower extremity pain and her pain is isolated to her low back
and is subjectively graded as 8/10, with 10 being the worst pain she has ever
experienced. The pain is interfering with her activities of daily living and she is
seeking definitive treatment. Figures 32a through 32c show current MRI scans. Based
on the current available medical literature, what is the most appropriate treatment?
Review Topic

1 Continued nonsurgical management to include long-acting narcotic analgesics


2 Referral for vertebral axial decompression
3 Referral to interventional pain management for a spinal cord stimulator
4 Intradiskal electrothermal therapy (IDET) at L5-S1
5 Lumbar spinal fusion at L5-S1

PREFERRED RESPONSE 5

The MRI scans reveal advanced degenerative disk disease at L5-S1. Nonsurgical
management has failed to provide relief and the patient is quite debilitated as a result
of her back pain. Fritzell and associates demonstrated that in a well-informed and
selected group of patients with severe low back pain, lumbar fusion can diminish pain
and decrease disability more efficiently than commonly used nonsurgical treatments.
In a recent updated Cochrane Review of surgery for degenerative lumbar spondylosis,
it was noted that while Fritzell and associates appeared to provide strong evidence in
favor of fusion, a more recent trial by Brox and associates demonstrated no difference
between those patients undergoing lumbar fusion compared to those receiving
cognitive intervention and exercise. The Cochrane Review suggests that this may
reflect a difference between the control groups. Fritzell and associates compared
lumbar fusion to standard 1990s “usual care,” whereas Brox and associates compared
lumbar fusion to a “modern rehabilitation program.” Bear in mind that this patient is a
certified athletic trainer and runs a hospital health fitness department; therefore, at
least for purposes of this question, it can be assumed that she has participated in a
“modern rehabilitation program.” The Cochrane Review goes on to state that
preliminary results of three small trials of intradiskal electrotherapy suggest that it is
ineffective and that preliminary data from three trials of disk arthroplasty do not
permit firm conclusions.

(SAE12SN.20) In the treatment of fungal osteomyelitis of the spine, what is the


preferred agent? Review Topic

1 Cefazolin
2 Vancomycin
3 Combination treatment with isoniazid, ethambutol, rifampin, and pyrazimide
4 Trimethoprim-sulfamethoxazole
5 Amphotericin B

PREFERRED RESPONSE 5

Amphotericin B is a broad-spectrum antifungal that is commonly used as the first-line


agent for treatment of fungal infections of the spine. Cefazolin, vancomycin, and
trimethoprim-sulfamethoxazole are better suited for the treatment of bacterial
infections. Combination treatment with isoniazid, ethambutol, rifampin, and
pyrazimide is used for the treatment of tuberculous infections.

(SAE09SN.79) Which of the following represents a contraindication for interspinous


process decompression for the treatment of lumbar spinal stenosis? Review Topic

1 Grade I degenerative spondylolisthesis


2 Inability to walk at least 100 feet
3 Cauda equina syndrome
4 Fixed sensory deficit
5 Intermittent foot drop

PREFERRED RESPONSE 3

Kondrashov and associates noted stable good outcomes at 4 years in 14 of 18 patients


treated with X-STOP interspinous process decompression as defined as an
improvement over preoperative Oswestry scores of 15 points or more. Similar results
were seen after 1 year in a European study by Siddiqui and associates. Exclusion and
inclusion criteria for these studies varied somewhat, but cauda equina syndrome was
the only exclusion criteria listed in both studies. All of the other choices did not
represent exclusion criteria in either study.

(SAE09SN.16) A 55-year-old woman with a long history of low back and left lower
extremity pain has failed to respond to exhaustive nonsurgical management. MRI
scans show bulging and degeneration at L3-4 and L4-5 as well as a normal disk at L2-
3 and L5-S1. She undergoes provocative lumbar diskography at L3-4, L4-5, and L5-
S1. Post-diskography axial CT images of L3-4 and L4-5 are shown in Figures 6a and
6b, respectively. The injections at L3-4 and L4-5 produce no pain. The injection at
L5-S1 produces 10/10 concordant back pain with radiation to the lower extremity.
What is the most appropriate recommendation at this time? Review Topic

1 Consider fusion surgery


2 Intradiskal ozone therapy
3 Lumbar laminectomy
4 Vertebral augmentation
5 Cognitive intervention and exercise

PREFERRED RESPONSE 5

The results of this patient’s lumbar diskography are equivocal at best. The two disks
most likely to be her pain generators, based on their MRI appearance, produced 10/10
pain, however it was nonconcordant and did not reproduce any of her typical left-
sided radicular symptoms. The only disk that produced concordant back pain was the
normal disk at the L5-S1 level and it reproduced radicular symptoms on the side
opposite of her typical pain. Based on these findings, it would be difficult to select a
level or levels to include in a lumbar fusion. As such, continued nonsurgical
management is the safest treatment option at the current time. Brox and associates
reported on a randomized clinical trial comparing lumbar fusion to cognitive
intervention and exercise and found similar results in both groups, with significantly
less risk in the latter.

(SAE12SN.74) A 42-year-old man reports a 3-day history of worsening lower back


pain. He denies any history of recent trauma or infections. He also reports difficulty
urinating and fecal incontinence in the last 24 hours. Examination reveals generalized
lower extremity weakness, diminished sensation in a saddle distribution, and loss of
rectal tone. What is the most appropriate management at this time? Review Topic

1 Immediate MRI of the lumbar spine and possible acute surgical intervention
2 General reassurance, anti-inflammatory medications, and an early home exercise
program
3 Radiographs of the lumbar spine and pain medications with 2 days of bed rest
4 Caudal epidural steroid injection with follow-up in 1 week
5 Outpatient MRI of the lumbar spine with follow-up in 1 week

PREFERRED RESPONSE 1

Cauda equina syndrome is a medical emergency that must be quickly diagnosed and
treated to avoid long-term complications. Cauda equina syndrome typically presents
with low back pain, unilateral or bilateral sciatica, saddle sensory disturbances,
bladder and bowel dysfunction, and variable lower extremity motor and sensory loss.
Although a number of pathologies can cause cauda equine syndrome, disk herniation
is the most common cause of acute onset cauda equina syndrome. Cauda equina
syndrome should be evaluated on an emergent basis and admission for work-up is
appropriate. Emergent MRI to evaluate the level of spinal compression and acute
decompression surgery is the most appropriate treatment in this situation. Nonsurgical
management consisting of medications, bed rest, and a home exercise program are not
appropriate. Whereas radiographs could be useful in a patient with traumatic onset of
symptoms, MRI is the best study for evaluation of the spinal canal. Office follow-up
and outpatient diagnositc testing are also inappropriate in this scenario.
(SAE12SN.10) A 42-year-old woman reports neck stiffness, upper extremity pain,
clumsiness, weakness, and instability of gait. Examination reveals 4+ of 5 strength in
the upper extremities and 3+ biceps, brachioradialis, and patellar reflexes with a
positive Hoffman sign bilaterally. MRI and CT scans are shown in Figures 10a and
10b. Based on the history and imaging findings, what is the most likely diagnosis?
Review Topic

1 Diffuse idiopathic skeletal hyperostosis


2 Ankylosing spondylitis
3 Ossification of the posterior longitudinal ligament
4 Rheumatoid arthritis
5 Degenerative cervical stenosis

PREFERRED RESPONSE 3

The sagittal T2-weighted MRI scan shows moderate-severe multilevel cervical


stenosis. The cord compression is noted to be not only at the disk levels but also at the
midvertebral body levels, and the posterior longitudinal ligament appears to be
thickened. The CT scan confirms that the posterior longitudinal ligament is indeed
thickened and ossified, compatible with a diagnosis of ossification of the posterior
longitudinal ligament. This diagnosis is most common in individuals of Japanese
descent and has a genetic linkage. The anterior osteophytes are smaller than those
seen in diffuse idiopathic skeletal hyperostosis and are not syndesmotic. Patients with
ankylosing spondylitis typically have non-marginal syndesmophytes. Patients with
rheumatoid arthritis may have evidence of instability at C1-C2 on flexion-extension
radiographs and subaxial subluxations.
(SAE12SN.11) An 18-year-old man who sustained a lumbar fracture-dislocation with
an associated complete spinal cord injury 6 weeks ago underwent instrumented
posterior thoracolumbar fusion a few days after the injury. While at a rehabilitation
facility, routine postoperative surveillance radiographs are obtained (Figures 11a
through 11d). What is the most appropriate next step in management? Review Topic

1 Observation alone with continued rehabilitation of the spinal cord injury


2 Thoracolumbosacral orthosis bracing for 3 months
3 Revision and extension of the posterior instrumentation and fusion
4 Anterior lumbar corpectomy and fusion
5 Anterior/posterior lumbar decompression and fusion

PREFERRED RESPONSE 3

The patient has sustained a traumatic spondylolisthesis at the level below the caudal
instrumented level, likely not appreciated at the index surgery. Surveillance
radiographs indicate that there is significant translation in the lumbar spine on sitting,
indicating an unstable injury. The lack of significant bony involvement indicates that
the injury is predominantly through the anterior and posterior ligamentous complexes,
and thus is unlikely to stabilize with nonsurgical management. Because the patient
reduces almost completely on lying supine, the most appropriate course of action is
extension of the posterior fusion to include the level of the instability. Because the
patient has a complete spinal cord injury below the level of the thoracic fracture,
decompression is not indicated.

(SAE09SN.56) A 24-year-old man who was involved in a high speed motor vehicle
accident is transferred for definitive care after having been diagnosed with an acute
spinal cord injury from a fracture-dislocation at C6-7. He has a complete C6
neurologic level and it is now approximately 10 hours from his injury. What is the
most appropriate pharmacologic treatment at this time? Review Topic
1 No pharmacologic intervention is recommended at this time
2 Administration of methylprednisolone with an initial bolus of 30 mg/kg followed by
5.4 mg/kg for 24 hours
3 Administration of methylprednisolone with an initial bolus of 30 mg/kg followed by
5.4 mg/kg for 48 hours
4 Administration of naloxone with an initial bolus of 30 mg/kg followed by 5.4 mg/kg
for 24 hours
5 Administration of naloxone with an initial bolus of 30 mg/kg followed by 5.4 mg/kg
for 48 hours

PREFERRED RESPONSE 1

The standard practice in the pharmacologic treatment of a spinal cord injury in the
United States has been the administration of methylprednisolone with an initial bolus
of 30 mg/kg followed by 5.4 mg/kg for 24 hours, in accordance with the findings of
the second and third National Acute Spinal Cord Injury Studies (NASCIS). Although
the studies have subsequently drawn criticism for their methodology and outcomes, it
has been generally accepted that beneficial neurologic outcomes were anticipated in
patients who were able to start the protocol within 8 hours of their initial injury.
Further improvement was noted in patients receiving the methylprednisolone within 3
hours of their injury and continuing an infusion for 48 hours. In this patient, who is
outside the 8-hour treatment window, no studies have supported starting the
methylprednisolone protocol at this time.

(SAE12SN.8) A 46-year-old man has a recurrent disk herniation on the left side at
L4-5 and has had persistent radicular symptoms for 3 months without neurologic
deficit. His previous surgery was performed 13 months ago. What is the best method
of surgical treatment? Review Topic

1 Posterior lumbar interbody fusion


2 Anterior lumbar interbody fusion
3 Revision diskectomy
4 Artificial disk replacement
5 Interspinous process spacer

PREFERRED RESPONSE 3

The patient is a candidate for a revision diskectomy. Outcomes of revision


diskectomy are nearly as good as those reported for primary diskectomy. Reduction in
leg pain is expected. An indication for fusion would be a concomitant instability at the
planned surgical level. Though controversial, artificial disk replacement might be
considered if there is a significant component of diskogenic pain without instability.
However, disk replacement is not the first choice of treatment in this case.
Interspinous process spacers are not currently indicated in this situation.

(SAE07PE.89) Where is the most common site for tuberculosis (TB) spondylitis in
children? Review Topic

1 Anterior aspect of the lower thoracic region


2 Anterior aspect of the cervical spine
3 Posterior elements of the lower thoracic region
4 Posterior elements of the cervical spine
5 Transverse process of the lower lumbar spine

PREFERRED RESPONSE 1

In children, the main route of infection in skeletal TB is through hematogenous spread


from a primary source. The mycobacterium is deposited in the end arterials in the
vertebral body adjacent to the anterior aspect of the vertebral end plate. Thus, the
anterior portion of the vertebral body is most commonly involved. The lower thoracic
region is the most common segment; next in decreasing order of frequency are the
lumbar, upper thoracic, cervical, and sacral regions.

(SAE12SN.28) Figures 28a and 28b show the posteroanterior and lateral radiographs
of a 38-year-old woman with adult idiopathic scoliosis. She reports symptoms of
long-standing lower back pain, progressive loss of height, and the inability to stand
upright at the end of the day. What radiographic finding has been found to most
closely correlate with symptoms of lower back pain? Review Topic
1 Thoracic scoliosis
2 Thoracic hypokyphosis
3 Lumbar disk degeneration
4 Thoracolumbar kyphosis
5 Lumbar hyperlordosis

PREFERRED RESPONSE 4

Adult idiopathic scoliosis and adult "de-novo" scoliosis can present with a number of
symptoms that relate to associated degenerative findings such as stenosis or
spondylolisthesis. In the absence of these associated conditions, increased levels of
pain in patients with scoliosis has been found to most closely correlate with sagittal
imbalance. Thoracolumbar and lumbar curves and thoracolumbar kyphosis have both
been found to closely correlate with increased symptoms and lower health-related
quality of life (HRQL) outcome scores. Thoracic scoliosis, thoracic hypokyphosis,
lumbar hyperlordosis, and lumbar disk degeneration have not been found to correlate
with increased symptoms.

(SAE09SN.60) A 45-year-old man undergoes an anterior cervical diskectomy and


fusion at C5-6 and C6-7 with instrumentation. During the first postoperative visit at 1
week, the patient reports difficulty swallowing and mild anterior cervical tightness.
The anterior wound is benign and the patient denies any dyspnea or shortness of
breath. A postoperative radiograph is seen in Figure 25. What is the most appropriate
management at this time? Review Topic
1 Admit for observation and reassurance
2 Surgical exploration and removal of the anterior instrumentation
3 Esophageal swallowing study
4 Soft cervical collar and early range-of-motion exercises
5 CT of the cervical spine

PREFERRED RESPONSE 1

The radiograph shows significant prevertebral soft-tissue swelling following a two-


level anterior cervical diskectomy and fusion. The incidence of dysphagia 2 years
after anterior cervical spine surgery is 13.6%. Risk factors for long-term dysphagia
after anterior cervical spine surgery include gender, revision surgeries, and multilevel
surgeries. The use of instrumentation, higher levels, or corpectomy versus diskectomy
did not significantly increase the prevalence of dysphagia. Lee and associates
demonstrated that while dysphagia after anterior cervical spine surgery is a common
early finding, it generally decreases significantly by 6 months with nonsurgical
management. A minority of patients experience moderate or severe symptoms by 6
months after the procedure. Female gender and multiple surgical levels have been
identified as risk factors for the development of postoperative dysphagia.

(SBQ09SP.1) A 45-year-old construction worker presents to your office with a chief


complaint of 3 weeks of low back pain. He states the pain started while he was at
work and he now has a difficult time working a full day. He reports the pain is worse
with activity but he cannot describe any certain positions that make it worse. He
denies any recent history of fever, night sweats, or weight loss. He reports normal
bowel and bladder function and denies symptoms radiating into his buttocks or legs.
On physical exam he has 5/5 strength in his lower extremities, a normal sensory and
reflex exam. Forward flexion of the lumbar spine elicits pain in his lower back. He is
able to touch his knees when bending forward, but is unable to touch his toes
secondary to pain. What is the most appropriate next step in management? Review
Topic
1 Lumbar spine x-rays
2 Lumbar spine MRI
3 Lumbar discography
4 Oral methylprednisoline taper
5 Physical therapy and NSAIDs

PREFERRED RESPONSE 5

The patient has classic lumbago with no concerning neurologic or systemic


symptoms. Initial treatment should consist of physical therapy and NSAIDs. No
further workup is necessary at the initial visit since he has no constitutional or
neurologic symptoms.

The evaluation of low back pain can be challenging. While the vast majority of these
can be treated conservatively without x-rays or MRI, it is critical to rule out more
serious conditions such as vertebral osteomyelitis and metastatic disease to the spine.
Risk factors such as diabetes and IV drug abuse increase suspicion for infection. For
metastatic disease, either a history of cancer, smoking, or night pain warrants further
evaluation. If the patient does not have such risk factors, has a clear trigger for the
back pain, or has a history of similar back pain events, then conservative treatment
with physical therapy and NSAIDs is sufficient.

Carragee et al. demonstrated that discography can lead to an increased risk of disc
degeneration and disc herniation, so it should not be used routinely.

Engers et al. preformed a Cochrane review on the treatment of acute and subacute low
back pain and determined that intensive patient education, such as physical therapy, is
effective for decreasing the pain.

Simmons et al. published a review that established criteria for which patients with low
back pain need x-rays at their initial visit. In general, x-rays are unnecessary for
patients with low back pain if:
1) it is the patient's first episode
2) the symptoms have been present for less than 7 weeks
3) the patient has not undergone any treatment
4) the patient's symptoms are improving

Incorrect Answers:
Answers 1 & 2: This patient's symptoms have only been present for 3 weeks, he is
neurologically intact, and he does not have any risk factors for vertebral osteomyelitis
or metastatic cancer to the bone. Therefore, x-rays and MRI are not indicated.
Answer 3: Discography can lead to an increased risk of disc degeneration and disc
herniation, so it should not be used in the initial management of low back pain.
Answer 4: While the use of a methylprednisoline dose pack is common place in the
treatment of acute radiculopathy, there is no clear evidence to support its use with
acute low back pain.
(SAE12SN.3) A 44-year-old man was involved in a low speed rear-end motor vehicle
accident 4 weeks ago. He predominantly reports pain in the back of the neck, with
occasional radiation into the trapezius region bilaterally. He denies any extremity
pain. The pain has not changed in intensity, but is worse with neck range of motion.
Cervical spine radiographs were negative for acute osseous trauma or instability.
What is the next most appropriate step in management? Review Topic

1 Continued observation
2 Cervical epidural injections
3 Nonsteroidal anti-inflammatory drugs (NSAIDs), activity modification, and
physical therapy
4 Cervical facet blocks
5 Cervical MRI

PREFERRED RESPONSE 3

The patient was involved in a low speed rear-end collision and sustained a whiplash-
type injury, with management most often being nonsurgical. After 4 weeks of
persistent pain, continued observation is not reasonable. Studies have shown that
treatment including NSAIDs, activity modification and a brief duration of physical
therapy allows for improved outcomes after whiplash-type injuries when compared
with observation alone. An MRI scan of the cervical spine is not indicated at this time
and represents an unnecessary expense. Cervical epidural and facet injections are not
indicated in the treatment of patients with whiplash injuries.

(OBQ06.24) A 12-year-old boy is emergently transported to the emergency


department following a motor vehicle accident. He was restrained in the back seat
with a lap belt. On a physical exam bruising is noted across his abdomen as shown in
Figure A. Lateral radiographs are shown in Figure B. Which of the following injuries
are most frequently associated with this injury pattern? Review Topic
1 Cardiac tamponade
2 Pulmonary contusion
3 Colonic rupture
4 Bladder rupture
5 Acute cholecystitis

PREFERRED RESPONSE 3

The clinical presentation is consistent with a Chance fracture of the spine. These
fractures are often associated with concomitant bowel injury.

A Chance fracture injury is a flexion-distraction injury of the spine. The anterior


column (vertebral body) collapses under compression and the posterior elements fail
under tension (rupture of the interspinous ligaments or avulsion fracture of the
spinous process). A common mechanism is a MVA where the child is wearing a
seatbelt, leading to a "seatbelt sign". In the presence of a "seatbelt" sign, on should
have a high suspicion for a bowel injury.

Reid et al. reported seven cases of pediatric patients with Chance fractures. All had
associated intraabdominal bowel injury.

Holland et al. retrospectively reviewed 28 pediatric patients with Chance fractures,


71% sustained following road trauma. Eleven percent had associated small bowel
injury, but all patients had abnormal abdominal CT scans.

Figure A is a clinical photo of a 'seatbelt sign' following a motor vehicle accident.


Figure B exhibits a "bony" Chance fracture where the spinous process has been
avulsed. Illustration A depict the failure of the anterior column under compression
and failure of the posterior column under tension.

Incorrect answers:
Answers 1, 2, 4, 5 These injuries are not highly associated with pediatric Chance
fractures following motor vehicle trauma.
(SBQ12SP.32) A 48-year-old man is involved in a motor vehicle accident and is taken
to an outside hospital where he undergoes CT imaging, displayed in Figures A-B.
Approximately 36 hours later he is transferred to your hospital for further evaluation
and management. On exam, he has tenderness over his upper cervical spine and is
neurologically intact with no myelopathic signs. What is the most appropriate
treatment method for this patient and why? Review Topic

1 Soft cervical orthosis because his gender puts him at a low risk of nonunion.
2 Halo vest immobilization because the degree of fracture displacement puts him at a
low risk of nonunion.
3 Anterior screw fixation because his delayed time to treatment puts him at an high
risk of nonunion.
4 Posterior C1-C2 fusion with rigid instrumentation because his age puts him at a high
risk of nonunion.
5 Posterior C1-C2 fusion with sublaminar wiring because the degree of fracture
angulation puts him at a high risk of nonunion.

PREFERRED RESPONSE 2

The patient has a type II odontoid fracture and should be managed with halo vest
immobilization as he has minimal fracture displacement (< 5mm) and no other risk
factors for nonunion.

Type II odontoid fractures comprise 35% of all C2 fractures and have the highest
nonunion rate. Commonly cited risk factors for nonunion include posterior
displacement, posterior angulation, delayed initiation of treatment, fracture
comminution and advanced age. Patients without these risk factors can often be
successfully managed with halo vest immobilization. Elderly patients are frequently
unable to tolerate halo vest immobilization and can be placed in a rigid cervical
orthosis. Patients at high risk for nonunion are generally managed surgically with
either anterior screw fixation, posterior rigid fixation (C1-C2 transarticular screw
construct versus C1 lateral mass screw + C2 pedicle screw construct), or posterior C1-
C2 sublaminar wiring.

Greene et al found that type II odonotoid fractures that were displaced >/= 6mm had a
nonunion rate of 86% compared to 18% in patients with < 6mm displacement. This
was statistically significant regardless of direction of displacement. Age was not
significantly associated with nonunion.
Koivikko et al performed a retrospective review of conservatively treated type II
odontoid fractures and identified risk factors for nonunion. Fracture gap > 1mm,
posterior displacement > 5mm, posterior angulation > 20°, delayed start of treatment
> 4 days and posterior redisplacement > 2mm were all correlated with nonunion. In
this study, anterior displacement, gender and age were unrelated to nonunion.

Figures A and B are coronal and sagittal CT scans of the cervical spine, respectively,
demonstrating a noncomminuted type II odontoid fracture with minimal posterior
displacement and angulation.

Incorrect Responses:
Answer 1: Gender is not associated with nonunion. A soft cervical orthosis does not
provide adequate stability.
Answer 3: The patients time to treatment (36 hours) is not delayed and therefore does
not confer an increased risk of nonunion.
Answer 4: The impact of age on nonunion risk is controversial. The two studies above
conclude there is no correlation between age and nonunion risk; however, some
studies have shown that age > 65-years is a risk factor for nonunion; Regardless, this
patient is 48-years-old and therefore not at an elevated risk of nonunion secondary to
his age.
Answer 5: The posterior angulation is < 20° and therefore does not confer an
increased risk of nonunion.

(SAE09SN.2) A patient who is an observant Jehovah’s Witness requires major


surgery for scoliosis that will likely result in significant blood loss. Which of the
following might the patient consider allowing the surgical team to use? Review Topic

1 Transfusion of whole blood


2 Transfusion of packed red blood cells
3 A cell saver with continuity maintained in a “closed circuit”
4 Transfusion of plasma
5 Transfusion of platelets

PREFERRED RESPONSE 3

Jehovah’s Witnesses will not accept the transfusion of blood or blood products such
as packed red or white cells, platelets, or plasma. However, many Jehovah’s
Witnesses will accept the use of a cell saver in a “closed circuit.”
(SAE09SN.45) What is one of the principle concerns when a fracture such as the one
seen in Figure 18 is encountered? Review Topic

1 Fractures of the lower extremities


2 Paroxysmal hypertension
3 Infection
4 Epidural hematoma
5 Gastrointestinal bleeding

PREFERRED RESPONSE 4

The injury shown is a fracture-dislocation and it is highly unstable. In addition to this


concern, spinal epidural hematomas have a much higher incidence in people with
ankylosing spondylitis following spine fracture. It is felt to be due to disrupted
epidural veins, with hypervascular epidural soft tissue in the setting of a rigid spinal
canal. Patients with ankylosing spondylitis may have other significant comorbidities,
especially cardiac and pulmonary, and these should be carefully assessed.

(SAE10PE.21) A 14-year-old boy is seen for back pain. Radiographic evaluation


reveals a grade III isthmic spondylolisthesis. What measurement is most useful in
predicting the likelihood of progression? Review Topic

1 Pelvic incidence
2 Slip angle
3 Sacral inclination
4 Lumbosacral joint angle
5 Sagittal rotation
PREFERRED RESPONSE 2

Slip angle has been shown to be highly predictive of the risk for increased slippage in
patients with spondylolisthesis. None of the other radiographic parameters listed has
been shown to be predictive of the risk for increased slippage.

(SAE12SN.17) Of the following signs or findings, which one is most consistent with
the diagnosis of cervical radiculopathy? Review Topic

1 Spurling sign
2 Hoffman sign
3 Clonus
4 Inverted brachioradialis reflex
5 Babinski sign

PREFERRED RESPONSE 1

The Spurling sign is elicited by extending the neck and having the patient rotate his or
her head toward the side of the symptoms; reproduction of symptoms, including those
of radicular pain, suggests cervical nerve root compression as a contributing factor.
The remaining signs and clinical findings are seen with cervical myelopathy. The
Hoffman sign is elicited by flicking the terminal phalanx of the third or fourth finger;
a positive response is seen as reflex flexion of the terminal phalanx of the thumb. The
inverted brachioradialis reflex is seen when the brachioradialis tendon is tapped and a
diminished brachioradialis reflex is noted but reflex contraction of the finger flexors is
seen.

(SAE10SM.13) A 19-year-old linebacker for a collegiate football team has had two
episodes of bilateral arm tingling and weakness after tackling; the symptoms resolved
after 30 minutes of rest. Three follow-up neurologic examinations have been normal.
Cervical spine CT and MRI scans are shown in Figures 13a through 13c. What is the
next best step in management? Review Topic
1 The addition of a neck roll to the helmet and continuation of play
2 Electrodiagnostic studies
3 A series of epidural steroid injections, followed by a return to play
4 Methylprednisolone dose pack, followed by a return to play in 1 week
5 No further participation in football

PREFERRED RESPONSE 5

Cervical spinal stenosis is a contraindication to participation in collision and contact


sports. Previously, the risks of permanent quadriparesis from cervical spinal stenosis
were thought to be unclear and athletes with cervical spinal stenosis were often
allowed to play contact sports. In 1996, Torg and associates reported that
developmental narrowing of the cervical canal in a stable patient does not appear to
predispose an individual to permanent catastrophic neurologic injury and therefore
should not preclude an athlete from participation in contact sports. However, the
current understanding is that the actual risks of permanent neurologic injury from
cervical stenosis are significant. The Torg ratio was previously used for diagnosis but
is more recently thought to be of low predictive value as reported by Cantu. Current
methods for diagnosis of cervical spinal stenosis rely on MRI and CT. Current
diagnosis is based on comparisons of measurements with normal values. A cervical
canal of less than 13 mm is considered stenotic whereas a diameter of less than 10
mm is considered absolute stenosis as reported by Crowl and Kong. This patient has
symptomatic stenosis and should not be cleared for contact sports. A neck roll will not
prevent neurologic injury in the presence of cervical spinal stenosis. Electrodiagnostic
studies are not likely to add any additional significant findings with central canal
stenosis. Cervical traction is not of value in the long-term. Epidural steroid injections
or a methylprednisolone dose pack are not of value in this situation.

(SAE12SN.59) A 40-year-old woman is admitted to the hospital with a history of


new-onset right lower extremity weakness resulting in frequent falls. She reports that
a few weeks prior to the onset of the lower extremity symptoms, she experienced an
episode of acute back pain, which has since resolved. Examination reveals 1-2/5
strength in the right hip flexors, abductors, and quadriceps. There is 0-1/5 strength
noted in the right ankle plantar and dorsiflexors. Deep tendon reflexes are hyperactive
in the lower extremities and there is clonus on dorsiflexion of the right ankle. The
patient has a positive Babinski sign on the right side. The patient denies neck or arm
symptoms, and the upper extremity neurologic examination is within normal limits.
Figures 59a and 59b show the MRI scans of the patient's cervical and lumbar spine.
What is the next most appropriate step in management? Review Topic

1 Multilevel anterior cervical diskectomy and fusion


2 Cervical laminectomy
3 Lumbar laminectomy
4 Lumbar laminectomy and fusion
5 Thoracic MRI

PREFERRED RESPONSE 5

Symptoms of thoracic disk herniation may present in one of three distinct patterns:
axial pain, radicular pain, or myelopathy. This patient demonstrates obvious upper
motor neuron signs, including lower extremity hyperflexia, clonus, and a positive
Babinski sign; therefore, the next most appropriate step is a MRI scan of the thoracic
spine. Thoracic disk herniations presenting with axial pain usually involve the mid to
lower thoracic region. The pain is usually worsened with activity and improved with
rest. Radicular pain syndromes are usually described as starting in the back and
radiating anteriorly in a band-like pattern to the anterior chest wall. The pain and
paresthesias generally follow a dermatomal distribution. The MRI scan of this
patient's cervical spine demonstrates spondylotic changes; however, she denied neck
or upper extremity symptoms and more importantly, has a normal neurologic
examination. There are degenerative changes involving the lumbar spine, but these
cannot explain the upper motor neuron findings.

(SBQ12SP.39) Which of the following is associated with increased risk of surgical


site infection following lumbar spinal decompression? Review Topic
1 Use of intra-wound vancomycin powder
2 Hair removal by clipping at the time of surgery
3 Use of morphine nerve paste applied to the dura after spinal decompression
4 Skin preparation with an alcohol-based prep solution rather than povidone iodine
5 Failure to use chlorhexidine gluconate wash the night prior to surgery

PREFERRED RESPONSE 3

The use of morphine nerve paste applied to the dura after decompressive spine
surgery has been used in an effort to mitigate postoperative pain but has been
associated with epidemic levels of surgical site infection.

Surgical site infection risk is modulated by a myriad of factors, both host factors and
perioperative factors. Perioperative strategies to reduce infection that have been
shown to be effective include: 1) hair removal by clipping (not shaving) at the time of
surgery, 2) use of an alcohol based skin preparation agent, 3) use of perioperative
prophylactic antibiotics, 4) limiting operating room traffic from OR personnel.

Kramer et al. did a retrospective analysis of 148 patients who underwent laminectomy
in order to identify risk factors for infection in the context of a sudden increase in
infections. Regression analysis showed that use of morphine paste resulted in an 18-
fold increase in the risk of surgical site infection. 17 of 148 had developed infection
during the epidemic period, compared to 15/995. Cultures were positive in 10 of 16
cases; 7 of these involved skin flora. Histological exam showed foreign body reaction.

Devin et al. did a multi-center analysis of patients undergoing spine surgery at seven
spine centers to determine whether there was a difference in infection risk based on
whether or not intra-wound vancomycin was used. They analyzed 2056 patients, of
whom 47% had vancomycin used in their wounds. The risk of surgical site infection
was higher in those patients that did not have vancomycin (relative risk 2.5).

Incorrect answers:
Answer 1: Intra-wound use of vancomycin has been shown to decrease infection risk
in spine surgery.
Answer 2: Hair removal by clipping (not shaving) decreases surgical site infection
Answer 4: There is some evidence that alcohol-based prep solutions decrease skin
flora more than water-based solutions
Answer 5: Although chlorhexidine gluconate wash is often recommended, it has not
been demonstrated to lower the risk of surgical site infection.
(SAE13PE.50) The AP radiograph of a 5-year-old boy shows a 20-degree left thoracic
scoliosis. He was noted by his pediatrician to have asymmetry on a forward bend test.
On examination he is neurologically intact except for decreased sensation on the
lateral aspect of both flanks and to pinprick in both hands. He has no pain. What is the
best initial step in treatment at this time? Review Topic

1 Observation
2 Initiate bracing
3 MRI scan of the entire spine
4 Spinal instrumentation with growing rod construct

PREFERRED RESPONSE 3

This patient has atypical scoliosis, given his young age and left thoracic curve. In
addition, he has abnormal neurologic findings. MRI scan to evaluate for neural axis
abnormalities is indicated. Abnormal MRI findings are present in 2% to 3.8% of all
patients with presumed idiopathic scoliosis. Abnormal MRI findings are more likely if
specific clinical factors are present, such as absence of thoracic apical segment
lordosis, atypical curve pattern, an abnormal neurologic examination, male gender,
and age younger than 11. In a patient with an atypical curve and neurologic indicators,
the yield of MRI scan for a neuraxis abnormality has been shown to be 25%. This
patient had both syringomyelia and a Chiari malformation that were treated
neurosurgically. Observation would have missed these findings. Bracing or spinal
instrumentation may eventually be treatment options for scoliosis given his young
age, but establishing a diagnosis first with an MRI scan of the spine is the most
appropriate initial step.

(SAE09SN.23) An adult patient with a grade I isthmic spondylolisthesis at L5-S1 is


most likely to have weakness of the Review Topic

1 flexor hallucis longus.


2 quadriceps.
3 gastrocsoleus.
4 extensor hallucis longus.
5 iliopsoas.

PREFERRED RESPONSE 4

Adult patients with isthmic spondylolisthesis most commonly have neurologic


symptoms due to foraminal stenosis at the level of the spondylolisthesis. In this
scenario, the patient is most likely to have weakness of the L5 myotome, which would
cause weakness of the extensor hallucis longus.
(OBQ13.93) A 40-year-old woman with history of intravenous drug abuse and
ongoing Staphylococcus aureus septicemia is referred for intractable neck pain with
radiation down her arm. She also complains of progressive hand weakness.
Examination reveals long tract signs in the lower extremities. Her MRI scan is shown
in Figure A. Besides intravenous antibiotics, what is the most appropriate next step in
treatment? Review Topic

1 Percutaneous CT guided aspiration, hard cervical orthosis until bony union.


2 Percutaneous CT guided aspiration, hard cervical orthosis, repeat aspiration at 6-12
weeks followed by anterior corpectomy and fusion if repeat aspiration is sterile.
3 Anterior cervical debridement and anterior corpectomy without instrumentation
4 Posterior cervical debridement and instrumented posterior fusion.
5 Anterior cervical debridement, corpectomy and fusion followed by a posterior
instrumented stabilization procedure

PREFERRED RESPONSE 5

This patient has spondylodiscitis of C5-C7 with neurological deficit, acute kyphotic
deformity and anterior prevertebral and epidural abscess. The abscess and necrotic
tissue need to be drained via an anterior approach, and the deformity needs to be
corrected and stabilized both anteriorly and posteriorly.

Spinal epidural abscesses are more common in immunodeficient patients and IV drug
abusers. They result in verebral body destruction and instability. In addition to
draining the abscess and necrotic bone, the instability has to be addressed by
instrumentation and fusion, even in the face of infection. Structural support can
include cages with morselized bone graft, or structural allograft fixation of anterior
and middle column and/or pedicle screw instrumentation of the posterior column.

Shousha et al. reviewed the treatment of cervical spondylodiscitis in 30 patients. They


found an epidural abscess in 80% of patients, and noncontiguous discitis in 47% of
cases. Thus, they recommend MRI of the entire spine in all cases, and surgery for
intractable neck pain, septicemia, epidural abscess, neurological compromise, gross
kyphotic deformity with extensive destruction, and failure of conservative treatment.

Heyde et al. reviewed cervical spondylodiscitis in 20 patients. All cases had


postoperative antibiotic therapy for 8–12 weeks. At 37 months (range 24–63), all
cases went on to union and there were no recurrences of infection. There was
improvement in preoperative neurological status in all cases, leading them to
recommend early surgical intervention.

Figure A is an MRI scan showing spondylodiscitis C5–C7 with partial destruction of


the vertebral bodies and epidural abscess formation. Illustration A shows the lateral x-
ray (left) and MRI (right) of the same patient 9 months postoperatively after
corpectomy C5 and C6 and posterior instrumentation.

Incorrect Answers:
Answers 1 and 2: Surgical stabilization is indicated in the presence of epidural
abscess, neurological deficit, gross kyphotic deformity and extensive destruction, and
ongoing septicemia.
Answer 3: Anterior corpectomy without instrumentation will not afford sufficient
stability.
Answer 4: The abscess cannot be adequately debrided via a posterior approach.
Anterior column support is also necessary.

(SAE07PE.63) A 12-year-old girl has back pain after falling 20 feet and landing in the
sitting position. She has no fractures or other injuries, and her neurologic examination
is normal. A lateral radiograph, transverse CT scan, and reformatted sagittal CT scan
are shown in Figures 25a through 25c. Which of the following methods is associated
with the best long-term outcome? Review Topic

1 Hyperextension casting of the thoracolumbar spine for 6 weeks


2 In situ posterior fusion with instrumentation
3 Posterior fusion with instrumentation, with sagittal plane correction
4 Posterior decompression, followed by posterior fusion with instrumentation, with
sagittal plane correction
5 Anterior decompression and partial corpectomy, with anterior instrumentation

PREFERRED RESPONSE 3

The patient has a displaced burst fracture. Fusion with instrumentation has shown
better results than casting alone. Posterior fusion with instrumentation, with sagittal
plane correction, yields the best results. Decompression occurs indirectly with
correction of the kyphosis. Anterior decompression is unnecessary.

(OBQ15.110) Which of the following is the strongest contraindication for expansive


open door laminoplasty for cervical myelopathy? Review Topic

1 Multi-level cervical spondylosis


2 C2-C7 rigid kyphosis of 15 degrees
3 Ossification of the posterior longitudinal ligament
4 C7 sagittal vertical axis of +5cm
5 Compression ratio of 0.3

PREFERRED RESPONSE 2

Expansive open door laminoplasty is a method of posterior cervical decompression. It


is contra-indicated for patients with cervical kyphosis unless the surgical plan
includes concomitant correction of deformity.

Open door laminoplasty allows decompression direct posterior decompression of the


neural elements, as well as an indirect anterior decompression by allowing the cord to
drift posteriorly. Posterior drift of the cord relies upon the presence of natural cervical
lordosis. In the setting of kyphosis, the cord is tensioned ventrally over the vertebral
bodies and discs, and does not have the redundancy to drift backward. Patients
without lordosis would therefore be expected to show less post-operative
improvement than those with normal cervical curvature.

Chiba et al. performed a retrospective review of patients treated with expansive open
door laminoplasty for cervical spondylotic myelopathy (CSM) and ossification of the
posterior longitudinal ligament (OPLL). They found that for patients with OPLL,
cervical kyphosis was associated with lower recovery rates than those patients with
pre-operative lordosis. The authors recommended against posterior decompression for
CSM in the setting of OPLL with concomitant cervical kyphosis.

Suda et al. performed a retrospective review of 114 patients who underwent expansive
open door laminoplasty for cervical myelopathy in order to evaluate clinical outcomes
and effects on cervical alignment. They found patients with C2-C7 kyphosis >13° had
much lower rates of improvement compared with those patients with less kyphosis.
The authors suggest an anterior decompression be used for patients with kyphosis
greater than 13°, unless kyphotic correction was planned in addition to a
laminoplasty.

Illustrations:
Illustration A demonstrates a lateral cervical spine x-ray which shows focal kyphosis
at C4-5 and C5-6. Illustration B demonstrates the C7-sagittal vertical axis. Illustration
C demonstrates the compression ratio.

Incorrect Answers:
Answer 1. Myelopathy caused by multi-level cervical spondylosis is an acceptable
indication for laminoplasty
Answer 3. Myelopathy caused by OPLL is an acceptable indication for laminoplasty
Answer 4. The C7 sagittal vertical axis (C7-SVA) is a measure of global sagittal
alignment, and is defined as the difference in distance between the C7 plumbline and
the posterosuperior corner of S1 body in the sagittal plane. A value of 5cm suggests
global kyphosis, however this it is not clear from this value alone if kyphosis is
occurring in the cervical spine.
Answer 5. The compression ratio for myelopathy is the ratio of the smallest
compressed diameter of the spinal cord to the largest diameter of the cord. A ratio
<0.4 implies a poor prognosis.

(OBQ14.174) A child presents with the radiograph shown in Figure A. Which of the
following conditions is LEAST likely to be associated with this disorder? Review
Topic

1 Unilateral absent kidney


2 Sever's disease
3 Ventricular septal defect
4 Tethered cord
5 Thumb hypoplasia
PREFERRED RESPONSE 2

Sever's disease (calcaneal apophysitis) is not associated with congenital scoliosis.

Congenital scoliosis is associated with other anomalies 60% of the time. These
anomalies can appear independently, or as part of the VACTERL syndrome (vertebral
anomalies, anorectal atresia, tracheoesophageal fistula, and renal and vascular
anomalies). Other associated orthopedic conditions include clubfoot, developmental
dysplasia of the hip, limb hypoplasia, Sprengel’s deformity, Klippel-Feil syndrome,
foot asymmetry, vertical tali, leg atrophy and pes cavus.

Hedequist et al. (2004) reviewed congenital scoliosis. They recommend surgery in


young children, severe deformities, or deformities that tend to progress rapidly,
truncal imbalance, and anomalies at the cervicothoracic and lumbosacral junction
(because of imbalance in the shoulders/neck and lumbar region respectively). Surgical
options include in situ fusion, convex hemiepiphysiodesis, hemivertebra excision,
correction and instrumented fusion, osteotomies with fusion, growing rods and
expandable ribs.

Hedequist et al. (2007) reviewed congenital scoliosis. They state that fully segmented
hemivertebra with definable disks above and below are more likely to cause curvature
compared with an unsegmented hemivertebra fused to the vertebra above and below.
Also, anomalies at the cervicothoracic and lumbosacral junctions produce more
visible deformities than that at other areas.

Figure A shows a spine with multiple hemivertebrae, examples of failure of formation


in congenital scoliosis.

Incorrect Answers:
Answer 1: Up to 20% of patients may have a urological anomaly that can include
unilateral absent kidneys, obstructive uropathy, horseshoe kidney, renal aplasia,
duplicate ureters, and hypospadias. Evaluation by MRI or ultrasound is needed.
Answer 3: Up to 26% of patients have cardiac defects. Ventricular septal defect is the
most common, followed by atrial septal defects. Other defects include patent ductus
arteriosus, tetralogy of Fallot and transposition of great vessels.
Answer 4: Intraspinal abnormalities occur in up to 37% of patients. Risk is higher
with segmentation and mixed defects. Anomalies include diastematomyelia,
syringomyelia, tethered cord, dural bands, cysts and tight filum terminale.
Answer 5: Thumb hypoplasia and radial club hand are limb defects found in
VACTERL syndrome and are associated with congenital scoliosis.
(OBQ14.237) Which of the following correctly describes a physical or molecular
change in the degenerative pathway of intervertebral disk disease? Review Topic

1 Decreased type II collagen in the nucleus pulposis


2 Increased water content in the nucleus pulposis
3 Increased type IX collagen in the nucleus pulposis
4 Decrease in the keratin sulfate-to-chondroitin sulfate ratio in the nucleus pulposis
5 Decrease in type I collagen in the annular fibrosus

PREFERRED RESPONSE 1

A reduction in type II collagen synthesis is a pathological process in the degenerative


pathway of intervertebral disk disease (IVD).

There are many physical and molecular changes in the degenerative pathway of
intervertebral disk disease. These include:
i) DECREASE: Synthesis of collagen types II and IX, nutritional transport, water
content, absolute number of viable cells, proteoglycans and pH.
ii) INCREASE: Synthesis of collagen types I and X, keratin sulfate-to-chondroitin
sulfate ratio, lactate and degradative enzyme activity.

Kepler et al. reviewed intervertebral disk degeneration and emerging biologic


treatments. They report treatment of IVD is challenging, as discs are avascular
structures and the delivery of therapeutic proteins via the bloodstream is limited.
Methods considered to enhance disk cellularity include in vitro expansion and
subsequent reimplantation of disk tissue, mesenchymal stem cell (MSC) implantation,
and injection of platelet-rich plasma (PRP) into the disk.

Illustration A is a non-exhaustive list of potential biological treatments for IVD (c.


2014)

Incorrect Answers:
Answer 2: Water content has been shown to decrease in intervertebral disk disease.
Answer 3: Synthesis of type IX collagen has been shown to decrease.
Answer 4: There is an increase in the keratin sulfate-to-chondroitin sulfate ratio in
intervertebral disk disease.
Answer 5: Synthesis of type I collagen has been shown to increase in intervertebral
disk disease.

(SBQ12SP.49) As part of his treatment plan for congenital scoliosis, a 7-year-old boy
is placed into halo gravity traction using 10 pins, inserted into the thickest part of the
skull, with a torque wrench to apply an insertional torque of 6 in-lb. After 1 week, he
is noted to have developed a complication at one pin site. Which of following changes
in technique would have been recommended to reduce the impact of pin
complications? Review Topic

1 Using more pins


2 Using fewer pins
3 Applying a higher insertional torque
4 Applying a lower insertional torque
5 Inserting pins by palpation without a torque wrench

PREFERRED RESPONSE 4

The recommended insertional torque for halo application in preadolescent patients is


1 to 5 in-lb. The insertional torque used in this case was too high.

Halo application is performed in children for multiple conditions, most often


stabilization for cervical spine trauma (as definitive treatment or in conjunction with
operative management) and for traction as part of a larger treatment plan for severe
curves in scoliosis. Due to reduced thickness of the pediatric skull, higher rate of pin
site infections and skull puncture, and unreliability of many torque wrenches, current
recommendations are to use a higher number of pins (8 to 12) with lower insertional
torque (1 to 5 in-lb) in case pins need to be removed or exchanged.

Caird et al. conducted a retrospective review of 13 patients under 4 years old who had
halo application for cervical trauma, arthrodesis, or scoliosis. They reported an
average of 8 pins used per patient (range 6-12) and insertional torque of 2 to 4 in-lb.
Nine patients (69%) suffered some complication, mostly pin site infection (6 cases),
and so they conclude halo application is safe in toddlers, but ambulation should be
limited.

Copley et al. performed a study manually comparing the accuracy at low settings of
torque wrenches from 4 different manufacturers. They found that overall, the
accuracy was within 10% of the intended torque in only 69.2% of all trials, and that
Jerome and Mountz wrenches were more accurate than PMT and Bremer at the low
settings recommended for pediatric patients.

Limpaphayom et al. looks at complication rates of using halo traction for correction of
spinal deformity or immobilization. They found an overall complication rate of 53%
(36/68), and 10% (7/68) of children required unanticipated surgery for treatment of
these complications. The most common complication was pin-site infections, with
76% (13/17) of these resolving with oral antibiotics alone. Traction-related neurologic
injuries that occurred were common, 31% (9/31) but all resolved with a decrease or
removal of traction weight, with complete resolution occurring immediately in 4 of 9
events.

Incorrect Answers:
Answer 1 and 2: 10 pins is an appropriate amount for young children.
Answer 3: 6 in-lb is already higher than the recommended torque, so lower torque
should be used.
Answer 5: Despite the unreliability of some torque wrenches, it has not been shown
that hand tightening is superior or associated with fewer complications.

(SAE09SN.59) The best patient-related outcomes, following the surgical treatment of


cauda equina syndrome secondary to a large L5-S1 disk herniation, are most closely
related to which of the following? Review Topic

1 Extent of bowel and bladder dysfunction


2 Extent of the motor deficit
3 Extent of the perianal saddle anesthesia
4 Timing of surgery
5 Location of the herniation

PREFERRED RESPONSE 4

The most predictable positive outcome from spinal surgery due to a cauda equina
syndrome is early surgical intervention before any significant neurologic deficit
develops. Meta-analysis studies demonstrate that surgical intervention more than 48
hours after the onset of cauda equina syndrome show an increased risk for poor
outcomes.

(OBQ14.38) A 16-year-old female with adolescent idiopathic scoliosis undergoes


posterior spinal fusion with instrumentation. The thoracic pedicle screws were placed
using a tap 1 mm smaller than the screw diameter and a straightforward trajectory that
runs parallel to the superior endplate. This techniques allows for which of the
following: Review Topic

1 Anatomic placement of the screws.


2 Increased depth insertion of the screws.
3 Increased maximal insertional torque.
4 Decreased resistance to screw pullout.
5 Decreased stability of the construct.
PREFERRED RESPONSE 3

Straightforward trajectory when placing pedicle screws in addition to prior tapping


1mm smaller than the screw diameter increase the maximal insertional torque and
resistance to screw pullout.

Contemporary segmental pedicle screw placement used in the treatment of scoliosis


deformity offer significantly higher screw pullout and deformity correction than prior
hook and wire constructs. Additionally, screw insertional torque has been found in
numerous studies to correlate with resistance to screw pullout. Several factors have
been found to increase maximum screw insertional torque, including tapping 1mm
smaller than the screw diameter and using the straightforward trajectory. It is
important to note that while undertapping makes for a stronger screw, there are some
studies that suggest not tapping at all makes for an even stronger screw.

Lehman et al. performed a biomechanical study evaluating maximum insertional


torque when tapping line to line, undertapping by 0.5mm, and undertapping by 1mm
in 34 fresh frozen cadavers. They found undertapping the thoracic pedicle by 1mm
increased maximum insertional torque by 47% when compared to undertapping by
0.5mm and by 93% when compared to line to line tapping.

Kuklo et al. performed a biomechanical study on thirty cadavers using the


straightforward technique (sagittal trajectory of the screws parallels the superior
endplate of the vertebral body) versus anatomic trajectory (22 degrees in the cephalo-
caudad direction in the sagittal plane). They found maximum insertional torque to be
2.58 pounds using the straightforward technique versus 1.86 pounds using the
anatomic trajectory (p=0.0005).

Incorrect Answers:
Answer 1: These techniques do not allow for anatomic placement of the screws. An
anatomic trajectory would also decrease the maximum insertional torque.
Answer 2: Depth insertion is not based upon tapping or using the straightforward
technique.
Answer 4: Resistance to screw pullout is directly correlated with maximum
insertional torque, and would be increased by undertapping 1mm and by using the
straightforward technique.
Answer 5: Stability of the construct is increased by using the techniques described in
the question as the maximum insertional torque and resistance to screw pullout would
increase.

(SAE13PE.79) What is the most common physical examination finding in a patient


with chronic painful spondylolysis? Review Topic

1 Positive straight leg raise


2 Pain with forward flexion
3 Pain with lumbar extension
4 Absent tendo-Achilles reflex

PREFERRED RESPONSE 3

Patients with spondylolysis typically demonstrate increased pain with lumbar


extension, not with forward flexion. In the absence of a disk herniation, a straight leg
raise test result should be negative. Pain with forward flexion is not common in
spondylolysis, and without nerve root impingement there should be no loss of the
tendo-Achilles reflex.

(SAE12SN.42) Figures 42a through 42c show the MRI scans of a 56-year-old woman
with progressively worsening low back and bilateral lower extremity pain. Based on
these images, what muscle or muscle group would be expected to be weak on physical
examination? Review Topic

1 Quadriceps
2 Hamstrings
3 Hip adductors
4 Extensor hallucis longus
5 Gastrocnemius

PREFERRED RESPONSE 4

Whereas subjective complaints of leg pain are common among patients seeking
surgical treatment for spondylolisthesis, documented neurologic deficit or
radiculopathy is seen less frequently. Subjective decreases to light touch over the
dorsum of the foot and mild weakness of the extensor hallucis longus are the most
common neurologic abnormalities, correlating with L5 nerve root irritation as seen
with L5-S1 spondylolisthesis. Many patients with spondylolisthesis report hamstring
tightness; however, these structures are not usually weak. Quadriceps and tibialis
anterior weakness is seen with L4 nerve root irritation. The gastrocnemius is generally
weak in S1 nerve root syndromes.

(SAE09SN.18) A 51-year-old woman with no preoperative neurologic deficit is


undergoing elective anterior cervical diskectomy and fusion (ACDF) with plating and
fusion for a C5-6 disk herniation with right-sided neck pain. Thirty minutes into the
surgery the neurophysiologic monitoring shows a rapid drop and then loss of
amplitude in the right cortical somatosensory-evoked potential waveform. All other
waveforms remained normal and unchanged, including right-sided cervical
(subcortical) and peripheral (Erb’s point), and those from the left-sided upper
extremity and both lower extremities. What is the most likely cause of the change?
Review Topic

1 Electrode placement
2 Stimulation failure
3 Anesthetic effect
4 Cord ischemia from retraction
5 Cerebral ischemia from retraction

PREFERRED RESPONSE 5

The change noted is focal and confined to the cortex, sparing the opposite side, both
lower extremities, and the subcortical waveforms, making all the choices unlikely
with the exception of carotid compression with focal cortical ischemia. This may be
associated with poor collateral flow from the opposite hemisphere due to an
incomplete circle of Willis.

(SAE09SN.31) Figures 11a and 11b show the T2-weighted MRI scans of the lumbar
spine of a 53-year-old woman who has low back and right lower extremity pain. What
structure is the arrow pointing to in Figure 11a? Review Topic
1 Ligamentum flavum
2 Lumbar synovial cyst
3 Tarlov cyst
4 Pseudomeningocele
5 Herniated nucleus pulposus

PREFERRED RESPONSE 2

The arrow is pointing to a cystic-appearing structure with high signal intensity on T 2-


weighted image sequencing. It appears to be contiguous with the hypertrophied right
facet joint, which appears to also have high signal intensity. The mass significantly
narrows the right lateral recess. The high signal intensity suggests that this is a fluid-
filled mass. In addition, the facet joints are degenerative and there is a very mild
degree of anterolisthesis on the sagittal image. These findings make a lumbar synovial
cyst the most likely diagnosis. Most lumbar juxtafacet cysts are observed at the L4-5
level, extradurally and adjacent to the degenerative facet joint. They may contain
synovial fluid and/or extruded synovium. Presentation is indistinguishable from that
of a herniated disk. The etiology of spinal cysts remains unclear, but there appears to
be a strong association between their formation and worsening spinal instability. They
occasionally regress spontaneously and may respond to aspiration and injection of
corticosteroids, though there is a high recurrence rate with nonsurgical management.
Synovial cysts resistant to nonsurgical management should be treated surgically. If the
patient’s symptoms can be attributable to radicular findings, a microsurgical
decompression that limits further destabilization should suffice. However, if there is
significant low back pain attributable to spinal instability, decompression and fusion
remains an appropriate option.

(SAE12SN.1) A 56-year-old man has a chief complaint of leg weakness and inability
to walk. Examination reveals 5 out of 5 motor strength in all lower extremity muscle
groups tested and normal sensation to light touch in both lower extremities. The
patient is slow in getting up from a seated position and has an unsteady wide-based
gait. An MRI scan of the lumbar spine is shown in Figure 1. What is the next most
appropriate course of action? Review Topic

1 Electromyography and nerve conduction velocity studies of bilateral lower


extremities
2 Multilevel lumbar laminectomy
3 MRI of the thoracic and cervical spine
4 MRI of the brain
5 Epidural steroid injections

PREFERRED RESPONSE 3

The patient is having gait problems suspicious for spinal cord compression. MRI of
the thoracic and cervical spine should be performed to evaluate for spinal cord
compression. Reports of leg weakness in the absence of discrete motor weakness on
manual testing, and the appearance of an unsteady wide-based gait are more
consistent with myelopathy as a cause of the gait difficulty rather than lumbar
stenosis. Although the MRI scan of the lumbar spine shows multilevel spinal stenosis
that is mild to moderate, it does not clearly explain the patient's signs and symptoms.
Electromyography and nerve conduction velocity studies of the lower extremities are
unlikely to add significantly to the diagnosis. Epidural steroid injections are not
indicated. Lumbar decompression is unlikely to help the patient because the source of
the patient's problem does not originate in the lumbar spine. MRI of the brain could
be considered as a secondary imaging study if the cervical and thoracic MRI scans fail
to identify an obvious cause for gait instability.

(SAE10PE.43) Which of the following is a characteristic of odontoid fractures in


children? Review Topic

1 Usually occur in the body of C2


2 Are reduced by gentle cervical flexion
3 Frequently progress to nonunion
4 Almost always occur at the basilar synchondrosis
5 Are commonly associated with neurologic injury

PREFERRED RESPONSE 4

Fracture of the odontoid process in children is usually caused by a fall, motor vehicle
accident, or minor trauma, and almost always occurs through the synchondrosis at the
base of the dens. Neurologic deficits are rare in isolated odontoid fractures in
children. Closed reduction by neck extension and immobilization using a cast, a
brace, or halo traction for 6 to 8 weeks is usually sufficient to allow the fracture to
heal.

(SBQ12SP.56) Figure A is an illustration of a skull with three colored zones. Placing


a pin for a halo vest orthosis in the red zone places what structures at risk? Review
Topic

1 Supratrochlear nerve and optic nerve


2 Supraorbital nerve and optic nerve
3 Supratrochlear nerve and cranial nerve VI
4 Supraorbital nerve and cranial nerve VI
5 Supraorbital nerve and supratrochlear nerve

PREFERRED RESPONSE 5

Placing a pin for a halo vest orthosis over the medial third of the orbit, as labeled in
red in Figure A, places the supraorbital nerve and supratrochlear nerve at risk of
iatrogenic injury.

Proper placement of anterior halo orthosis pins is approximately 1 cm above the


superior orbital rim (below the equator of the skull) and over the LATERAL third of
the orbit. Staying lateral is critical to avoid the supraorbital and supratrochlear nerves
which are located more medially. Injury to the supratrochlear nerve may present with
loss of sensation to the medial eyelid, conjunctiva, and some of the forehead. Injury to
the supraorbital nerve may present with loss of sensation over the upper eyelid,
forehead, scalp, as well as weakness of the upper eyelid.

Bono presents a review article on the use of halo fixators for cervical trauma. He
recommends placing anterior pins within a region approximately 1 cm above the eye
brow (which should be at or below the equator), and over the lateral third of the orbit.
He provides further detail on appropriate indications and techniques.

Garfin et al. present a retrospective review of 179 patients treated with a halo fixator
to identify common complications. Complications included pin loosening (36%), pin-
site infection (20%), pressure sores under the vest (11%), dysphagia (2%), and nerve
injury (2%). Of note, injury to the supraorbital or supratrochlear nerve occured in 3
patients, all of whom had pins placed over the medial third of the orbit. They
recommend anterior pin placement of the lateral third of the orbit to avoid this
complication.

Illustration A is a drawing to show the safe zone for anterior pin placement relative to
the temporalis muscle, supraorbital nerve, and supratrochlear nerve. Illustration B
shows a surgical dissection of the Supraorbital nerve and Supratrochlear nerve.

Incorrect answers:
Answers 1 and 2: The optic nerve is not known to be at risk from a medially placed
pin.
Answers 3, 4: Cranial nerve VI is not known to be at risk from a medially placed pin

(SBQ12SP.23) Which of the following statements about spinal anatomy is true?


Review Topic

1 The ratio of cortical to cancellous bone in the pedicle is higher in the thoracic spine
than the sacrum
2 The pedicle screw trajectory at L3 is angled more lateral than at L1
3 The diameter of the S1 pedicle is smaller than the L1 pedicle
4 The S1 nerve root sits on the sacral ala and is at risk with percutaneous sacral iliac
fixation
5 The starting point for S1 pedicle screw is superior and medial to the inferior tip of
the inferior articlular process of L5

PREFERRED RESPONSE 1

The ratio of cortical to cancellous bone in the pedicle is higher in the thoracic spine
than the sacrum.
Knowledge of characteristic regional spinal anatomy is essential for safe pedicle
screw placement. Fixation is best in cortical bone. The sacrum has the lowest ratio of
cortical to cancellous bone in the spine. Weight-bearing potential and availability of
bone for hardware fixation is related to the density of that bone. Cortical bone has
higher density than cancellous, and therefore higher ratios of cortical to cancellous
bone are optimal for weight-bearing and fixation. The pedicles of the thoracic spine
have the highest ratio of cortical to cancellous bone. Local bone density must be
carefully considered when planning spinal fixation in osteoporotic bone. In the
lumbosacral spine, pedicle diameter and medial angulation increase moving proximal
to distal.

Wray et al. performed a biomechanical study in which they compared a medialized,


caudal-to-cephalad ("cortical") screw trajectory to a more traditional pedicle screw
trajectory. They found that the cortical trajectory engaged higher quality (denser)
bone than a traditional pedicle screw trajectory.

Illustration A shows the differences between the cortical and traditional trajectories
described by Wray et al. Illustration B shows their micro-CT analysis, which again
shows the trajectories, as well as the drilled out bone volume and an example of
lateral wall breach in the traditional trajectory (although breach rate was not different
between the two trajectories in their study). Illustration A shows the starting point of
the S1 pedical screw.

Incorrect answers:
Answer 2. The pedicle screw trajectory at L3 is angled more medial than at L1.
Answer 3. The S1 pedicle is larger in diameter than the L1 pedicle.
Answer 4. The L5 nerve root sits on the sacral ala and is at risk with percutaneous
sacral iliac fixation
Answer 5. The starting point for S1 pedicle screw is inferior (caudal) and lateral to the
inferior tip of the inferior articlular process of L5.

(SAE09SN.10) A 46-year-old woman who was involved in a motor vehicle accident


reports a 4-month history of right-sided lower back pain and pain radiating into the
right thigh. The patient underwent an extensive 3-month course of physical therapy
and now is dependent on narcotic medication for pain control. Epidural injection
therapy has failed to improve her symptoms. Examination is significant for weakness
of hip flexion in the seated position and for decreased sensation to light touch in the
medial anterior thigh region. Straight leg raise is negative, but the femoral stretch test
reproduces anterior thigh pain. A CT myelogram image, at L3-L4, is shown in Figure
3. What is the most appropriate management at this time? Review Topic
1 Repeat epidural steroid injections
2 Wide lumbar laminectomy
3 Microdiskectomy from either a midline approach or far lateral approach
4 Referral to pain management
5 Minimally invasive posterior lumbar interbody fusion

PREFERRED RESPONSE 3

The CT scan reveals a right-sided lateral disk protrusion at L3-4 that has been
symptomatic for more than 4 months despite appropriate nonsurgical management.
Relative surgical indications include persistent radiculopathy despite an adequate trial
of nonsurgical management, recurrent episodes of sciatica, persistent motor deficit
with tension signs and pain, and pseudoclaudication caused by underlying stenosis.
Whereas studies have shown improvement in patients with sciatica from a lumbar
disk herniation treated either nonsurgically or surgically, those undergoing surgical
treatment had an overall greater improvement of symptoms.

(SAE12SN.33) Figures 33a and 33b show the standing posteroanterior and lateral
radiographs of a 59-year-old woman with adult idiopathic scoliosis. She underwent a
prior decompressive laminectomy and fusion at L4-S1 to address lumbar stenosis. She
now reports progressive lower back pain and a feeling of being shifted to the right. If
surgical intervention is considered, what is the most important goal in improving her
health-related quality of life (HRQL) outcomes? Review Topic
1 Correction of the thoracolumbar curve
2 Sagittal balance
3 Coronal balance
4 Correction of the thoracic curve
5 Shoulder balance

PREFERRED RESPONSE 2

Sagittal balance is the most reliable predictor of clinical symptoms and HRQL
outcomes on the SRS 29, SF-12, and Oswestry Disability Index. Coronal balance,
shoulder balance, curve magnitude, and degree of curve correction are less critical in
determining clinical symptoms and outcomes.

(SBQ12SP.24) A 39-year-old man presents to clinic with a 3-week history of low


back pain that radiates to the right lower extremity. On examination, he has mildly
decreased sensation over the dorsum of the foot and positive straight leg raise on the
right side. MRI images are shown in Figure A and B. Which of the following is true
regarding this patient's condition? Review Topic

1 Nonoperative management with NSAIDS and physical therapy is effective for 50%
of patients
2 Surgical treatment is indicated in patients with diminished sensation
3 Surgical treatment is equivalent to nonoperative management in terms of pain and
function
4 Good surgical outcome is associated with mainly back complaints
5 Size of disc herniations typically decrease over time without surgical intervention

PREFERRED RESPONSE 5

The patient is presenting with a lumbar disc herniation at the L4-L5 level. The size of
disc herniations decrease in most patients over time without surgical intervention.

Lumbar disc herniations are the result of recurrent torsional strain, which leads to
small tears of the annulus fibrosus, ultimately allowing herniation of the nucleus
pulposis. First line treatment consists of NSAIDS, muscle relaxants and physical
therapy and is effective in 90% of patients. Second line treatment typically involves
epidural and selective nerve root corticosteroid injections. Microdiscectomy is
reserved for patients with more than 6 weeks of disabling pain that has failed
nonoperative management, progressive weakness, or cauda equina syndrome.

In the Spine Patient Outcomes Research Trial (SPORT), Weinstein et al. investigated
patient outcomes and satisfaction after operative and nonoperative management of
lumbar disc herniations. While the randomized arm of the study did show statistically
significant differences in the intent-to-treat analysis due to significant crossover of
patients, the observational cohort revealed a significant improvement in pain,
function, and disability for patients treated with surgery versus nonoperative
measures.

Benson et al. looked at the natural history of massive herniated discs in 37 patients
with 7-year follow up. They found a more than 60% reduction in disc size over this
time period. Reduction in disc size did not correlate with clinical improvement.

Figure A and B are sagittal and axial T2 MRI images, respectively, showing a right
sided lumbar disc herniation at the L4-L5 level.

Incorrect Answers:
Answer 1: Nonoperative management is effective in 90% of patients.
Answer 2: Surgery is reserved for patients with more than 6 weeks of disabling paint
that has failed nonoperative management, progressive weakness, or cauda equina
syndrome.
Answer 3: Surgery leads to greater improvement in pain and function than
nonoperative management.
Answer 4: Good surgical outcomes are associated with mainly leg complaints,
positive straight leg raise, and weakness correlating with nerve root impingement on
MRI.
(SAE09TR.74) A 40-year-old man sustains a fracture-dislocation of C4-5.
Examination reveals no motor or sensory function below the C5 level. All extremities
are areflexic. The bulbocavernosus reflex is absent. The prognosis for this patient’s
neurologic recovery can be best determined by Review Topic

1 myelography with CT.


2 spinal cord-evoked potentials.
3 repeat physical examinations.
4 MRI.
5 electromyography and nerve conduction velocity studies.

PREFERRED RESPONSE 3

The patient has spinal shock. Steroid administration and MRI are appropriate
therapeutic and diagnostic procedures. Myelography with CT is of little value unless
there is an unusual skeletal variant. Spinal cord-evoked potentials have no value. The
best method to determine the patient’s neurologic recovery is repeated physical
examinations over the first 48 to 72 hours.

(SAE12SN.83) Treatment consisting of halo vest immobilization is most likely to fail


with which of the following cervical injuries? Review Topic

1 C1 lateral mass fracture


2 C2 pars fracture
3 C4 burst fracture
4 C5 burst fracture
5 C6-C7 facet fracture-dislocation

PREFERRED RESPONSE 5

Facet joint fracture or dislocation is associated with an increased risk of loss of


alignment with halo vest immobilization. The recently published study by van
Middendorp and associates confirms the findings of prior studies that facet fracture-
subluxations or dislocations are difficult to immobilize with a halo vest due to a
limited ability to maintain reduction and alignment. C2 pars fractures, burst fractures,
and C1 lateral mass fractures can be managed with halo vest immobilization.
(SAE09SN.13) When compared to smokers who do not quit, an improvement in the
rate of lumbar fusion is seen in patients who cease smoking for at least how many
months postoperatively? Review Topic

1 1 month
2 2 months
3 4 months
4 6 months
5 12 months

PREFERRED RESPONSE 4

The effects of cigarette smoking and smoking cessation on spinal fusion have been
studied extensively. Although permanent smoking cessation is ideal, significant
improvements in fusion rates are seen in patients who avoid smoking for greater than
6 months postoperatively.

(SAE10BS.94) Which of the following levels of the spine is most frequently involved
in ossification of the posterior longitudinal ligament (OPLL)? Review Topic

1 1-2
2 C4-6
3 T4-T8
4 T10-L1
5 L2-L5

PREFERRED RESPONSE 2

OPLL is a cause of spinal stenosis and myelopathy most frequently seen in the Asian
population but affects other populations as well. Men are affected more often than
women and the peak age at onset is in the sixth decade of life. It is often associated
with other conditions such as diffuse idiopathic skeletal hyperostosis (DISH) and
ankylosing spondylitis. The levels most frequently involved are C4, C5, and C6. More
than 95% of the ossification is localized in the cervical spine.
(SAE09SN.12) Osteoporotic vertebral compression fractures are associated with
Review Topic

1 neurologic deterioration in 33% of patients.


2 osteomalacia in 50% of patients.
3 a further fracture risk rate of 20%.
4 chronic pain in 75% of patients.
5 a 2-year mortality rate that is less than that associated with hip fractures.

PREFERRED RESPONSE 3

Osteoporotic vertebral compression fractures are associated with neurologic


complications in less than 1% of patients. After the initial fracture however, patients
have a 20% risk of further fractures. The mortality rate of patients with vertebral
fractures exceeds that of patients with hip fractures when they are followed beyond 6
months.

(SAE09TR.87) A 40-year-old woman sustains a flexion injury to her neck. Physical


examination is normal. A lateral radiograph of the cervical spine is shown in Figure
57a. MRI scans of the cervical spine are shown in Figures 57b and 57c. Treatment
should include Review Topic

1 skeletal traction and reduction, followed by a halo jacket and nonsurgical


stabilization.
2 skeletal traction, closed reduction, and posterior fixation/fusion.
3 skeletal traction, anterior decompression and fusion, followed by posterior
stabilization and fusion.
4 skeletal traction and surgical posterior fusion, followed by anterior decompression
and fusion.
5 general anesthesia, closed reduction, and a halo jacket for 3 to 4 months until stable.
PREFERRED RESPONSE 2

This is a classic bilateral facet dislocation. When there is no evidence of a disk


herniation, treatment should include careful skeletal traction, closed reduction, and
posterior fusion. There is no role for anterior procedures. These fractures are unstable
and require surgical intervention.

(SBQ12SP.18) An otherwise healthy 36-year-old male is found to have an L4-5


posterolateral disk herniation on MRI. Which of the following would most likely be
found on clinical examination? Review Topic

1 Weakness in hip flexion and adduction


2 Decreased sensation along the plantar aspect of the foot
3 Weakness in foot inversion
4 Decreased sensation along the anterior knee
5 Weakness in foot plantar flexion

PREFERRED RESPONSE 3

An L4-5 posterolateral disk herniation would affect the L5 nerve root. Foot inversion
is controlled by the tibialis posterior, which is innervated by the L5 nerve root.

Posterolateral (paracentral) disk herniations are the most common type of lumbar disk
herniation, and affect the descending/lower nerve root. This is in contrast to foraminal
(far lateral) disk herniations, which affect the exiting/upper nerve root. Characteristic
motor findings of an L5 radiculopathy include weakness to great toe extension (EHL),
ankle dorsiflexion (tibialis anterior - can also be caused by L4 radiculopathy), foot
inversion (tibialis posterior), and hip abduction (gluteus medius).

Overdevest et al. investigated 150 patients with lumbar disc herniations who
presented with either a moderate or severe motor deficit, and looked at outcomes
depending on whether patients were treated with early surgery or prolonged
conservative treatment. They found at 1 year complete recovery of motor deficit was
found in 81% of patients allocated to early surgery and in 80% of patients allocated to
prolonged conservative treatment. They conclude that while patients treated with
early surgery had faster recovery of the motor deficit compared with prolonged
conservative treatment, the difference was no longer significant at 1 year.

Radcliff et al. looked at the impact of epidural steroid injections in patients with
Lumbar Disc Herniations in the SPORT study. They found while epidural steroid
injections did not make a difference on long term outcomes, patients who had the
epidural steroid injection where more likely to change from their randomized assigned
surgical treatment group to the nonoperative treatment group.
Illustration A shows the two differences between how a paracentral and far lateral
disc herniation affect the descending and exiting nerve root.
Illustration B shows an axial MRI of a paracentral disc herniation.
Illustration C shows an axial MRI of a far lateral disc herniation.

Incorrect Answers:
Answer 1: Hip flexion is controlled by the iliopsoas while hip adduction is controlled
by the adductors, both of which are innervated by L2,3.
Answer 2: Plantar foot sensation is provided by S2.
Answer 4: Anterior knee sensation is provided by L4.
Answer 5: Foot plantar flexion is controlled by the gastroc-soleus complex, which is
innervated by S1.

(SAE12SN.23) Which of the following areas of the vertebral segment has the highest
ratio of cortical to cancellous bone? Review Topic

1 Thoracic vertebral bodies


2 Lumbar vertebral bodies
3 Sacrum
4 Pedicles of the lower lumbar spine
5 Pedicles of the thoracic spine

PREFERRED RESPONSE 5

The weight-bearing potential of bone is influenced by the ratio of cortical to


cancellous bone. The area of the spinal anatomy that has the highest ratio is the
pedicles of the thoracic spine. This is followed by the lumbar pedicles. The vertebral
bodies have a lower ratio than the pedicles, with the sacrum having the very lowest
ratio.

(SAE12SN.94) The findings in Brown-Sequard syndrome include loss of which of the


following? Review Topic

1 Greater loss of upper extremity motor function than lower extremity function
2 Ipsilateral motor function and ipsilateral pain and temperature sensation
3 Ipsilateral motor function and contralateral pain and temperature sensation
4 Contralateral motor function and ipsilateral pain and temperature sensation
5 Lower of extremity proprioception and balance

PREFERRED RESPONSE 3

Brown-Séquard syndrome is most commonly seen after penetrating injuries to the


spinal cord and results in ipsilateral loss of motor function and contralateral loss of
pain and temperature sensation. Patients with central cord syndrome have greater
weakness in the upper extremities than in the lower extremities. Loss of
proprioception is typically seen in patients with posterior cord syndrome.

(SAE09SN.34) A 32-year-old motorcycle rider is involved in a motor vehicle accident


and radiographs show a burst fracture at L2 with 20 degrees of kyphosis. The
neurologic examination is consistent with unilateral motor and sensory involvement
of the L5, S1, S2, S3, and S4 nerve roots. He has no other injuries. CT demonstrates
20% anterior canal compromise with displaced laminar fractures at the level of injury.
What is the best option for management of this patient? Review Topic

1 Bed rest for 6 weeks, followed by mobilization in a thoracolumbosacral orthosis


until the fracture has healed
2 Anterior corpectomy with strut grafting and placement of an anterior plate spanning
L1 to L3
3 Anterior corpectomy with strut grafting, followed by posterior spinal fusion and
instrumentation
4 Posterior spinal fusion and instrumentation from T11 to L4
5 L2 laminectomy and posterior spinal fusion and instrumentation from T11 to L4

PREFERRED RESPONSE 5

The patient has a burst fracture with probable unilateral entrapment of the cauda
equina within the elements of the fractured lamina. A dural tear is likely in this
scenario as well. It is recommended that this type of burst fracture be treated
surgically with laminectomy, freeing of the entrapped nerve roots, and dural repair
followed by stabilization of the fracture by either a posterior or combined approach.
The degree of kyphosis and the extent of anterior canal compromise does not warrant
corpectomy in this patient. Therefore, after completing the laminectomy and dural
repair, posterior fusion and instrumentation should be sufficient to stabilize the
fracture.
(OBQ13.138) A 12-year-old girl who is 3 months postmenarchal undergoes full-time
brace treatment for scoliosis. The posteroanterior radiograph (Figure A ) taken at that
time reveals a right thoracic curve measures 28 degrees, and the left lumbar curve
measures 23 degrees. At age 15, after 3 years of bracing, a repeat posteroanterior
radiograph is obtained, now revealing a right thoracic curve measuring 11 degrees and
the left lumbar curve measuring 19 degree, and Risser 4. Which statement best
represents the indicated course of action in this patient? Review Topic

1 Discontinuation of bracing as she has reached skeletal maturity.


2 Continue full-time bracing until skeletal maturity.
3 Continue nocturnal bracing until skeletal maturity.
4 Posterior spinal fusion.
5 MRI of the cervical, thoracic and lumbar spine.

PREFERRED RESPONSE 1

This patient has adolescent idiopathic scoliosis (AIS) and has reached skeletal
maturity. Bracing was successful and discontinuation of bracing is appropriate.

Curves <25° can be treated with observation, while flexible curves from 25° to 45° in
skeletally immature patients (Risser 0, 1, 2) should be treated with bracing. Bracing
success is most commonly defined as <5° curve progression and failure is 6° or more
curve progression at orthotic discontinuation (skeletal maturity), absolute progression
to >45° either before or at skeletal maturity, or discontinuation in favor of surgery.
Skeletal maturity is defined Risser sign 4, <1cm change in height over 2 visits 6
months apart, 2 years postmenarchal.

Richards et al. attempted to define parameters for future AIS bracing studies.
Outcome measures should include patients with (1) <5° curve progression vs >6°
progression at maturity, (2) curves exceeding 45° at maturity, or those who have had
surgery recommendation/undergone.

Negrini et al. performed a Cochrane systematic review. Basing conclusions on 2


studies, they found that (1) a brace treated curve progression (74% success) better
than observation (34% success) and electrical stimulation (33% success), and (2) a
rigid brace is more successful than an elastic one (SpineCor) at curbing curve
progression.

Figures A is a standing PA radiograph showing.

Incorrect Answers:
Answers 2 and 3: Brace cessation is appropriate as she has reached skeletal maturity,
and bracing was successful. Only observation is necessary at this stage.
Answer 4: Surgery is only indicated if bracing was unsuccessful.
Answer 5: MRI is indicated if red flags are present. These include atypical curve (left
thoracic, short angular curve, apical kyphosis), rapid progression, hyperkyphosis,
structural abnormalities, neurologic symptoms or pain, foot deformities and
asymmetric abdominal reflexes.

(SAE09SN.19) A 68-year-old woman undergoes a complicated four-level anterior


cervical diskectomy and fusion at C3-7 with iliac crest bone graft and instrumentation
for multilevel cervical stenosis. Surgical time was approximately 6 hours and
estimated blood loss was 800 mL. Neuromonitoring was stable throughout the
procedure. The patient’s history is significant for smoking. The most immediate
appropriate postoperative management for this patient should include Review Topic

1 normal postoperative orders with frequent neurologic evaluations for the first 24
hours.
2 administration of IV steroids and placement of a soft cervical collar for 24 hours.
3 placement of both deep and superficial surgical drains prior to wound closure.
4 administration of IV mannitol and placement of a soft collar.
5 maintaining intubation for up to 24 to 48 hours.

PREFERRED RESPONSE 5

Airway complications after anterior cervical surgery can be a catastrophic event


necessitating emergent intubation for airway protection. Multilevel surgeries requiring
long intubation and prolonged soft-tissue retraction as well as preexisting
comorbidities may predispose a patient to postoperative airway complications. Sagi
and associates reported that surgical times greater than 5 hours, blood loss greater
than 300 mL, and multilevel surgery at or above C3-4 are risk factors for airway
complications. In surgical procedures with the aforementioned factors, serious
consideration should be given to elective intubation for 1 to 3 days to avoid urgent
reintubation.
(SAE10PE.83) A 3-year-old child sustains a T2/T3 fracture-dislocation with complete
paraplegia secondary to a car accident in which the child was an unrestrained
passenger. What is the likelihood that this child will develop subsequent spinal
deformity in the future? Review Topic

1 0% if bracing is used
2 25%
3 50%
4 75%
5 90% or greater

PREFERRED RESPONSE 5

More than than 90% of preadolescent children who sustain a significant spinal cord
injury subsequently develop scoliosis. Conversely, progressive paralytic spinal
deformity is uncommon in the postadolescent patient. Bracing has not been shown to
be effective in the prevention of scoliosis in the preadolescent patient with spinal cord
injury.

(SAE12SN.86) Which of the following statements best describes the instantaneous


axis of rotation (IAR) for the functional spinal unit? Review Topic

1 The IAR is calculated by movement around six different axes.


2 The IAR consists of potential movements or 12 degrees of freedom.
3 The IAR is fixed and consistent and is the axis about which each vertebral segment
rotates.
4 The IAR can be derived from AP and lateral radiographs.
5 Degenerative disease, fractures, and ligamentous injuries can affect the IAR.

PREFERRED RESPONSE 5

The instantaneous axis of rotation is the axis about which each vertebral segment
rotates, but is theoretical depending on how it is defined, and varies depending on
multiple factors. It is not a fixed point but can move depending on the position of the
spine, and it is affected by degenerative conditions, fractures, injuries, and other
anatomic changes of the spine. There are three axes of movement with 6 degrees of
freedom (rotation and translation movements about each axis).
(SAE13BS.1) A 70-year-old man is experiencing neck pain, progressive weakness,
and numbness in his arms and legs without bowel or bladder dysfunction or
dysphagia. Upon examination, he has atrophy of his upper extremities but normal
muscle bulk in his legs. Strength is diffusely 4/5 throughout. Cervical spine
radiographs show spondylosis. Electromyography (EMG) reveals fibrillations with
increased amplitude in the extensor carpi radialis and pronator teres. Nerve
conduction studies demonstrate slowing conduction diffusely in the sural, peroneal,
and ulnar nerves, and severe slowing in the median nerve. Testing of the tongue and
thoracic paraspinal muscles does not show fibrillations or positive short waves. What
is the most likely diagnosis? Review Topic

1 Amyotrophic lateral sclerosis


2 Cubital tunnel syndrome
3 Normal aging
4 Cervical radiculopathy

PREFERRED RESPONSE 4

Physical examination and presentation is consistent with possible cervical


radiculopathy vs motor neuron disease. EMG findings are most consistent with
cervical radiculopathy. There is denervation (fasciculations and positive short waves)
of the C6 innervated muscles consistent with radiculopathy. However, evaluation of
other body regions does not show evidence of denervation (tongue, thoracic
paraspinal muscles). Fasciculations in the hand muscles were not widespread. Nerve
conduction suggests the presence of a peripheral polyneuropathy with possible
superimposed median neuropathy. Amyotrophic lateral sclerosis is a motor neuron
disease that affects both upper and lower neurons. Presentation includes rapid
progression of weakness, muscle atrophy, fasciculations, spasticity, dysarthria,
dysphagia, and respiratory compromise.

(SAE09SN.50) Kyphosis from a vertebral osteoporotic compression fracture often


results in progressive kyphosis due to Review Topic

1 progressive increase in lumbar lordosis.


2 load transfer to the superior adjacent vertebra.
3 normalization of load transfer with working kyphosis.
4 reduced strain at the occipito-cervical junction.
5 reduced strain at the apex of the deformity.
PREFERRED RESPONSE 2

Kayanja and associates, in a number of biomechanical studies, showed that in a


kyphotic spine the strain is located at the apex of the deformity, the force is
transmitted to the superior adjacent vertebrae, and that realignment and cement
augmentation effectively normalize the load transfer.

(OBQ14.55) A 25-year-old unrestrained passenger presents with neck pain and


extremity weakness after a motor vehicle accident. The patient is awake, oriented, and
cooperative on exam. Motor exam shows partial weakness in the upper extremities
and no spontaneous motion of lower extremities. Reflex exam shows his
bulbocavernosus reflex is intact. A radiograph of the cervical spine is shown in Figure
A. A CT scan of the chest, abdomen, and pelvis shows no evidence of internal
bleeding. He is given adequate fluid resuscitation in the emergency room. Which
pairing of vital signs in this patient would be most consistent with neurogenic shock?
Review Topic

1 HR 120, BP 65/40
2 HR 55, BP 65/40
3 HR 120, BP 120/80
4 HR 55, BP 120/80
5 HR 100, BP 160/80

PREFERRED RESPONSE 2

Neurogenic shock (NS) classically presents in the trauma setting with hypotension
and an associated relative bradycardia.

When evaluating a patient with an acute spinal cord injury and hypotension, it is
important to differentiate between hypovolemic shock (HS), neurogenic shock (NS)
and spinal shock (SS). Hypovolemic shock is identified as hypotension and
tachycardia, and is caused by fluid loss depleting intravascular volume and pressure.
Neurogenic shock (NS) and spinal shock (SS) both present with hypotension and
bradycardia for a transient period immediately after the injury (average 24-72 hours).
The primary distinguishing factor is that spinal shock is accompanied by flaccid
paralysis below the injury level (similar to complete spinal cord injury) including an
absent bulbocavernosus reflex indicating complete disruption of spinal cord
signalling. Spinal shock is caused by peripheral neurons becoming temporarily
unresponsive to brain stimuli.

Neurogenic shock is caused by disruption of autonomic pathways and responses,


leading to loss of sympathetic tone and decreased systemic vascular resistance (vessel
dilation). Therefore, with the central and peripheral nervous system still functioning,
some motor and reflex activity can be seen depending on the underlying level of
spinal cord injury, including an intact bulbocavernosus reflex proving the patient is
not in spinal shock. The diagnosis of NS cannot be made until spinal shock has been
ruled out.

Mallek et al performed a retrospective chart review of trauma patients with spinal


cord injury. They found overall an 8.8% rate of NS but a much higher rate (24.4%) in
patients with cervical spine injury. They do note however that the stereotypic
bradycardia with hypotension is somewhat rare.

Furlan et al review the pathophysiology leading to cardiovascular complications from


supraspinal dysregulation of the autonomic nervous system. They provide evidence-
based guidelines for hemodynamic monitoring and recommendations to keep
MAP>85. They emphasize the risk of DVT and need for low-molecular weight
heparin prophylaxis.

Figure A shows a lateral radiograph of a cervical spine with obvious C2-3 injury.

Illustration A is a schematic diagram from Furlan et al depicting cardiovascular


control mechanisms by the central nervous system.

Incorrect answers:
Answer 1: HR 120, BP 65/40 is consistent with Stage III/IV hypovolemic shock
(tachycardia, hypotension)
Answer 3 HR 120, BP 120/80 is consistent with Stage I hypovolemic shock
(tachycardia, normotensive)
Answer 4: The BP and HR are normal
Answer 5: HR 100, BP 160/80 (hypertension) can be a chronic condition or a
response to pain

(SAE09SN.7) A 42-year-old woman underwent an instrumented posterior spinal


fusion at L3-S1 with transforaminal lumbar interbody fusion. She had an excellent
clinical result with complete resolution of leg pain. Three months later she now
reports increasing back pain and weakness in her legs. Examination reveals weakness
in the quadriceps and tibialis anterior. Radiographs show no interval changes in the
position of the hardware. MRI scans are shown in Figures 2a through 2c. What is the
next most appropriate step in management? Review Topic

1 Observation
2 Oral antibiotics only
3 IV antibiotics only
4 Irrigation and debridement of the surgical site
5 Irrigation and debridement of the surgical site with hardware removal

PREFERRED RESPONSE 4

The MRI scans reveal a postoperative infection. Observation and antibiotics are not
appropriate choices. There is a large fluid collection and this requires decompression
because the patient has neurologic changes. There is considerable debate regarding
the removal of hardware. Many contend that biofilm on the implants can harbor the
infection. However, these complications usually can be treated with serial irrigations,
debridements, and IV antibiotics. The incidence of infection has been widely studied
with varying rates in fusions with instrumentation. Rates appear to be increased with
instrumentation, yet these infections usually can be managed without hardware
removal.

(SAE12SN.96) A 75-year-old woman who sustained a fall now reports neck pain and
upper extremity weakness. Examination reveals 4 of 5 strength in the upper
extremities and 5 of 5 strength in the lower extremities. Radiographs show multilevel
degenerative disk disease. An MRI scan is shown in Figure 96. Her clinical
presentation is most compatible with which of the following? Review Topic
1 Brachial plexus injury
2 Anterior cord syndrome
3 Posterior cord syndrome
4 Central cord syndrome
5 Brown-Séquard syndrome

PREFERRED RESPONSE 4

The MRI scan shows advanced multilevel degenerative changes and moderate to
severe stenosis at C3-C4 and C4-C5 with associated cord signal change. The patient
has greater weakness in the upper extremities than in the lower extremities. This
pattern is most compatible with central cord syndrome. Patients with brachial plexus
injury will have unilateral weakness. Patients with anterior cord syndrome will have
greater weakness in the legs than in the arms, and those with Brown-Séquard
syndrome will have ipsilateral motor deficits and contralateral pain and temperature
deficits.

(SAE10PE.94) A 7-year-old boy is seen for follow-up for a scoliotic deformity. His
parents are concerned because his deformity seems to have increased. He has no pain
and is neurologically intact. A radiograph is shown in Figure 94, and measurement of
his curve reveals that it has increased 10 degrees. What is the most appropriate
recommendation for this patient at this time? Review Topic
1 Observation
2 Bracing
3 A "growing rod"
4 Distraction instrumentation and posterior arthrodesis
5 Hemivertebra excision and limited fusion

PREFERRED RESPONSE 5

Nakamura and associates have reported good results in patients with resection for
hemivertebra-related congenital scoliosis who have a progression of their deformity.
Because of the progression, observation is not appropriate for this patient's deformity.
Bracing has not been shown to alter the progression of congenital scoliosis. The
"growing rod" technique is also not effective in preventing progression related to
hemivertebra. Distraction instrumentation carries an increased risk of neurologic
complications in children with congenital spine deformities. Progression after
posterior arthrodesis alone can occur through the so-called "crankshaft phenomenon."

(SAE13PE.82) Figures 82a through 82c show the radiograph and 3-dimensional (3-D)
CT scans of a 2-year-old boy whose scoliosis has progressed 15 degrees during the
past year. The child is clinically healthy. He has been walking since 11 months of age.
An MRI scan of the entire spine revealed no other anomalies. What additional study is
indicated? Review Topic
1 Renal ultrasound
2 Blood cultures
3 Flexion-extension cervical spine radiographs
4 Platelet count

PREFERRED RESPONSE 1

Renal anomalies are found in as many as one-third of patients with congenital


scoliosis, so a renal ultrasound should be obtained. There may be other anomalies,
including cardiac. There are no other anomalies on MRI, so flexion-extension cervical
spine radiographs are not indicated. There is no associated marrow or platelet problem
with hemivertebra. There is no indication for blood cultures because this is a
noninfection disorder. The radiographs and 3-D CT scans show a hemivertebra
scoliosis already beyond 45 degrees. Resection of the hemivertebra with stabilization
is the indicated treatment. The scoliosis will get worse with observation and bracing.
Fusion posteriorly can only minimally correct and not stop progression of the
scoliosis.

(SAE09SN.24) When performing a long fusion to the sacrum in an osteopenic patient


in whom optimal sagittal balance is restored, which of the following is a benefit of
extending the distal fixation to the pelvis, rather than the sacrum alone? Review Topic

1 Decreased risk of sacral fractures


2 Decreased risk of proximal functional kyphosis
3 Easier contouring of the instrumentation
4 Reduced risk of late pubic ramus fractures
5 Improved coronal plane correction
PREFERRED RESPONSE 1

In osteopenic individuals, even those with excellent obtained or maintained balance,


long instrumented fusions to the sacrum impart a high degree of strain, and the
sacrum may fail in a transverse fracture or fracture-dislocation pattern. The risk of
proximal functional kyphosis is unrelated to distal fixation as are coronal plane
correction and rod contouring. Pubic ramus fractures have been shown to be
associated with both fixation to the sacrum alone as well as to the ilium.

(SAE11OS.135) During spinal deformity surgery, which of the following is the most
specific early indicator of an intraoperative injury to the spinal cord? Review Topic

1 Somatosensory-evoked potentials
2 Transcranial motor-evoked potential monitoring
3 Transcutaneous electroencephalogram neuromonitoring
4 Stimulus-evoked transpedicular electromyography (EMG)
5 Brainstem auditory-evoked responses (BAERs)

PREFERRED RESPONSE 2

Transcranial motor-evoked potentials provide the most specific early indicator of an


intraoperative spinal cord injury. Somatosensory-evoked potentials are routinely used
but do not have the sensitivity and specificity of motor-evoked potentials. EMG
evaluations are routinely used for root evaluation following pedicle screw placement.
BAERs are typically used in monitoring brain surgery.

(OBQ06.66) A 3-year-old girl developed torticollis eight months ago after a severe
respiratory tract infection. A initial trial of halter traction was attempted without
success. A trial of halo traction was then performed for 3 weeks and then a dynamic
computed tomographic (CT) was obtained and shown in Figure A. Panel (a) shows an
axial image with maximal rotation to the left. Panel (b) shows an axial image with
maximal rotation to the right. What is the most appropriate next step in management?
Review Topic
1 No further treatment
2 Closed reduction under conscious sedation
3 Closed reduction under general anesthesia in the operating room with neurologic
monitoring
4 Occipitocervical fusion
5 Posterior atlantoaxial fusion

PREFERRED RESPONSE 5

The clinical presentation is consistent with chronic torticollis caused by Atlantoaxial


rotatory displacement (AARD). Because both halter traction and halo traction were
attempted and failed, the next most appropriate next step in management is posterior
atlantoaxial fusion.

Common causes of Atlantoaxial rotatory displacement (AARD) include infection,


trauma, and recent neck surgery. Diagnosis is challenging and is best confirmed with
dynamic CT (CT with the head turned maximally to either side and at neutral). If the
symptoms are acute (less than 7 days) then initial treatment with a soft collar and anti-
inflammatory medications is indicated. If the condition has been present for more than
a week, more aggressive treatment with halter traction (present 1 week to 1 month) or
halo traction (present for 1-3 months) is indicated. If nonoperative modalities fail, the
condition has been present for > 3 months, or the patient has neurologic deficits, then
posterior C1-C2 fusion is indicated.

Copley et al discuss the evaluation and treatment of various congenital and traumatic
conditions of the pediatric cervical spine. They report that the underlying mechanism
of Atlantoaxial rotatory displacement (AARD) is inflammation and spasm which can
be caused by infection, prior surgery, trauma, and rheumatoid arthritis.

Subach et al reviewed at 20 children with atlantoaxial rotatory subluxation. They


found that of the 20 patients treated overall, conservative management failed in 6
(30%), and they required posterior fusion because of recurrence of the atlantoaxial
rotatory subluxation or unsuccessful reduction. The major factor predicting the failure
of conservative management was the duration of subluxation before initial reduction.
Patients with long-standing subluxation were more likely to experience recurrence
and require surgery.

Figure A shows an asymmetric placed odontoid within the ring of C1. There is an
increased distance from the odontoid to the right arch of C1 which is fixed and
minimally changes with maximal rotation to the left. This radiographic finding is
indicative of fixed subluxation. Illustration A further demonstrates this.
Incorrect Answers:
Answer 1: The patient remains subluxed. Further treatment is required.
Answer 2: Closed reduction under conscious sedation is not a treatment option for
AARD.
Answer 3: Closed reduction under general anesthesia in the operating room with
neurologic monitoring is not a treatment option for AARD.
Answer 4: Occipitocervical fusion should be avoided due to the associated loss of
motion in a child and is not required in AARD

(SBQ12SP.1) A 65-year-old female with a history of breast cancer presents with


bilateral buttock and leg pain that is worse with walking and improves with sitting. In
addition, she reports that she feels unsteady on her feet and requires holding the
railing when going up and down stairs. On physical exam she is unable to complete a
tandem gait and has hip flexion weakness, ankle dorsiflexion weakness, and ankle
plantar flexion weakness. Her reflex exam shows 3+ bilateral patellar reflexes.
Radiographs and an MRI are shown in Figure A and B. What is the next most
appropriate step in management. Review Topic

1 Lumbar epidural injection


2 Physical therapy with core strengthening and anti-inflammatory medications as
needed
3 Lumbar decompression
4 Lumbar decompression and fusion
5 MRI of the cervical and thoracic spine

PREFERRED RESPONSE 5

The clinical scenario is consistent with a patient with symptoms of degenerative


spondylolisthesis AND symptoms of myelopathy. Myelopathy must be ruled out by
performing an MRI of the cervical and thoracic spine.

Tandem stenosis occurs in approximately 5 to 25% of patients. Because of the


stepwise progressive nature of myelopathy, treatment of myelopathy often takes
precedence over lumbar spinal stenosis.
Rhee et al. found that the sensitivity and specificity of specific physical exam findings
varies. Both the upward babinski reflex and the presence of clonus were found to be
very non-sensitive (13%). The most sensitive provacative test was found to be the
Hoffman sign (59%).

Salvi et al. reviewed the classic presentations for cervical myelopathy including
demographics, history, and physical exam findings (the inability to preform a tandem
gait, hyperreflexia, an abnormal babinksi and hoffman reflex, the inability to preform
rapid movements and bilateral muscle weakness). Additionally they identify other
potential causes for myelopathy, including multiple sclerosis, amyotrophic lateral
sclerosis, multifocal motor neuropathy, and Guillain-Barre´syndrome.

Maezawa et al. showed that gait analysis can identify a pattern in patients with
myelopathy. Patients with severe myelopathy have a characteristic gait with
hyperextension of the knee in the stance phase without plantar flexion of the ankle in
the swing phase. They also have decreased walking speed and stride length with a
prolonged stance phase.

Figure A and B show a classic degenerative spondylolisthesis.

Incorrect Answers:
Answers 1-4 are all incorrect because they fail to recognize the patient has
myelopathy. While the patient may benefit from physical therapy, an epidural steroid
injection, or surgery, the most important next step is to identify if the patients
myelopathic symptoms are coming from spinal cord compression, or from another
potential cause.

(SAE12SN.79) Which of the following would be associated with the spinal deformity
shown in Figures 79a and 79b? Review Topic

1 Improved gait
2 Deformity progression
3 Delayed satiety
4 No further risk of fracture
5 Improved lung function

PREFERRED RESPONSE 2

The images delineate progressive osteoporotic collapse. As outlined by Kado and


associates, Schlaich and associates, and Gold and associates, the progression of spinal
deformity and the functional consequences of vertebral compression fractures are
persistent even in those patients who are pain free. Vertebral compression fractures
are associated with deteriorating gait, early satiety, further future fracture risk, and
deteriorating lung function.

(SAE09SN.35) A patient who underwent a L4-L5 hemilaminotomy and partial


diskectomy for radiculopathy 8 weeks ago now reports increasing low back pain
without neurologic symptoms. A sagittal T 2-weighted MRI scan is shown in Figure
13a, and a contrast enhanced T1-weighted MRI scan is shown in Figure 13b. What is
the most appropriate management for the patient’s symptoms? Review Topic

1 Physical therapy
2 CT-guided needle biopsy and IV antibiotics
3 Revision laminotomy and diskectomy
4 L4-L5 anterior debridement and fusion
5 Open repair of the L4-L5 pseudomeningocele

PREFERRED RESPONSE 1

The MRI scans show Modic changes in the L4-L5 vertebral bodies due to
spondylosis. There is no increased fluid signal or enhancement in the L4-L5 disk to
suggest infection or any other pathologic process. Therefore, the patient’s pain should
be treated with a course of physical therapy and rehabilitation. There is no infection;
therefore, IV antibiotics and debridement are not indicated. Similarly, a
pseudomeningocele is not present. A revision diskectomy is useful for recurrent
radiculopathy but would not be helpful for degenerative low back pain.

(SBQ12SP.29) A 17-year-old female is undergoing posterior instrumented fusion


from T5-T12 for adolescent idiopathic scoliosis. At the time of the correction
maneuver, the neurophysiologist notifies you of a 60% decrease in somatosensory
evoked potential (SSEP) amplitude throughout bilateral lower extremities. Which of
the following is an acceptable approach to manage this finding? Review Topic

1 Immediate wake-up test with examination for clonus


2 Drop the mean arterial pressure (MAP) to ~60mmHg
3 Discontinue instrumentation and optimize MAP to 85mmHg or greater
4 Immediate infusion of intravenous corticosteroids
5 Modification of the anesthesia plan to include inhalational agents only followed by
repeated SSEP testing

PREFERRED RESPONSE 3

The patient has a significant drop in SSEP amplitudes at the completion of the
corrective maneuver. The most appropriate response is to raise the MAP to 85 mmHg
or greater, discontinue the instrumentation, re-evaluate the SSEPs, and if there is no
improvement, to consider reversing the reduction of the deformity.

Intra-operative neurophysiologic monitoring is an effective method to monitor insults


to the spinal cord and its exiting roots during spinal instrumentation. The common
measurements include SSEPs, which monitor sensory potentials transmitted through
the dorsal column system, and motor-evoked potentials (MEPs), which monitor motor
response to a trans-cranial stimulus. Decreases in amplitude and latency of the circuits
are recorded, however diminished signal amplitudes are more sensitive for neurologic
injury, and decreases of of >50-60% being highly concerning. The wake-up test
involves reversal of anesthesia so that an intra-operative neurologic examination can
be performed.

Devlin et al. reviewed the basic science and practice of neurophysiologic monitoring
in spine surgery. They proposed an algorithmic approach to managing intraoperative
alerts which include discontinuation of inhalational anesthetics, increasing the MAP
to >90 mmHg, discontinuing instrumentation, and performing a wake-up test if
neurologic signals fail to normalize.

Herdmann et al. reviewed the practice of neurophysiologic monitoring and the effects
of anesthesia upon signal transduction. They report that anesthesia affecting a
neuron's intrinsic excitability can alter the results of monitoring. Inhalational
anesthetics and decreased MAPs can be responsible for decreased amplitudes.
Vitale et. al. developed a consensus-based intraoperative checklist for management of
lost neuromonitoring signals. In this checklist, the first steps across the surgical and
anesthetic teams should include: stop the case and announce signal losses to the room,
optimize the mean arterial pressure, discuss the status of anesthetic agents, and
discuss reversible surgical actions just prior to signal loss.

Incorrect Answers:
1. Wake-up test is not appropriate without first addressing reversible causes of
amplitude decrease
2. MAP should be increased to >90 mmHg
4. Consideration of corticosteroids is reasonable, although it is not the next most
appropriate step
5. Modification of the anesthesia plan should include decreasing the use of
inhalational anesthetic

(SAE07PE.95) A 15-year-old boy reports a 2-day history of progressive left buttock


pain and severe limping. He denies any history of trauma or radiation of the pain. He
has an oral temperature of 100.4 degrees F (38 degrees C). Examination reveals that
the lumbar spine and left hip have unguarded motion. The abdomen is nontender.
There is moderate tenderness of the left sacroiliac region with no palpable swelling.
Pain is elicited when the left lower extremity is placed in the figure-4 position
(FABER test). Laboratory studies show a peripheral WBC count of 11,500/mmP3P
(normal to 10,500/mmP3P) and an erythrocyte sedimentation rate of 38 mm/h (normal
up to 20 mm/h). Radiographs of the pelvis, hips, and lumbar spine are normal. A
nucleotide bone scan (posterior view) is shown in Figure 44. Initial management
should consist of Review Topic

1 oral nonsteroidal anti-inflammatory drugs.


2 intravenous antistaphylococcal antibiotics.
3 incision and debridement of the retroperitoneal abscess.
4 incision and debridement of the left sacroiliac joint.
5 arthrotomy and irrigation of the left hip joint.

PREFERRED RESPONSE 2

The symptoms, physical findings, and laboratory studies are most consistent with a
diagnosis of infectious sacroiliitis, usually caused by Staphylococcus aureus. Initial
radiographs will be normal, and the diagnosis of sacroiliitis is often delayed. A
technetium Tc 99m bone scan will localize the problem in 90% of patients but may
occasionally give a false-negative result in early cases. If suspicion is high, a gallium
scan or MRI scan may help confirm the diagnosis of sacroiliitis. Needle aspiration of
the sacroiliac joint is difficult; therefore, antibiotic selection is usually empiric or
based on blood cultures. Sacroiliitis that is the result of connective tissue
inflammatory disease is usually bilateral and without fever or leukocytosis. The lack
of hip irritability, spinal rigidity, and abdominal tenderness helps to rule out other
causes of limping with fever, such as psoas abscess, diskitis, and septic hip.

(SAE07PE.98) A 12-year-old girl who is Risser stage 3 has had intermittent mild
midback pain for the past 4 weeks. The pain is worse after prolonged sitting and after
carrying a heavy backpack at school. She occasionally takes acetaminophen, but the
pain does not limit sport activities. Examination reveals a mild right rib prominence
during forward bending. Neurologic examination is normal. Radiographs show a 20-
degree right thoracic scoliosis with no congenital anomalies or lytic lesions.
Management should consist of Review Topic

1 back muscle stretching and reduced weight in the backpack.


2 consultation with a pain management specialist.
3 MRI of the thoracic spine.
4 a technetium Tc 99m bone scan.
5 a thoracolumbosacral orthosis.

PREFERRED RESPONSE 1

Mild scoliosis is not a painful condition, but it usually presents during adolescence.
Intermittent back pain is reported by 25% to 30% of adolescents whether or not
scoliosis is present. Such pain is often attributed to muscle strain from tight muscles,
poor posture, or heavy school backpacks. The clinician must distinguish typical pain
(mild, intermittent, nonlimiting) from atypical pain. The latter requires more careful
examination and imaging studies (bone scan or MRI) to determine the source of pain.
The patient’s age and right thoracic curve pattern are typical for idiopathic scoliosis;
therefore, imaging of the neuroaxis is not necessary to look for cord syrinx, tethering,
or tumor. Brace treatment is not required for this small curve unless future
progression is demonstrated.

(SAE09SN.6) A 45-year-old man reports that he awoke 2 weeks ago with severe pain
in his right arm. Examination reveals weakness in the biceps, brachialis, and wrist
extensors. There is decreased sensation in the thumb and index finger and a
diminished brachioradialis reflex. Assuming this patient has a posterolateral herniated
nucleus pulposus, what level is involved? Review Topic

1 C2-3
2 C3-4
3 C4-5
4 C5-6
5 C6-7

PREFERRED RESPONSE 4

This is a classic C6 nerve injury, and it is most likely the result of a herniated nucleus
pulposus at C5-6. The C5 nerve root controls the elbow flexors, shoulder abductors,
and external rotators. The C7 nerve root controls the elbow extensors, wrist pronators,
and the triceps reflex.

(SAE08AN.31) What structure is located at the tip of the arrow in Figure 18? Review
Topic
1 Left L3 nerve root
2 Right L3 nerve root
3 Right L4 segmental artery
4 Right L4 nerve root
5 Left lateral disk herniation

PREFERRED RESPONSE 2

The structure shown is the exiting nerve root at the L3-4 disk, which is the right L3
root.

(SAE09SN.81) A 23-year-old man is involved in a motor vehicle accident. An AP


radiograph is shown in Figure 29a, and axial and sagittal CT scans are shown in
Figures 29b and 29c. Neurologic examination shows 1/5 strength of his quadriceps
and iliopsoas on the right, with 1/5 quadriceps function on the left. Definitive
treatment of his injury should consist of Review Topic

1 anterior corpectomy with interbody strut.


2 posterior fusion with instrumentation and posterolateral decompression.
3 closed reduction and a thoracolumbosacral orthosis (TLSO).
4 anterior reduction and instrumentation.
5 supine bed rest for 6 weeks, followed by immobilization in a TLSO.

PREFERRED RESPONSE 2

The imaging studies show a fracture-dislocation. Surgical treatment of this injury


consists of a decompression reduction, stabilization, and fusion. A posterolateral
decompression can also be performed as necessary. An isolated anterior procedure in
this type of injury is contraindicated. The anterior longitudinal ligament is most likely
intact; therefore, an anterior procedure further destabilizes the spine. Reduction by an
anterior approach would also be difficult. Nonsurgical management of the neurologic
injury in this patient is not indicated.

(SAE08OS.105) Which of the following changes is seen with age and degeneration in
the intervertebral disk? Review Topic

1 Increased number of blood vessels in the outer annulus fibrosus


2 Increased number of notochordal cells in the nucleus pulposus
3 Increase of diffusion gradient between the annulus fibrosus and the nucleus
pulposus
4 Decreased type 1 collagen
5 Decreased water content of the nucleus pulposus

PREFERRED RESPONSE 5

The intervertebral disk consists of annulus fibrosus, nucleus pulposus, and endplate.
Nucleus cells have a critical need for glucose because they obtain their energy
primarily by glycolysis, even in the presence of oxygen. Disk cells do not require
oxygen to remain alive, but they die at low glucose levels or acidic pH. Nutrients are
supplied from the blood vessels at the margins of the disk and have to traverse the
cartilaginous endplate and the fibrous annulus in order to reach the disk cells. The loss
of the nutrient supply through the vertebral body will starve the cells in the disk center
and may be a major factor in disk degeneration. The gross appearance of the nucleus
pulposus is clear watery gelatinous matrix in the very young disk, but with age the
nucleus pulposus becomes more opaque, and less hydrated and firm. The cellular
composition of the young disk consists of many notochordal cells, but after 10 years
of age, notochordal cells are not seen in the disk. Notochordal cells are the remnant of
embryonal cells in the nucleus pulposus.
(SAE12SN.70) A 78-year-old woman reports a 1-week history of severe low back
pain. She denies any trauma or recent falls. She is neurologically intact, and is able to
ambulate, although she does require the use of a walker. Radiographs of the lumbar
spine show a T11 compression fracture with a 20% loss of anterior column height.
What is the most appropriate management at this time? Review Topic

1 Bed rest until symptoms resolve


2 Analgesics and progressive rehabilitation
3 Anterior thoracic corpectomy and arthrodesis with instrumentation
4 Posterior thoracic decompression and fusion
5 Vertebral cement augmentation

PREFERRED RESPONSE 2

The patient has sustained a thoracic compression fracture, which is very common in
elderly patients, and can occur with minimal to no trauma. There is approximately a
20% loss of anterior vertebral body height. The patient is neurologically stable, and is
able to ambulate with an assistive device. Initial treatment should consist of
progressive mobilization with analgesics as needed. Fractures treated in this manner
have a high rate of success, and surgical treatment is often not necessary. In the
absence of neurologic impairment or impending structural instability, surgical
decompression and fusion is not indicated. Bed rest is contraindicated. Cement
augmentation is a reasonable treatment option when a patient fails nonsurgical
management, although recent studies have called into question its efficacy when
compared with placebo.

(OBQ15.141) A 75-year-old man presents with worsening low back and bilateral leg
pain. The pain worsens with ambulation and improves with sitting. On exam, he has
strong DP and PT pulses. Straight leg raise is negative. A MRI of the lumbar spine is
performed and is pictured in Figure A. On further questioning, which of the following
is the patient also likely to report? Review Topic
1 Increased pain walking down stairs
2 Increased pain while using a stationary bike
3 Increased pain with coughing or sneezing
4 Increased pain after driving a car for a long distance
5 Pain that is worst first thing in the morning and gradually improves as the day goes
on

PREFERRED RESPONSE 1

The patient has lumbar spinal stenosis with neurogenic claudication and therefore is
likely to experience worsening pain with activities that result in lumbar extension,
such as walking down stairs.

Lumbar spinal stenosis often results from degenerative changes of the intervertebral
disc and facet joints which ultimately narrows the space available for the thecal sac
and exiting nerve roots. Patients can present with neurogenic claudication, reported as
worsening leg and/or back pain with ambulation and diminished walking capacity.
MRI may demonstrate disc degeneration/bulging, hypertrophy of the ligamentum
flavum and facet capsule, and narrowing of the central canal. Nonoperative
management includes NSAIDs, PT and epidural steroid injections (ESI). Surgery is
reserved for patients who have failed nonoperative measures and includes
decompressive laminectomy with or without fusion depending on presence of
instability.

Issack et al reviewed degenerative lumbar spinal stenosis. Unlike patients with


vascular claudication, patients with neurogenic claudication are able to improve
walking tolerance with postural changes, specifically with flexed-forward posture
(such as leaning forward on a shopping cart). They are unable to improve their
symptoms simply by cessation of walking. Patients with neurogenic claudication tend
to lack the trophic changes of the skin on the legs/feet as well as diminished pulses
characteristic of vascular disease.

Young et al reviewed the utilization of lumbar ESI for low back and leg pain. The
authors concluded that lumbar ESI are a reasonable nonsurgical option to provide
temporary symptomatic relief. Fluoroscopic guidance facilitates accurate placement
of the injection into the epidural space, while its nonuse may lead to higher
percentage of technical failures. Lastly, the transforaminal approach is more selective
than the interlaminar approach and can provide diagnostic information as well as
symptom relief.

Figures A and B are T1 sagittal and T2 axial MR images, respectively, of the lumbar
spine demonstrating significant central canal stenosis most notable at L4-L5 with
broad disc protrusion, facet degeneration and infolding of the ligamentum flavum.

Incorrect Responses:
Answer 2: Pain with use of a stationary bike is more suggestive of vascular
claudication. This patient would likely not experience pain with use of a stationary
bike, as he would be in a flex-forward position.
Answers 3 + 4: Pain with increased intraabdominal pressure (coughing, sneezing) and
pain with prolonged sitting are more characteristic of lumbar disc herniation.
Answer 5: Pain that is worse in the morning and improves as the day goes on is more
characteristic of ankylosing spondylitis.

(SAE12SN.31) A 78-year-old man has a history of worsening bilateral calf pain with
activity. MRI scans are shown in Figures 31a through 31d. His symptoms are not
improved with forward flexion of the lumbar spine. His lower extremity pain is
relieved when he sits or ceases activity. Which of the following tests would be most
helpful in establishing a diagnosis? Review Topic
1 Selective nerve root blocks
2 Electromyography and a nerve conduction velocity study of the lower extremities
3 Ankle-brachial index (ABI)
4 Bilateral hip radiographs
5 Post-myelography CT scan

PREFERRED RESPONSE 3

The differential diagnosis of degenerative lumbar stenosis is extensive. Vascular and


neurogenic claudication frequently coexist in the older population. Therefore, it is
important to determine the specific etiology of a patient's lower extremity claudication
prior to any surgical intervention. Vascular claudication is relieved with cessation of
activity, whereas neurogenic claudication requires that the patient sit down or flex the
lumbar spine forward to increase the canal diameter. Because this patient does not
experience improvement in his symptoms with sitting or forward flexion, it is likely
that he is experiencing vascular claudication. The ankle-brachial index (ABI) is the
ratio of the blood pressure in the lower legs to the blood pressure in the arms.
Compared with the arm, lower blood pressure in the leg is a sign of peripheral
vascular disease. The ABI is calculated by dividing the systolic blood pressure in the
arteries at the ankle and foot by the higher of the two systolic blood pressures in the
arms. An ABI value between 0.40 to 0.80 is moderately decreased and such patients
often experience symptoms such as intermittent claudication. Selective nerve root
blocks prove to be more useful in identifying specific level(s) of involvement in
patients experiencing radicular pain and paresthesias. Their utility is less helpful in
lower extremity claudication. Electrophysiologic studies are rarely useful, except in
identifying the presence and source of a peripheral neuropathy. About 80% of patients
with symptomatic lumbar stenosis will demonstrate electromyographic changes.
Osteoarthritis of the hip may be associated with buttock, groin, hip, and thigh pain.
Decreased range of motion and hip joint pain, especially in internal rotation and
abduction, are common findings in patients with degenerative arthritis of the hip.
While post-myelography CT has been found superior to MRI as a single study for the
preoperative planning of decompression for lumbar spinal stenosis, it will not assist in
differentiating vascular from neurogenic claudication.

(OBQ15.99) A 27-year-old professional football player complains of acute onset neck


and radiating left arm pain after making a tackle. For approximately 1 week after
injury his left deltoid strength was 4/5. An MRI is performed, which demonstrates a
C4-5 disc herniation without evidence of cord compression. He was treated with a
brief course of oral steroids followed by aggressive physical therapy. At this time he
is asymptomatic and his neurologic exam is normal. If the patient returns to
professional football play, what is his increased risk of sustaining a catastrophic spinal
cord injury? Review Topic

1 There is no increased risk for spinal cord injury.


2 5-10%
3 20-25%
4 30-35%
5 Greater than 50%

PREFERRED RESPONSE 1

This patient is a professional football player who likely is suffering from an acute left
sided C4-5 cervical disc herniation causing a C5 radiculopathy. After non-operative
treatment and return to sport, his likelihood of sustaining a catastrophic spinal cord
injury is less than 5%.

A C5 radiculopathy from an acute disc herniation can manifest as pain in the neck and
affected arm, as well as weakness in the affected myotome. The natural history of this
pathology is symptomatic improvement over time. In professional athletes, there are
few studies to guide treatment, but oral methylprednisolone has been shown to
improve symptoms and expedite return to play. The risk of sustaining catastrophic
spinal cord injury after return to play is considered low, and has been reported to be
0%.

Wong et. al. performed a systematic review of the literature identifying the natural
history, clinical course, and prognostic factors of symptomatic cervical disc
herniations with radiculopathy. They found substantial symptomatic improvement
within the first 4-6 months after onset, with maintained improvements for 2-3 years.
No patients in their review developed progressive neurological deficits or myelopathy
at any point during follow up.

Meredith et. al. performed a retrospective chart review of 16 professional football


players with cervical disc herniations. The authors recommended surgery if patients
had MRI with cord compression and signal change within the cord, but otherwise
encouraged nonoperative treatment with return to sports after symptoms improved
and repeat MRI demonstrated no cord compression. Symptoms generally improved
with a course of anti-inflammatory medications including NSAIDs, oral
methylprednisolone, and epidural steroid injections. Nine of the 16 patients were able
to return to play, and at one year after return to play there were no catastrophic spinal
cord injuries among the group.

Incorrect Answers:
Answers 2-5 all suggest rates of subsequent catastrophic spinal cord injury higher
than what is reported in the literature.

(SAE12SN.1) A 56-year-old man has a chief complaint of leg weakness and inability
to walk. Examination reveals 5 out of 5 motor strength in all lower extremity muscle
groups tested and normal sensation to light touch in both lower extremities. The
patient is slow in getting up from a seated position and has an unsteady wide-based
gait. An MRI scan of the lumbar spine is shown in Figure 1. What is the next most
appropriate course of action? Review Topic

1 Electromyography and nerve conduction velocity studies of bilateral lower


extremities
2 Multilevel lumbar laminectomy
3 MRI of the thoracic and cervical spine
4 MRI of the brain
5 Epidural steroid injections
PREFERRED RESPONSE 3

The patient is having gait problems suspicious for spinal cord compression. MRI of
the thoracic and cervical spine should be performed to evaluate for spinal cord
compression. Reports of leg weakness in the absence of discrete motor weakness on
manual testing, and the appearance of an unsteady wide-based gait are more
consistent with myelopathy as a cause of the gait difficulty rather than lumbar
stenosis. Although the MRI scan of the lumbar spine shows multilevel spinal stenosis
that is mild to moderate, it does not clearly explain the patient's signs and symptoms.
Electromyography and nerve conduction velocity studies of the lower extremities are
unlikely to add significantly to the diagnosis. Epidural steroid injections are not
indicated. Lumbar decompression is unlikely to help the patient because the source of
the patient's problem does not originate in the lumbar spine. MRI of the brain could
be considered as a secondary imaging study if the cervical and thoracic MRI scans fail
to identify an obvious cause for gait instability.

(SBQ12SP.92) A 36-year-old man presents to the emergency department after being


involved in a motor vehicle collision. He is complaining of back pain and imaging
shows the findings in Figure A. On neurological examination, he does not have any
deficits. MRI shows approximately 25% canal encroachment and no evidence of
injury to the posterior ligamentous complex. Which of the following is the most
appropriate course in management? Review Topic

1 Strict bedrest for six weeks then progressive weightbearing


2 Ambulation as tolerated with or without a TLSO
3 Surgical decompression and anterior stabilization
4 Surgical decompression and posterior stabilization
5 Surgical decompression and combined anterior/posterior stabilization

PREFERRED RESPONSE 2

The patient has a L1 burst fracture with minimal retropulsion of bony fragments in the
spinal canal. In the absence of neurological deficits and injury to the PLC, the most
appropriate treatment is ambulation as tolerated with or without a
thoracolumbrosacral orthosis (TLSO).

Thoracolumbar burst fractures are typically caused by an axial load with flexion and
commonly found in this location due to increased motion at these segments. With an
intact posterior ligamentous complex (PLC) and no neural compromise, TLSO is the
mainstay of treatment. If there is evidence of neurological deficit and/or PLC injury,
decompression and fusion are indicated. The degree of acceptable kyphosis is
controversial. The choice of anterior versus posterior approach is based on ease of
decompression.

Vaccaro et al. introduced a new classification system for thoracolumbar injuries,


TLICS, based on morphological appearance, integrity of the posterior ligamentous
complex, and neurological status. They advocate use of the system for nonoperative
versus operative decision making and communication between surgeons.

Bailey et al. completed a randomized, nonblinded controlled trial to determine the


efficacy of bracing for AO type A0-A3 thoracolumbar burst fractures. Both groups
were encouraged to ambulate as tolerated and the no brace group had bending
restrictions for 8 weeks. They found no difference in the Roland Morris Disability
Questionnaire (RMDQ) score at 3 months after injury.

Figure A is sagittal CT scan of the lumbar spine showing a burst fracture of L1 with
minimal retropulsion. Illustration A is the TLICS classification with score of 4 being
the branch point for nonoperative versus operative management.

Incorrect Answers:
Answer 1: Bedrest is not required for management of burst fractures.
Answer 3-5: In the absence of neurological compromise or PLC injury, operative
management is not indicated.

(SAE11UE.47) A 45-year-old man reports a history of a popping sensation and pain


in the right shoulder while lifting boxes 6 months ago. The pain has persisted with
loss of motion of the shoulder. Radiographs and MRI scans are shown in Figures 47a
through 47d. Which of the following studies is likely to produce a significant positive
result? Review Topic
1 Rheumatoid factor
2 HLA-B27
3 Synovial fluid analysis
4 MRI of the upper cervical spine
5 Urine screen for tetrahydrocannabinol (THC)

PREFERRED RESPONSE 4

The patient has a neuropathic joint secondary to syringomyelia that can be seen on a
cervical MRI scan. The patient sustained minimal trauma that lead to a chronic
anterior glenohumeral dislocation. He did not seek treatment for several months and
has a massive rotator cuff tear and hygroma on MRI in addition to the chronic
dislocation. Rheumatoid arthritis does not present with a neuropathic picture, except
theoretically as the result of numerous intra-articular cortisone injections. This
Charcot picture is inconsistent with ankylosing spondylitis or gout. Cannabis use is
not typically associated with seizures that could produce anterior as well as posterior
shoulder dislocations.

(SAE10PE.22) An 8-year-old girl has asymmetry on a forward bend test of the spine.
She is asymptomatic and has a normal clinical neurologic examination. Radiographs
are shown in Figures 22a and 22b. What should be the next step in her work-up?
Review Topic
1 MRI of the cervical thoracic lumbar spine
2 Supine side bending radiographs of the spine
3 Return to the clinic in 12 months with repeat radiographs
4 Anterior and posterior spinal fusion with instrumentation
5 Echocardiogram and renal ultrasound

PREFERRED RESPONSE 1

There are several reasons to obtain an MRI of the entire spinal cord of this patient to
evaluate for abnormalities. These include her young age and the presence of a left-
sided curve. For juvenile scoliosis patients with more than a 20-degree Cobb angle,
there is an approximately 20% prevalence of a neurologic abnormality. Therefore,
recommendations for work-up include an MRI scan of the entire spine.

(SAE09SN.49) An 82-year-old man is seen in consultation after being admitted for a


fall from ground level. There was no loss of consciousness and the patient recalls
striking his head and sustaining a hyperextension-type injury to the cervical spine.
Examination reveals an 8-cm head laceration with only mild axial neck tenderness.
He has generalized weakness throughout the upper extremities and maintained motor
function of the lower extremities. There are no obvious sensory deficits, and the
bulbocavernous reflex and deep tendon reflexes are maintained. What is the most
appropriate diagnosis at this time? Review Topic

1 Anterior cord syndrome


2 Central cord syndrome
3 Posterior cord syndrome
4 Brown-Séquard syndrome
5 Spinal shock
PREFERRED RESPONSE 2

Incomplete cord syndromes have variable neurologic findings with partial loss of
sensory and/or motor function below the level of injury. Incomplete cord syndromes
include the anterior cord syndrome, the Brown-Séquard syndrome, central cord
syndrome, and posterior cord syndrome. Central cord syndrome is characterized with
greater motor weakness in the upper extremities than in the lower extremities. The
pattern of motor weakness shows greater distal involvement in the affected extremity
than proximal muscle weakness. Anterior cord syndrome involves a variable loss of
motor function and pain and/or temperature sensation, with preservation of
proprioception. The Brown-Séquard syndrome involves a relatively greater ipsilateral
loss of proprioception and motor function, with contralateral loss of pain and
temperature sensation. Posterior cord syndrome is a rare injury and is characterized by
preservation of motor function, sense of pain, and light touch, with loss of
proprioception and temperature sensation below the level of the lesion. Spinal shock
is the period of time, usually 24 hours, after a spinal injury that is characterized by
absent reflexes, flaccidity, and loss of sensation below the level of the injury.

(SBQ13PE.25) A 7-year-old girl presents with a early onset scoliosis as seen in the
PA radiograph in Figure A. Images do not demonstrate any vertebral anomalies.
Physical exam shows normal neurologic function in her lower extremities. An MRI of
the spinal axis should be obtained to rule out all of the following pathologies
EXCEPT: Review Topic

1 Atlantoaxial rotatory instability


2 Syringomyelia
3 Spinal cord tumor
4 Dysraphism
5 Tethered cord
PREFERRED RESPONSE 1

Early-onset scoliosis is associated with syringomyelia, spinal cord tumor, dysraphism,


and tethered cord, and therefore they must be ruled out with a preoperative MRI.
There is no association between early-onset scoliosis and atlantoaxial rotatory
instability.

Early-onset scoliosis can be classified as idiopathic, neuromuscular, or congenital. For


this patient without bony or neurologic abnormalities, it would likely be diagnosed as
juvenile (bewteen ages 3 and 10 years) idiopathic scoliosis. Commonly associated
spinal pathologies (even in patients with no radiographic or neurologic abnormalities)
include syringomyelia, spinal cord tumor, dysraphism, tethered cord, and Arnold-
Chiari malformation. Before considering any surgical intervention, such as a growing
rod construct, these intra-spinal conditions must be ruled-out with an MRI of the
spinal axis.

Gillingham et al. present a review of the etiology, diagnosis, and management of


early-onset scoliosis. They cite other studies that demonstrated rates as high as 21% of
patients with early-onset scoliosis and a normal neurologic exam having occult spinal
pathology such as those mentioned above. They recommend MRI for patients with
early-onset scoliosis measuring greater than 20° prior to any surgical intervention.

Figure A is a PA radiograph of a skeletally immature patient with a 90° right thoracic


curve. Illustration A is an PA radiograph of the same patient after treatment with a
growing-rod construct and correction of the scoliotic curve to 46°.

Incorrect Answers:
Answers 2: Syringomyelia in pediatric patients is typically seen in the thoracic spine,
and can be diagnosed by MRI.
Answer 3: The most common spinal cord tumors in children are astrocytomas and
developmental tumors (e.g., dermoid, epidermoid, and teratomas). The gold standard
imaging for these tumors is MRI.
Answer 4: Dysraphism is a split in the spinal cord that is believed to be in the same
spectrum of deformity as a tethered cord. The gold standard imaging is MRI.
Answer 5: A tethered cord is commonly asymptomatic until adulthood when
increased strain on the tethered cord results in neurologic deficits. Occasionally a
tethered cord is seen incidentally on prenatal ultrasounds. MRI is the gold standard
imaging for ruling out a tethered cord in both children and adults. A tethered cord
frequently requires release to prevent neurologic sequelae.

(SAE10BS.17) A patient has a C6 spinal cord injury. Following stabilization of the


spine, the patient should be advised that their expected maximum level of function
and ambulatory capacity will be achieved with use of which of the following devices?
Review Topic

1 An electric wheelchair with puffer control


2 An electric wheelchair with hand controls
3 A manual wheelchair and sliding board transfers
4 A manual wheelchair and independent transfers
5 Crutches with long leg braces for short distance ambulation

PREFERRED RESPONSE 3

A patient with an injury at the level of: C4 injury needs puffer control; C5 can use
hand controls; C6 can use a manual wheelchair and sliding board transfers; C7 allows
independent transfers; and no cervical injury routinely allows ambulation with
crutches and leg braces.

(SAE10BS.10) A patient has a C6-7 herniated nucleus pulposus. What is the most
likely distribution of symptoms? Review Topic

1 Pain in the posterior neck and scapula, numbness over the clavicle, and weakness of
the head and neck extensors
2 Pain in the shoulder, numbness over the lateral shoulder, weakness of the deltoid
3 Pain at the elbow, numbness over the thumb and index finger, weakness of the
biceps and wrist extensors
4 Pain at the forearm and hand, numbness of the middle finger, weakness of the
triceps and finger extensors
5 Pain at the wrist, numbness in the little and ring fingers, weak finger flexors

PREFERRED RESPONSE 4

A C6-7 herniated nucleus pulposus is most likely to produce a C7 radiculopathy


resulting in pain at the forearm and hand, numbness of the middle finger, and
weakness of the triceps and finger extensors. Alternative answers represent sequelae
of symptomatic disk herniations at C3-C4 (1), C4-C5 (2), C5-C6 (3), and C7-T1 (5),
respectively.
(SBQ12SP.27) A 24-year-old male sustains the injury shown in Figure A. What was
the most likely mechanism of injury? Review Topic

1 Hyperextension
2 Flexion-distraction
3 Flexion-compression
4 Rotational
5 Pure axial load

PREFERRED RESPONSE 3

Figure A shows a quadrangular fracture pattern of C5. These injuries are observed
with flexion-compression loads.

Quadrangular fractures of the cervical spine are considered flexion teardrop fractures.
However, they present with a larger anterior lip fragment compared to the classic
teardrop fracture pattern. The radiographic findings include a quadrangular-shaped
fragment from the anterior one-third of the vertebral body with significant posterior
vertebral subluxation, angular kyphosis, and an increased interspinous space with
facet subluxation due to disruption of the posterior elements. These are unstable
fractures, and almost always require anterior and posterior stabilization.

Moore et al. studied the reliability of Cervical Spine Injury Severity Score to measure
stability after cervical spine trauma. The classification system is based on
morphologic descriptions and, secondly, on stability based on a quantifiable value.
They showed that the Cervical Spine Injury Severity Score had excellent reliability
with intra-observer intraclass correlation coefficients (ICC) >0.97 and inter-observer
ICC >0.88.

Vaccaro et al. published the subaxial cervical spine injury classification system
(SLIC). This systems involves 3 main categories (injury morphology, disco-
ligamentous complex, and neurologic status). The overall injury severity score is
obtained by summing the scores from each category. They propose that SLIC < 4 can
be treated non-operatively.

Allen et al. published a classification system of cervical spine injuries which breaks
injuries of the subaxial spine into six phylogenic groups based on mechanism of
injury. These include: 1) flexion-compression 2) vertical-compression 3) flexion-
distraction 4) extension-compression 5) extension-distraction 6) lateral flexion. Facet
dislocation is caused by flexion-distraction force.

Figure A shows a lateral radiograph of the cervical spine demonstrating a typical


flexion-compression fracture with anteriorly displaced quadrangular fragment.
Illustration A shows a flexion-compression injury. Illustration B shows the
mechanism of injury for a quadrangular fracture of the cervical spine. This unstable
fracture pattern is characterized by anterior column failure in flexion/compression and
posterior column failure in tension. Illustration C shows an illustration of the fracture
morphology according to the Allen and Ferguson classification.

Incorrect Answers:
Answer 1: Hyperextension injuries usually result in central cord injuries
Answer 2: Flexion-distraction injuries usually result in facet dislocations.
Answer 4: Rotational injuries will usually cause soft-tissue injury or high cervical
injury.
Answer 5: Pure axial load will usually result in cervical burst fractures.

(SAE09SN.42) Radiating pain associated with a posterolateral thoracic disk


herniation typically follows what pattern? Review Topic

1 Extending down the spine into the lumbosacral region


2 Down the inner aspect of either upper extremity
3 Cephalad up to the cervicothoracic junction
4 Around or through the chest to the anterior wall
5 Down the contralateral lower extremity

PREFERRED RESPONSE 4

Although symptomatic thoracic disk herniations can affect more caudal structures,
even to the point of paralysis, the pattern of radiating pain has been described as either
following the dermatomal band around the chest or feeling to the patient as if the pain
passes straight anteriorly to the chest wall.

(SAE12SN.29) A 75-year-old woman is undergoing a T10-S1 lumbar decompression


and fusion for severe degenerative scoliosis. During the deformity corrective
maneuver, intraoperative neuromonitoring revealed a sustained 80% decrease in
somatosensory-evoked potential (SSEP) amplitudes. Appropriate lead placement and
functioning has been reconfirmed by the neuromonitoring technician. The
anesthesiologist has ruled out any anesthetic-related or hemodynamic issues. What is
the next appropriate step in management? Review Topic

1 Completion of the surgical procedure with continued monitoring


2 Reversal of the corrective maneuver and consideration of a wake-up test
3 Administration of high dose corticosteroids intraoperatively
4 Removal of all instrumentation
5 Discontinue monitoring

PREFERRED RESPONSE 2

The most appropriate management is discontinuation of the spinal instrumentation


procedure, including releasing any distractive forces. Given the ongoing changes,
proceeding with the procedure and/or resetting the baseline amplitudes is inadvisable.
If the SSEPS amplitudes fail to return in a timely fashion, it is strongly recommended
to consider a wake-up test. The purpose of intraoperative neuromonitoring is to
provide a real-time assessment of the functional integrity of the central and peripheral
nervous systems during surgery to prevent iatrogenic injury. Sustained decreased
SSEP amplitudes of greater than 50% and transcranial electric Motor-Evoked
Potentials (tceMEP) amplitudes of greater than 75% are indicative of a possible
significant intraoperative neurologic complication. Although intraoperative
corticosteroids might be helpful, especially in the event of a continued SSEP change,
the most important next step would be to release the distractive forces and reevaluate
the patient's neurologic status.

(SAE07PE.44) A 4-month-old infant is referred for evaluation of congenital scoliosis.


The child has no congenital heart anomalies, and a renal ultrasound shows that he has
one kidney. Examination reveals mild scoliosis and a large hairy patch on the child’s
back. Neurologic evaluation is normal for his age. A clinical photograph and
radiograph are shown in Figures 19a and 19b. Initial management should consist of
Review Topic
1 referral to a plastic surgeon to remove the hairy patch.
2 MRI of the entire spine.
3 physical therapy and repeat evaluation and radiographs in 1 year.
4 anterior and posterior fusion of the anomalous regions of the spine to prevent
deformity.
5 voiding cystourethrography.

PREFERRED RESPONSE 2

Congenital anomalies of the spine, including failure of formation and failure of


segmentation, are associated with other anomalies in other organ systems that develop
at the same time. These include anomalies in the genitourinary system, cardiac
anomalies, Sprengel’s deformity, radial hypoplasia, and gastrointestinal anomalies
including imperforate anus and trachealesophageal fistula. Spinal dysraphism is the
most common associated abnormality. McMaster found an 18% incidence before the
common use of MRI. Bradford and associates reported on 16 of 42 patients with
congenital spinal anomalies and spinal dysraphism using MRI. Neural axis lesions
may be associated with visible midline abnormalities such as a hairy patch or nevus.
The child has already had a cardiac and renal work-up, and based on the findings of
the hairy patch and congenital vertebral anomalies, MRI of the entire spine is prudent
at this time. Spinal fusion is indicated for progressive congenital scoliosis or kyphosis.
Physical therapy does not affect the natural history of congenital scoliosis.

(SAE12SN.91) A 35-year-old man who has had a 6-month history of low back pain
and tenderness now reports worsening pain and stiffness in the hips and entire back.
An AP radiograph of the pelvis demonstrates fusion of the sacroiliac joints bilaterally.
What is the next most appropriate step in management? Review Topic

1 Anesthetic injections in both sacroiliac joints


2 Sacroiliac fusion with plate fixation
3 Anti-inflammatory medications, physical therapy, and HLA-B27 testing
4 Patient reassurance and follow-up as needed
5 Immediate bilateral sacroiliac joint aspiration and culture

PREFERRED RESPONSE 3

The patient has a classic presentation of early ankylosing spondylitis. Sacroiliac joint
fusion is the earliest radiographic finding and is typically followed by cephalad spinal
progression. Early treatment of ankylosing spondylitis consists of nonsteroidal anti-
inflammatory drugs and physical therapy to preserve spinal motion. HLA-B27 testing
is positive in most (about 95%) patients; however, it is not pathognomonic because it
can be positive with other conditions. Considering the progressive nature of this
disease, further work-up in a patient with potential ankylosing spondylitis is not
warranted. Sacroiliac joint anesthetic injections and sacroiliac fusion are not
recommended treatments for early ankylosing spondylitis. Aspiration of the sacroiliac
joints can be done if sacroiliac joint infection is suspected; however, in the absence of
fever or other constitutional symptoms, infection is unlikely.

(SAE12SN.7) Figures 7a through 7d show the radiographs and MRI scans of a 69-
year-old woman with neck and upper extremity pain and progressive deformity of the
cervical spine. What is the most likely diagnosis? Review Topic

1 Postlaminectomy kyphosis
2 Ankylosing spondylitis
3 Occipitocervical dissociation
4 C3-4 pseudarthrosis
5 Klippel-Feil syndrome

PREFERRED RESPONSE 1

Laminectomy without fusion for the treatment of cervical spondylotic myelopathy


currently plays a minor role in the management of this disorder because of its many
disadvantages. The actual incidence of postlaminectomy kyphosis is unknown, but is
estimated to be between 11% and 47%. It can result in recurrent myelopathy if the
spinal cord becomes draped over the kyphosis. In addition to the neurologic sequelae,
the kyphosis itself can be a source of neck pain and deformity. Spondylolisthesis can
develop, contributing to further cord compression. In this case, the patient had
undergone a previous C4-5 anterior cervical diskectomy and fusion followed by a
posterior laminectomy from C2 through C7, without fusion. This has resulted in
severe kyphosis (i.e. postlaminectomy kyphosis) with grade II-III spondylolisthesis at
C3-4 and a grade I spondylolisthesis at C2-3. While ankylosing spondylitis can also
result in a chin-on-chest deformity secondary to ankylosis, there is no evidence of
marginal syndesmophytes in the imaging studies to suggest this diagnosis. The
occiput is hyperextended on C1 on the lateral upright radiograph to compensate for
the kyphosis in an attempt to maintain horizontal gaze. This results in an unusual
appearing relationship on the imaging studies. However, there is no widening of the
distance between C1 and the occiput and no evidence of soft-tissue injury on the MRI
scans to suggest an acute injury. C3-4 demonstrates an unstable spondylolisthesis and
was never intended to be included in the C4-5 fusion. Klippel-Feil syndrome is the
failure of segmentation of the cervical spine. The classic triad includes congenital
fusion, low hairline, and a web neck.

(SAE12SN.25) What factor is associated with the highest risk for in-hospital
complications for patients undergoing a lumbar fusion for degenerative
spondylolisthesis? Review Topic

1 Hospital size
2 Gender
3 Race
4 Age
5 One comorbidity

PREFERRED RESPONSE 4

Age and having three or more comorbidities is associated with a higher rate of
complications in patients undergoing a lumbar fusion for lumbar degenerative
spondylolisthesis. Race, gender, and hospital size have not been found to be
associated with higher complication rates.

(OBQ15.205) A 39-year-old man presents with back pain radiating to the left lower
extremity for the past 6 weeks. Magnetic resonance images of the pathology are
shown in Figure A, B, and C. Which of the following would you expect on his clinical
examination? Review Topic

1 Pain/parasthesias near the medial malleolus due to impingement of L4 nerve root


2 Pain/parasthesias near the medial malleolus due to impingement of L3 nerve root
3 Pain/parasthesias near the medial epicondyle of femur due to impingement of L4
nerve root
4 Pain/parasthesias near the medial epicondyle of femur due to impingement of L3
nerve root
5 Intermittent claudication due to impingement on segmental vessel

PREFERRED RESPONSE 4

The patient presents with MRI suggestive of far lateral disc herniation at the L3-L4
disc space. He would be expected to have symptoms in L3 nerve distribution, near the
medial epicondyle of the femur. Associated motor manifestations may include
quadriceps weakness and/or diminished patellar reflex.

Far lateral disc herniations constitute approximately 5-10 percent of disc hernations in
the lumbar spine. Given the more vertical anatomy of lumbar nerve roots, at a given
disc level, a central or paracentral herniation will affect the traversing nerve root (e.g.
nerve root of level below) and a far lateral herniation will affect the exiting nerve root
(e.g. nerve root of the level above). Similar to the more common central and
paracentral herniations, approximately 90% of patients will improve without surgical
intervention. When surgical intervention is needed, the paraspinal approach of Wiltse
is utilized.

Marquadt et al. reported long term outcomes of surgical management of far lateral
disc herniations. At an average of 146 months follow up, 56.3% of patients had
complete relief of symptoms and 27.6% had permanent residual symptoms. Over 75%
of patients subjectively rated their outcomes as excellent.

Figure A and B are T2 and T1 axial MRI images, respectively, showing the L3-L4
disc space with a left far lateral disc herniation. Figure C is a left parasagittal T2 MRI
image showing impingement on the L3 nerve root. Structures are labeled in
illustration A. Illustration B shows the dermatomes of the lower extremity.

Incorrect Answers:
Answers 1 and 3: A far lateral disc herniation will affect the exiting L3 nerve root.
Answer 2: Impingement on the L3 nerve root will lead to symptoms near the medial
epicondyle of the femur. The skin over the medial malleolus is innervated by the L4
nerve root
Answer 5: A far lateral disc herniation would not impinge on the segmental vessels.
Further, given the abundant collateral flow, there would be no symptoms with vessel
compression.

(SAE12SN.97) A patient who underwent an L5-S1 hemilaminotomy and partial


diskectomy for radiculopathy 3 weeks ago now reports increasing leg and back pain
with radicular signs. An axial T2-weighted MRI scan is shown in Figure 97a, an axial
T1-weighted MRI scan is shown in Figure 97b, and a contrast enhanced T1-weighted
MRI scan is shown in Figure 97c. What is the most appropriate management for the
patient's symptoms? Review Topic

1 Irrigation and debridement of deep wound infection


2 CT-guided needle biopsy and IV antibiotics
3 Revision laminotomy and diskectomy
4 L4-L5 anterior debridement and fusion
5 Open repair of the L4-L5 pseudomeningocele

PREFERRED RESPONSE 3

The MRI scans show a recurrent disk herniation. There is no increase fluid signal or
enhancement to suggest infection or any other pathologic process. There is no
infection; therefore, IV antibiotics and debridement are not indicated. Similarly, a
pseudomeningocele is not present. In addition, with progressive weakness, physical
therapy is not appropriate. A revision diskectomy is useful for recurrent
radiculopathy.
(SAE09SN.64) A previously healthy 29-year-old man reports a 2-day history of
severe atraumatic lower back pain. He denies any bowel or bladder difficulties and no
constitutional signs. Examination is consistent with mechanical back pain. No focal
neurologic deficits or pathologic reflexes are noted. What is the most appropriate
management? Review Topic

1 Radiographs, including anterior, lateral, and oblique views


2 MRI of the lumbar spine and follow-up at the clinic in 1 week
3 Caudal epidural steroid injection
4 Reassurance, limited analgesics, and early range of motion as tolerated
5 Immediate MRI of the lumbar spine and possible urgent surgical decompression

PREFERRED RESPONSE 4

In general, a previously healthy patient with an acute onset of nontraumatic lower


back pain does not need diagnostic imaging before proceeding with therapeutic
treatment. In the absence of any “red flags” during the history and physical
examination, such as trauma or constitutional symptoms (ie, fevers, chills, weight
loss), the appropriate treatment for acute onset lower back pain is purely symptomatic
treatment including limited analgesics and early range of motion. Diagnostic imaging
is not necessary unless the initial treatment is unsuccessful and symptoms are
prolonged. Miller and associates suggested that the use of radiographs can lead to
better patient satisfaction but not necessarily better outcomes.

(SAE07PE.37) A 6-year-old girl has a painless spinal deformity. Examination reveals


2+ and equal knee jerks and ankle jerks, negative clonus, and a negative Babinski.
The straight leg raising test is negative. Abdominal reflexes are asymmetrical. PA and
lateral radiographs are shown in Figures 15a and 15b. What is the next most
appropriate step in management? Review Topic

1 MRI of the spinal axis


2 Physical therapy
3 A brace for scoliosis
4 Observation, with reevaluation in 6 to 12 months
5 Posterior spinal fusion from T6 to T12

PREFERRED RESPONSE 1

The patient has an abnormal neurologic exam as shown by the abnormal abdominal
reflexes. Furthermore, she has a significant curve and is younger than age 10 years.
These findings are not consistent with idiopathic scoliosis. MRI will best rule out
syringomyelia or an intraspinal tumor. Bracing and surgery are not indicated for this
small curvature prior to obtaining an MRI scan.

(SAE12SN.24) A 36-year-old man has a 2-day history of acute lower back pain with
severe radicular symptoms in the left lower extremity. The patient has a positive
straight leg test at 40 degrees on the left side and mild decreased sensation on the
dorsum of the left foot. What is the most appropriate management at this time?
Review Topic

1 Urgent admission to the hospital for surgical intervention


2 Immediate MRI of the lumbar spine as an outpatient
3 Anti-inflammatory medications and activity modification
4 Caudal epidural steroid injection
5 Electromyography

PREFERRED RESPONSE 3

In the absence of any severe progressive neurologic deficits or other red flags, the
most appropriate management for an acute lumbar disk herniation is nonsurgical care.
Conservative treatments such as limited bed rest, anti-inflammatory medications, and
judicious use of pain medications are appropriate in this clinical situation. Up to 90%
of patients will experience a resolution of symptoms without the need for surgical
intervention within a 3-month window. In the acute setting, with no neurologic
deficits, immediate MRI of the lumbar spine is neither beneficial nor warranted.
Likewise, without signs of an acute deficit, emergent surgical intervention and caudal
epidural steroid injections are not needed.
(OBQ12.167) A 55-year-old man presents with low back pain that has progressed
over the last year. He reports the pain is worse with activity, especially when bending
forward and lifting objects. He denies any pain in the buttocks or lower extremities.
On physical he has age-appropriate motion in the lumbar spine. He is neurologically
intact in the lower extremities. Figure A shows his axial and sagittal T2-weighted
MRI scans. A histological sample of this lesion would most likely show Review
Topic

1 dense, compact concentric lamellae of fibrocollagenous tissue with occasional


fibroblast-like chondrocytes
2 semifluid gelatinous matrix with oval chondrocytes
3 ossified nidus surrounded by a radiolucent halo, in turn surrounded by dense,
reactive osteosclerosis
4 irregular fascicles of collagenous stroma with pleomorphic cells with foamy
cytoplasm and marked atypia in a storiform pattern
5 synovial cells covering a stroma with vascular granulation tissue

PREFERRED RESPONSE 5

The clinical presentation is consistent with a synovial cyst. Histology would most
likely show synovial cells covering a stroma with vascular granulation tissue.

Juxtafacet cysts may include synovial cysts or ganglion cysts. Synovial cysts are lined
with epithelium (cuboid synovial cells) and contain clear or xanthochromic fluid.
Ganglion cysts which have no synovial lining, and contain gelatinous material from
myxoid degeneration of the fibrous adventitial tissue.

Xu et al. reviewed the treatment of 195 synovial cysts. They found that patients
treated with laminectomy had the highest risk of cyst recurrence (3%). In contrast,
decompression with instrumented fusion had the lowest incidences of cyst recurrence
(0%) or back pain (although they had the longest hospital stay, and greatest blood
loss).

Figure A is a T2-weighted MRI (left, axial; right, sagittal) showing a facet synovial
cyst arising from an arthritic left L3-4 facet joint. It occupies much of the space in the
spinal canal and indents and displaces the thecal sac. Illustration A shows a
hemorrhagic synovial cyst showing synovial cell lining, fibroconnective tissue with
widespread hemorrhage, neoangiogenesis, and hemosiderin microdeposits.
Incorrect Answers:
Answer 1: Described histology is characteristic of the annulus fibrosus.
Answer 2: Described histology is characteristic of the nucleus pulposus
Answer 3: Described histology is characteristic of osteoid osteoma
Answer 4: Described histology is that of a malignant fibrous histiocytoma, or
pleomorphic soft tissue sarcoma.

(SAE09SN.95) A 56-year-old mechanic has had pain in the hypothenar region of his
dominant right hand for the past 6 months. He reports weakness in his grip and pain is
worse with activity. Which of the following examination findings is most suggestive
of a cervical etiology? Review Topic

1 Relief of symptoms with shoulder abduction (placing hand over the head)
2 Hypothenar atrophy
3 Reproduction of pain with hyperflexion and contralateral rotation of the head
4 Positive Tinel’s sign at the levator scapulae
5 Subluxable ulnar nerve at the cubital tunnel

PREFERRED RESPONSE 1

Hypothenar atrophy is a nonspecific sign that can be seen in ulnar neuropathy, C8


radiculopathy, or even cervical myelopathy; however, the atrophy usually is not
unilateral and includes other muscle groups. The Spurling test is an excellent method
of eliciting cervical radicular pain but involves hyperextension and ipsilateral rotation
of the cervical spine, resulting in nerve root compression by reducing the cross-
sectional area of the ipsilateral neuroforamen. Tinel’s sign at the levator scapulae, if
present, is indicative of an upper cervical (C3 or C4) radiculopathy. A subluxable
ulnar nerve at the cubital tunnel, while often asymptomatic, points toward cubital
tunnel syndrome as an etiology for this patient’s pain. The shoulder abduction relief
(SAR) sign (relief of upper extremity pain with shoulder abduction) is virtually
pathognomic of cervical radiculopathy because this maneuver results in relaxation of
a compressed and/or inflamed cervical nerve root. The SAR sign is the converse
analog of the straight leg raising sign in the lumbar examination for lumbar
radiculopathy, as it relieves tension in the nerve root, thereby relieving symptoms.
(SAE08OS.61) A 45-year-old man sustained the injury shown in Figures 18a through
18c. He is neurologically intact. Which of the following is the most appropriate
treatment? Review Topic

1 A lumbar corset
2 A thoracolumbosacral orthosis (TLSO) and immediate mobilization with repeat
scans in 6 weeks
3 Anterior corpectomy and fusion alone
4 Posterior laminectomy without fusion
5 Posterior fusion with instrumentation

PREFERRED RESPONSE 5

The CT scan reveals a thoracolumbar compression fracture with some height loss and
minimal kyphosis. The T2-weighted MRI scan shows discontinuity of the posterior
interspinous and supraspinous ligaments and ligamentum flavum. The STIR images
demonstrate a discrete region of hyperintensity in the posterior region at the level of
the injury. Thoracolumbar fractures with posterior ligamentous injury are potentially
unstable. A lumbar corset brace would provide insufficient immobilization of this
injury. A laminectomy alone is not indicated and in fact would further destabilize the
injury. Whereas an anterior corpectomy could be performed, it should be
accompanied by rigid instrumentation. If nonsurgical management with a TLSO and
immediate mobilization is elected, serial radiographs should be obtained in the early
post-injury period. Posterior fusion with instrumentation allows mobilization without
fear of secondary displacement.

(OBQ14.27) A 65-year-old woman presents with neck pain for 18 months. She has
taken NSAIDs and undergone physical therapy without improvement. Over the past 6
months, she has also noticed progressive hand clumsiness and difficulty with gait.
Sagittal and axial MRI images are shown in figures A and B, respectively. What is the
most appropriate next step in management? Review Topic
1 Physical therapy with emphasis on neck muscle strengthening and posture
improvement
2 Epidural steroid injection
3 Anterior cervical discectomy and fusion
4 Laminectomy alone
5 Multi-level laminectomy and fusion

PREFERRED RESPONSE 3

This patient presents with clinical and radiographic evidence of cervical myelopathy
with progression of gait imbalance and hand clumsiness. The most appropriate
management is anterior cervical discectomy and fusion (ACDF).

Cervical myelopathy is typically caused compression on the spinal cord and


classically present with neck pain, parasthesias, clumsiness, gait imbalance, and/or
urinary retention. Nonoperative management is reserved for patients without
functional impairment. ACDF is the treatment of choice for focal compression from
anterior disc-ostephyte changes. Posterior decompression is indicated in patients with
multi-level disease. It is important to note in patients with significant kyphosis, the
kyphosis must be reversed in order for decompression alone to be effective.

Hsu reviewed posterior decompression techniques in the cervical spine. They state
that the choice of anterior versus posterior approach is determined based on sagittal
spine alignment, extent and location of pathological involvement, and patient
preference. They recommend laminectomy and fusion for cervical stenosis and
kyphosis <10 degrees. The use of BMP-2 as an adjunct in the cervical spine is not
recommended by the author.

Emery reviewed cervical spondylotic myelopathy and recommend nonoperative


treatment in patients with minimal symptoms without pathologic reflexes or gait
imbalances. They suggest that the preferred posterior techniques are now
laminectomy and fusion or laminoplasty.

Figure A and B are sagittal and axial T2 MRI sequences showing a degenerative disc
osteophyte complex at C5-C6 with resultant canal stenosis and cord compression.
Illustration A is a lateral C spine radiograph in a patient who is status post ACDF at
C5-C6.

Incorrect Answers:
Answers 1 and 2: The patient has neurological impairment in the form on gait
imbalance and clumsiness. Nonoperative treatment is not recommended.
Answer 4: Laminectomy alone is not recommended for decompression. There is a risk
of post-operative kyphosis.
Answer 5: Laminectomy and fusion is recommended in multi-level disease with
kyphosis less than 10 degrees.

(SAE12SN.56) During the application of halo skeletal fixation, the most appropriate
position for the placement of the anterior halo pins is approximately 1 cm above the
superior orbital rim and Review Topic

1 lateral placement, directly within the temporalis muscle.


2 within the lateral third of the superior orbital rim.
3 lateral to the superior orbital rim.
4 medial third of the superior orbital rim.
5 lateral between the temporalis muscle and zygomatic temporal nerve.

PREFERRED RESPONSE 2

Halo fixation is the most rigid form of cervical orthosis but complications can arise
from improper placement of the fixation pins. A relatively safe zone for anterior pin
placement is located 1 cm above and within the lateral third of the superior orbital
rim. This position avoids the supraorbital and supratrochlear nerves over the medial
one third of the orbit. The more lateral positions in the temporal fossa have very thin
bone and can interfere with the muscles of mastication.

(SAE10PE.70) A patient with Pott's disease, tuberculosis of the spine, is more likely
to have which of the following early findings? Review Topic

1 Acute onset back pain and neurologic dysfunction


2 Preservation of the disk space between two affected adjacent end plates
3 Involvement of the cervical spine and torticollis
4 Elevated WBC count and markedly elevated erythrocyte sedimentation rate
5 Lordotic deformity in late stages of the disease
PREFERRED RESPONSE 2

Tuberculosis of the spine typically has an indolent presentation. Unlike pyogenic


infections of the spine, the disk space is usually preserved. Most commonly, the
thoracic and lumbar spine are affected. Laboratory studies may be nonspecific.
Delayed presentation usually results in neurologic compromise and a kyphotic
deformity. Treatment includes a multidrug regimen. Surgery is indicated for
deformity correction or failure of medical treatment.

(SAE12SN.69) A 47-year-old man is seen in consultation in the ICU after being


admitted and treated emergently for a dissecting aortic aneurysm. Current
examination reveals generalized weakness of the lower extremities with a significant
decrease in pain and temperature sensation from approximately the waist down.
Proprioception is maintained. What is the most likely diagnosis at this time? Review
Topic

1 Anterior cord syndrome


2 Central cord syndrome
3 Brown-Sequard syndrome
4 Posterior cord syndrome
5 Spinal shock

PREFERRED RESPONSE 1

Incomplete cord syndromes include anterior cord syndrome, Brown-Sequard


syndrome, central cord syndrome, and posterior cord syndrome. The anterior cord
syndrome involves a variable loss of motor function and pain and/or temperature
sensation, with preservation of proprioception as seen in this patient. The Brown-
Sequard syndrome involves an ipsilateral loss of proprioception and motor function,
with contralateral loss of pain and temperature sensation. The posterior cord
syndrome is a rare injury and is characterized by preservation of motor function, sense
of pain and light touch, with loss of proprioception and temperature sensation below
the level of the lesion. The central cord syndrome is characterized with greater motor
weakness in the upper extremities than in the lower extremities. The pattern of motor
weakness shows greater distal involvement in the affected extremity than proximal
muscle weakness. Spinal shock is the period of time, usually 24 hours, after a spinal
injury characterized by absent reflexes, flaccidity, and loss of sensation below the
level of the injury.
(SAE09SN.83) A 25-year-old man is unresponsive at the scene of a high-speed motor
vehicle accident and remains obtunded. Initial evaluation in the emergency
department reveals a left-sided femoral shaft fracture and a right-sided humeral shaft
fracture. The cervical spine remains immobilized in a semi-rigid cervical collar, and
the initial AP and lateral radiographs obtained in the emergency department are
unremarkable. What is the most appropriate management at this time? Review Topic

1 Lateral radiographs with passive flexion/extension views


2 Helical CT scan of the cervical-thoracic region
3 Careful manual palpation of the cervical spine for subtle defects or step-offs
4 MRI of the cervical spine
5 Continued use of the cervical collar until the patient becomes responsive for
examination

PREFERRED RESPONSE 2

Clearance of the cervical spine can be difficult in the obtunded or unresponsive


patient. Various trauma series have been reported to detect up to 95% of cervical
fractures but only when ideal imaging views have been obtained, which is not often
possible in the unresponsive or uncooperative patient. Passively performed cervical
flexion-extension under live fluoroscopy has been suggested but is not without
inherent risk in the potentially unstable cervical spine. CT of the cervical spine has
gained acceptance for the evaluation of these patients given the excellent evaluation
of the osseous anatomy and for the common availability in most emergency
departments. Sanchez and associates, using a protocol to evaluate for cervical spine
injuries after blunt trauma, were able to detect 99% of cervical fractures with 100%
specificity.

(OBQ13.224) A skeletally mature GMFCS V child with spastic quadriplegic cerebral


palsy presents with progressive scoliosis and inability to sit upright in a wheelchair.
Radiographs are shown in Figures A and B, depicting a long C-shaped 75 degree
curve with pelvic obliquity. Which is the most appropriate treatment option? Review
Topic
1 Bracing and molded wheelchair inserts
2 Anterior release and fusion from T2 to L5
3 Anterior release and fusion from T10 to L3 and posterior fusion from T2 to pelvis
4 Posterior fusion from T2 to pelvis
5 Posterior fusion from T2 to L4

PREFERRED RESPONSE 4

This patient has a large cerebral palsy (CP) scoliotic curve with pelvic obliquity.
Posterior instrumentation should be from T2 to the pelvis.

Scoliosis in CP differs from adolescent idiopathic scoliosis. Curves can be classified


into double curves (thoracic and lumbar) with minimal pelvic obliquity (Weinstein
Group I) and large curves (lumbar/thoracolumbar) with marked pelvic obliquity
(Group II). In the presence of pelvic obliquity (>15°), caudal instrumentation should
end at the pelvis to reduce the risk of pseudoarthrosis and late loss of correction.
Cephalad instrumentation should end high in the thoracic spine (T2) to decrease
proximal junctional kyphosis.

Imrie et al. reviewed the management of spinal deformity in CP. They advocate
posterior-only pedicle screw fusion for curves <100° that bend down to 50% on
traction films, from T2-3 to the pelvis, using iliac screws. They advocate single-stage
anterior release and posterior fusion for curves >120° with poor flexibility, or
associated severe pelvic obliquity or sagittal deformity.

McCarthy et al. reviewed scoliosis in CP. They advocate proximal fixation to the
upper thoracic spine (T1-2), and distal fixation to L4-5 (pelvic obliquity <15°) or to
the pelvis (pelvic obliquity >15°). They add anterior release for larger, rigid curves
that do not bend out to <60°, and in skeletally immature children.

Figures A and B are PA and lateral scoliosis radiographs demonstrating 75° of


thoracolumbar scoliosis and severe pelvic obliquity, and progressive lumbar
hyperlordosis. Illustrations A and B are PA and lateral scoliosis radiographs
demonstrating correction of curve and pelvic obliquity using the Galveston Technique
with segmental fixation utilizing sublaminar wires.

Incorrect Answers:
Answer 1: Bracing is generally less effective for treatment of scoliosis in children
with CP and is used to improve sitting balance or slow curve progression in skeletally
immature children. Similarly, wheelchair inserts are used to improve sitting balance.
For progressive curves >40-50° that interfere with sitting in patients >10yrs, surgery
is recommended.
Answers 2 and 3: Anterior release and fusion is indicated for larger curves (>100-
120°) that are more rigid (do not bend out to <60°) and in skeletally immature
children (to prevent crankshaft phenomenon) and is not indicated in this case as the
curve can be corrected by a posterior approach alone. Anterior release and fusion is
not used independent of posterior fusion.
Answer 5: Posterior fusion to L4 or L5 is less desirable in this patient because of the
degree of pelvic obliquity (>15°). Patients with pelvic obliquity >15° that do not
undergo pelvic fixation tend to have good initial correction of obliquity, with
progressive loss of correction with time, and increased risk of pseudoarthrosis.

(SBQ12SP.26) Figure A is a lateral cervical spine radiograph of a 70-year-old female


who presents with two months of posterior midline neck pain that is worse with
motion. She has no history of trauma. She denies any symptoms of arm pain, gait
instability, or dexterity problems with her hands. Physical exam of the lower
extremities shows 2+ patellar reflexes and flexion of the great toe with a Babinski
test. What should the patient be told regarding these radiographic findings? Review
Topic

1 She requires surgical decompression to prevent progressive neurologic deterioration


2 85% of individuals over the age of 60 years of age demonstrate these findings
3 She is indicated for MRI to rule out cervical radiculopathy
4 She would benefit from modalities such as heat, cold, and transcutaneous electrical
stimulation
5 She has <10% chance of having symptomatic improvement with non-operative
treatment
PREFERRED RESPONSE 2

The patient has symptomatic cervical spondylosis. She should be told that 85% of
asymptomatic individuals over the age of 60 demonstrate these radiographic findings.

Cervical spondylosis is defined as age-related degenerative changes within the


cervical spinal column. It can present as axial neck pain, cervical radiculopathy,
cervical myelopathy, or as a combination of each. Radiographs demonstrate loss of
disc height, arthrosis of the facet and uncovertebral joints, and endplate sclerosis.
Imaging such as cervical spine radiographs and MRI should be interpreted carefully
and correlated with clinical symptoms, as radiographic evidence of spondylosis is
frequently seen in asymptomatic patients.

Boden et. al. reviewed the cervical MRI of 63 asymptomatic volunteers with no
history of cervical spine symptoms. Of those, 19% were interpreted as having an
abnormality. These findings included herniated nucleus pulposus, bulging disc, and
foraminal stenosis. 60% of patients over 40 years demonstrated disc degeneration or
narrowing at one level.

Rao et al. reviewed the presentation, pathogenesis, and management of cervical


spondylosis. In this review, they report that 85% of patients older than 60 years
demonstrate cervical spondylosis on radiography.

Figure A is a lateral cervical spine radiograph demonstrating spondylosis at multiple


levels. There is disc height loss, facet sclerosis, and anterior osteophyte formation.

Incorrect Answers:
Answer 1: The patient has two weeks of axial neck pain with no neurologic deficits.
She is unlikely to develop progressive neurologic loss with these findings.
Answer 3: Cervical radiculopathy is a clinical rather than a radiographic diagnosis
Answer 4: Rao et al. article reports no proven benefit to these alternative modalities
Answer 5: Rao et al. article reports complete resolution of symptoms in 43% of
patients treated non-operatively

(SAE12SN.38) A 29-year-old woman is seen in the emergency department with a 24-


hour history of severe back and leg pain precipitated by weight-lifting. The patient
reports bilateral leg pain and is unable to urinate. She has dense anesthesia in the
perineal region on examination. A MRI scan is shown in Figure 38. The patient is
taken to surgery urgently. What is her prognosis for recovery? Review Topic
1 Improvement in her pain and sensory symptoms following surgery but may have
residual bladder dysfunction
2 Decreased pain following surgery; sensory deficits and bladder function are not
likely to improve
3 No change in her symptoms following surgery
4 Complete resolution of pain and will have normal sensation and bladder function
following surgery
5 Improvement in her pain and complete return of bladder function following surgery;
sensation may not return

PREFERRED RESPONSE 1

The patient with cauda equina syndrome should be taken to surgery urgently to
provide the best chance of symptom resolution. However, many studies indicate that
patients with cauda equina syndrome do not return to a completely normal status even
following urgent surgery. Whereas pain is typically relieved after surgery, other
deficits, especially bladder and sexual dysfunction, may persist. Particularly in light of
the patient's severe saddle anesthesia, she may have a poor prognosis for recovery of
normal bladder function.

(SAE09SN.29) Up to what time frame are the risks minimized in anterior revision
disk replacement surgery? Review Topic

1 3 days
2 1 week
3 10 days
4 2 weeks
5 6 weeks

PREFERRED RESPONSE 4
Revision anterior exposure within 2 weeks of total disk replacement incurs relatively
little additional morbidity because adhesion formation is minimal. Surgeons should
have a low threshold for revising implants that are clearly dangerously malpositioned
or show early migration within this 2-week window. Beyond this time period, a
revision strategy must be individualized to the particular clinical situation. A posterior
fusion with instrumentation with or without a laminectomy is currently the most
effective salvage procedure.

(SAE08OS.93) Figures 30a through 30c show the CT scans and standing lateral
radiograph of a 30-year-old woman who was injured in a fall while snowboarding.
She is neurologically intact in the lower extremities and has normal rectal tone and
sensation. She has tenderness at the thoracolumbar junction but no ecchymosis or
palpable defect. Mobilization in a thoracolumbosacral orthosis for this patient has
been proven to result in which of the following clinical outcomes? Review Topic

1 20% to 25% chance of neurologic deterioration


2 Significant risk of late symptoms of spinal stenosis
3 Higher complication rate than surgical treatment
4 Equivalent clinical outcome compared to surgical treatment
5 Clinical outcome that depends more on the status of the adjacent disks

PREFERRED RESPONSE 4

Surgical and nonsurgical treatment have been shown to be equivalent in terms of


clinical outcome in a prospective randomized trial of stable burst fractures. There is
relatively limited risk of neurologic injury with careful mobilization in appropriate
bracing of patients with this injury pattern. There is only infrequent symptomatic late
stenosis, with many patients experiencing resorption of the intra-canal fragments. The
status of the adjacent disks has not been shown to be a prognostically important
factor. The complication rates of surgical and nonsurgical treatments appear to be
similar if not less for nonsurgical treatment.
(SAE09SN.85) A 56-year-old woman sustained the fracture shown in Figures 30a and
30b in a motor vehicle accident. What mechanism is most likely responsible for the
injury? Review Topic

1 Flexion distraction
2 Vertical shear
3 Extension distraction
4 Flexion compression
5 Axial load

PREFERRED RESPONSE 5

The CT scans show a burst fracture that results from an axial load injury. The
radiographic hallmark of a burst fracture is compression of the posterior cortex of the
vertebral body with retropulsion of bone into the spinal canal. AP radiographs often
show widening of the interpedicular distance with a fracture of the lamina.

(OBQ13.162) A 48-year-old man is brought in by emergency services after falling


down a flight of stairs. He complains of weakness in both hands. Examination reveals
weak grip bilaterally. Injury CT scans are shown in Figure A. What is the most
appropriate treatment option? Review Topic
1 Hard cervical orthosis
2 Immobilization in cervico-thoraco-lumbo-sacral orthosis
3 Anterior decompression and fusion of C4-C7
4 Posterior decompression and fusion of C5-C6
5 Posterior decompression and fusion of C3-T2

PREFERRED RESPONSE 5

This patient has ankylosing spondylitis (AS) and has suffered a shear fracture through
C5-6. Due to the presence of neurological deficits, posterior decompression and
fusion with a long construct (such as from C3-T2) is indicated.

The C-spine is the most common site of fracture in AS and is most susceptible to
hyperextension injuries. When surgical intervention is required, multiple points of
fixation both above and below the fracture are necessary. This is due to co-existing
osteoporosis and abnormally increased forces from long lever arms of the ankylosed
spine, both of which make the construct susceptible to failure and screw pullout.

Kubiak et al. reviewed the orthopaedic management of AS. They report bone scan,
MRI or fine-cut CT is necessary because fractures are often missed on plain x-rays
because of distortion of anatomy or difficulty with positioning.

Whang et al. reviewed spinal injuries in 12 patients with AS and 18 patients with
DISH. Most injuries involved C5-C7. Patients with AS were more likley to have
severe neurologic injury (41% ASIA A) than DISH (44% ASIA E). There was 81%
good-excellent outcome and 4 deaths related to halo vest use.

Figure A is a sagittal CT reconstructed image showing a nondisplaced shear fracture


through the C6 vertebral body and C5 posterior elements. Illustrations A and B are
postop AP and lateral radiographs showing posterior decompression and C3-T2 fusion
with lateral mass fixation in the cervical spine and pedicle screw fixation in the upper
thoracic spine.

Incorrect Answers:
Answers 1 and 2: This is a highly unstable (3-column) injury. Surgical stabilization is
necessary to prevent further neurologic injury.
Answers 2,3,4: Short segment decompression and fusion is less preferred because of
the long lever arms and likelihood of construct failure. Long constructs are best
performed through a posterior approach. If there is significant osteoporosis and the
risk of construct failure is high, a 360-approach may be necessary.

(OBQ15.7) A 42-year-old female with chronic discogenic back pain undergoes


lumbar spine surgery via retroperitoneal approach. Figure A is the postoperative
radiograph of her lumbar spine. Six weeks after the surgery the patient develops
worsening low back pain. You obtain a radiograph of her lumbar spine, pictured in
Figure B. What is the next best step in management? Review Topic

1 Facet joint injection and lumbosacral orthosis


2 Posterior stabilization
3 Revision arthroplasty via far lateral approach
4 Revision to arthrodesis via far lateral approach
5 Revision to arthrodesis via retroperitoneal approach

PREFERRED RESPONSE 5

This patient has a failed lumbar total disc replacement (TDR) with anterior dislocation
of the polyethylene inlay and requires revision surgery, either revision arthroplasty or
anterior interbody fusion, via retroperitoneal approach.

Management of failed TDR is dictated by (1) patient symptoms and (2) radiographic
implant position. Asymptomatic patients with implant subsidence without extrusion
can be managed with close observation. Patients with persistent symptomatic back
pain relating to facet joint or implant microinstability in the setting of an otherwise
well-positioned TDR can be treated with posterior stabilization alone. Symptomatic
patients with unacceptable implant position (i.e. complete dislodgement, fracture)
require either revision arthroplasty or conversion to arthrodesis (anterior +/-
posterior).

Harrison et al. authored a report detailing a case of failed L5-S1 TDR with anterior
dislocation of the polyethylene inlay and bilateral L5 pars fractures noted 2 weeks
postoperatively. It is unclear whether the pars defect was missed following previous
microdiscectomy, acquired during instrumentation, or acquired postoperatively due to
abnormal endplate forces during rehabilitation. The patient underwent removal of
TDR components and revision to anterior arthrodesis followed by percutaneous
fixation of the bilateral L5 pars fractures.

Patel et al. published a review of the literature regarding revision lumbar TDR.
Retroperitoneal revision procedures are complicated by higher risk of injury to
vascular structures (16.7% versus 3.6% at primary surgery) and urogenital structures
due to adhesions. Revision surgery should only be performed by surgeons familiar
with the retroperitoneal approach to the spine. Alternative surgical approaches, such
as the far lateral approach, should be considered to access the anterior spine at L4-L5
and above.

Madigan et al. reviewed the management of lumbar degenerative disc disease (DDD).
They state that TDR is a motion-sparing surgical option designed to temper the
problem of adjacent-segment disease. They report that long-term studies are limited,
but point out that several short-term comparisons between TDR and ALIF show
equivalent improvement in pain and function, equal complication rate, shorter
hospitalization and lower rate of reoperation in the TDR group.

Figure A is a lateral radiograph of the lumbar spine showing L5-S1 total disc
replacement. Figure B is a lateral radiograph of the lumbar spine showing L5-S1 total
disc replacement with anterior dislocation of the polyethylene inlay.

Incorrect Responses:
Answer 1: Facet joint injection and LSO brace will not address the failed TDR, which
is the cause of her pain.
Answer 2: Posterior stabilization alone is not sufficient to address the failed TDR with
anterior polyethylene dislocation.
Answer 3 + 4: The far lateral approach is not feasible for access to the L5-S1 level
due to the iliac crests.

(SAE09SN.88) What is the typical axial plane transverse angulation of the thoracic
pedicles? Review Topic

1 5 degrees medial at T1 and T2; 10 degrees from T3 to T10


2 5 degrees lateral at T1; neutral at T2; 5 degrees medial from T3 to T12
3 10 degrees medial from T1 to T10; 15 degrees medial at T11 and T12
4 10 degrees medial from T1 to T12
5 25 degrees medial at T1; 15 degrees at T2; and 10 degrees medial from T3 to T10

PREFERRED RESPONSE 5

Thoracic pedicles typically are angled 25 degrees medially at T1 so the starting point
is more lateral. T2 angles about 15 degrees, and then the pedicles average about 5 to 7
degrees down to T10. At T11 and 12, the angulation is minimal.

(SAE09SN.3) Which of the following is a contraindication to laminoplasty in a


patient with cervical spondylotic myelopathy? Review Topic

1 Space available for the cord of less than 8 mm


2 Ossification of the posterior longitudinal ligament
3 Fixed cervical kyphosis
4 Previous posterior surgery
5 Concomitant cervical radiculopathy

PREFERRED RESPONSE 3

Laminoplasty or any posterior decompressive procedure is contraindicated in patients


with cervical spondylotic myelopathy and cervical kyphosis. The residual kyphotic
posture of the cervical spine results in persistent spinal cord compression. The other
choices are not contraindications for laminoplasty. Concomitant cervical
radiculopathy can be addressed at the time of laminoplasty with a keyhole
foraminotomy.

(SAE12SN.58) An 80-year-old man who was involved in a fall from ground height is
evaluated in the emergency department for head lacerations and mild neck pain.
Examination reveals only mild tenderness of the posterior neck region with some
limitation of motion. Neurologic examination is normal. Radiographs of the cervical
spine are shown in Figures 58a and 58b. What is the next most appropriate step in
management for this patient? Review Topic
1 Admission for 24-hour observation for possible intracranial hematoma
2 Discharge with a soft collar and follow-up in 24 hours
3 CT of the cervical spine
4 Repeat radiographs of the cervical spine, including flexion and extension views
5 Hard cervical collar for 6 weeks

PREFERRED RESPONSE 3

The patient has radiographic findings compatible with diffuse idiopathic skeletal
hyperostosis (DISH) of the cervical spine. Characteristics of DISH include flowing,
non-marginal osteophytes at four or more levels. Patients with DISH develop a
significant loss of flexibility of the spine. The spine acts more as a long bone with
minimal force needed to create unstable fractures. Any minor trauma in patients with
DISH should be worked up aggressively to rule out occult fracture. In this patient,
radiographs fail to clearly rule out a fracture; therefore, CT of the cervical spine is
indicated. Without a suspicion of history of a head injury, admission specifically for a
possible intracranial hematoma is not warranted. The more concerning injury in a
patient with DISH is occult neck fracture. Treatment with a soft or hard collar is not
advised until a fracture is ruled out. Repeat radiographs are unlikely to show any
occult fractures, and flexion and extension views would not be advised in a patient
with a suspected vertebral fracture.

(SAE07PE.72) Figures 31a and 31b show the radiograph and MRI scan of an
otherwise normal 3-month-old infant who has a spinal deformity. MRI reveals no
intraspinal anomalies. What is the next step in management? Review Topic
1 Posterior spinal fusion with instrumentation
2 Anterior-posterior hemiepiphysiodesis
3 Brace management
4 Cardiac and renal evaluation
5 Hemivertebrectomy and fusion

PREFERRED RESPONSE 4

Congenital scoliosis in an infant warrants evaluation of the renal, cardiac, and


neurologic systems because frequently there is concurrent pathology. Progression in
this instance is possible but not certain; therefore, progression must be documented
prior to any surgical intervention. Close observation with serial radiographs every 4 to
6 months is appropriate. All of the surgical options listed may be reasonable choices
in the future, but cardiac evaluation is the most important issue at this time.

(SAE09SN.73) Which of the following is a true statement regarding thoracic disk


herniations? Review Topic

1 Are most commonly discovered during the fifth to seventh decades of life
2 Occur with similar frequency as cervical disk herniations
3 Occur most commonly in the midthoracic or apical region of the spine
4 Can be found in 40% of asymptomatic individuals
5 Are best treated surgically with posterior laminectomy and excision

PREFERRED RESPONSE 4

Symptomatic herniations of the thoracic spine are much less common than those of
the cervical or lumbar region. They tend to occur most commonly during the third to
fifth decades of life and although they can be found at all levels, they are most
common in the lower third near the thoracolumbar region. Posterior laminectomy and
disk excision has the highest rate of neurologic deterioration and is not recommended.
Multiple studies have shown that herniated thoracic disks can be found at one or more
levels in 40% of asymptomatic individuals.

(SAE10PE.82) In infantile idiopathic scoliosis, which of the following factors


suggests progression? Review Topic

1 Age at presentation
2 Rib overlap of the apical vertebra
3 Rib vertebral angle difference of greater then 10 degrees
4 Male gender
5 Family history

PREFERRED RESPONSE 2

Infantile idiopathic scoliosis occurs more commonly in boys, with a 3 to 1 male to


female ratio. Neural axis abnormalities, hip dysplasia, and congenital heart disease are
all associated with the condition; spontaneous correction frequently occurs. Curve
progression can be predicted by the rib vertebral angle difference or the phase of the
rib head. Rib overlap of the apical vertebral body or a rib vertebral angle difference of
greater than 20 degrees indicates that the curve is likely to progress. Gender, family
history, and age at presentation have not been found to be risk factors for progression.

(SAE07PE.90) Examination of a 13-year-old boy with asymptomatic poor posture


reveals increased thoracic kyphosis that is fairly rigid and accentuates during forward
bending. The neurologic examination is normal. Spinal radiographs show 10 degrees
of scoliosis at Risser stage 2, and there is no evidence of spondylolisthesis. A standing
lateral view of the thoracic spine is shown in Figure 41. The kyphosis corrects to 50
degrees. Management should consist of Review Topic
1 electrical stimulation.
2 a Charleston bending brace at night.
3 an extension-type spinal orthosis.
4 posterior spinal fusion with instrumentation.
5 anterior spinal release and posterior spinal instrumentation.

PREFERRED RESPONSE 3

The radiograph shows excessive thoracic kyphosis (normal 20 degrees to 50 degrees)


with multiple contiguous vertebral wedging and end plate irregularity, all consistent
with the diagnosis of Scheuermann’s kyphosis. The patient is skeletally immature;
therefore, there is the potential for progression of the kyphotic deformity. Extension
bracing has shown efficacy in the treatment of Scheuermann’s kyphosis that measures
50 degrees to 74 degrees, and has actually reduced the curvature permanently in some
patients. A thoracolumbosacral orthosis may be used if the apex of kyphosis is at T7
or lower. Indications for surgical treatment are controversial, but spinal fusion most
likely should not be considered for a painless kyphosis measuring less than 75
degrees.

(SAE09SN.82) Surgical treatment for symptomatic disk herniations is associated with


which of the following? Review Topic

1 Substantial rate of nerve root injury


2 Early relief of pain sustained out to 2 years
3 Recurrent herniation rate of 35%
4 Outcomes that are substantially worse than nonsurgical management
5 10% rate of infectious diskitis
PREFERRED RESPONSE 2

The recently published SPORT trial verifies that surgical treatment of symptomatic
disk herniations is associated with early and sustained pain relief. The trial also
verifies that nonsurgical management is associated with improved symptoms as well.
Nerve root injury, recurrent herniation, and diskitis are known complications of
surgery, but all are less common than described above.

(OBQ14.249) An obese (BMI = 35) 72-year-old woman with diabetes mellitus,


hyptertension and a 22-pack-year smoking history is scheduled to undergo posterior
spinal fusion from T10 to S1 with a pedicle subtraction osteotomy at L3 for the spinal
deformity seen in Figure 1. Which of the following risk factors is most predictive of
major complication following surgery Review Topic

1 Age > 60 years


2 > or = 2 comorbid conditions
3 History of tobacco use
4 BMI > 30 kg/m²
5 Using a pedicle subtraction osteotomy

PREFERRED RESPONSE 1

The patients age (> 60 years) is the most significant risk factor for a major
perioperative complication during posterior spinal fusion for adult spinal deformity
correction.

The surgical treatment of adult spinal deformity often requires multilevel arthrodesis
with complex osteotomies including three column osteotomies such as pedicle
subtraction (PSO) and vertebral column resection (VCR). They can involve both
anterior and posterior surgical approaches. Surgical time, blood loss, length of
hospital stay, and length of recovery can be greater than it is for the more common
degenerative conditions.

Auerbach et al. characterized the risk factors for the development of major
complications in patients undergoing 3-column osteotomies for adult spinal deformity
correction. They also aimed to determine whether the presence of complications
affected the ultimate clinical outcome. They found age > 60 years, > or = 3 comorbid
conditions and preoperative sagittal imbalance of = 40mm was associated with a
major complication. However, the presence of a major complication did not affect the
ultimate clinical outcomes at 2 years or more.

Daubs et al. conducted a retrospective analysis of forty-six patients = 60 years of age


who underwent major spinal deformity surgery requiring a minimum 5-level
arthrodesis procedure to determine the rate of complication and outcomes. The overall
complication rate was 37% and the major complication rate was 20%. Increasing age
was a significant factor (p<0.05) in predicting the presence of a complication, while
presence of comorbidities was found to have no association.

Figure A is a standing preoperative lateral radiograph of the spine demonstrating a


thoracic kyphosis of ~25° and thoracolumbar kyphosis of ~25°. Illustration A
demonstrates proper sagittal balance after spinal fusion from T10 to S1 and L3 PSO.

Incorrect Answers:
Answer 2: There is no consensus in the literature regarding the impact of
comorbidities on major perioperative complications.
Answer 3, 4 and 5: A history of tobacco use, a BMI > or = 30 kg/m², and osteotomy
type (PSO or VCR) were not found to have an association with the development of
major complications.

(SAE12SN.53) A 66-year-old woman reports chronic mild low back pain. Over the
last 3 years, she has noticed worsening buttock and posterior leg pain with standing
and walking. Sitting seems to improve the pain. She also reports numbness in both
legs with walking. An MRI scan and standing radiographs of the lumbar spine are
shown in Figures 53a through 53c. She has undergone two epidural injections with
good, but short-term relief. Further treatment to alleviate this patient's symptoms
should consist of which of the following? Review Topic
1 Manual therapy
2 Multilevel laminectomy
3 Facet injections
4 L4-5 laminotomy
5 Laminectomy and fusion

PREFERRED RESPONSE 2

The patient reports symptoms that are classic for neurogenic claudication secondary
to lumbar spinal stenosis. Nonsurgical management has failed to provide relief, thus a
surgical approach is a reasonable treatment option at this point. Studies have shown
significant benefit in patients with lumbar stenosis who choose to undergo surgical
treatment. Manual or manipulative therapy is unlikely to provide relief. Facet
injections are not effective for neurologic symptoms. An L4-5 laminotomy will not
adequately address the patient's pathology. In the absence of instability on imaging
studies, arthrodesis is not indicated.

(SAE08OS.136) Figures 45a and 45b show the CT scans of a 41-year-old man who
was ejected from a vehicle after a high-speed collision. Examination reveals weak
(2/5) toe flexion on the right side, but otherwise he has no voluntary motor function in
his lower extremities and no sensation. He has some perianal sensation. He has no
other associated injuries and his upper extremities are neurologically intact. Which of
the following represents the best treatment option? Review Topic
1 Posterior stabilization and fusion only
2 Anterior stabilization and fusion only
3 Anterior decompression and stabilization
4 Nonsurgical treatment with mobilization in a thoracolumbosacral brace
5 Multilevel laminectomy

PREFERRED RESPONSE 3

The CT scans show a burst fracture of L1. In distinction from a fracture-dislocation,


the levels above and below the injury (T12 and L2) are reasonably well-aligned,
although there is a marked amount of kyphosis. The axial image displays retropulsed
bone fragments from the posterior vertebral body that are causing nearly 100% canal
compromise. As the patient has an incomplete neurologic injury (evidenced by some
motor [toe flexion, S2] and sensory [perianal sensation, S3-5] function below the level
of the injury), a decompressive procedure should be strongly considered. Studies have
demonstrated that an anterior corpectomy achieves the most complete spinal canal
decompression after a burst fracture, and is best performed in concert with a
stabilization procedure. Posterior stabilization and fusion without laminectomy could
restore alignment and provide stability, but it is unlikely to provide adequate spinal
canal decompression. Anterior stabilization and fusion, without decompression,
would not maximize neurologic recovery. Nonsurgical treatment in a patient with an
incomplete neurologic injury should be considered only for those who are medically
unfit for surgery. Multilevel laminectomy will cause further destabilization of the
spine and will not adequately decompress the spinal canal because the dural sac is
being compressed along its anterior aspect by retropulsed bone fragments.

(SBQ12SP.25) A 63-year-old female undergoes lumbar decompression and


instrumented fusion for the condition shown in Figure A and B. Which of the
following factors are associated with increased risk of early post-operative
complications? Review Topic
1 Increasing Age
2 Intraoperative blood loss
3 Number of levels fused
4 All of the above
5 Answers 2 and 3 only

PREFERRED RESPONSE 4

Age, intraoperative blood loss, and number of spinal levels fused are associated with
increased rates of inpatient complication following lumbar decompression and fusion
for degenerative spondylolisthesis. Older age correlates with the highest risk for
complications.

Lumbar decompression and fusion is well supported in the literature to be effective in


the treatment of degenerative spondylolithesis when nonoperative modalities fail.
Common complications associated with lumbar decompression and fusion include
asymptomatic or symptomatic pseudarthroses (estimated 5-30%), surgical site
infection (0.10-2%),and adjacent level degeneration (2-3%). Age, intraoperative blood
loss, longer operative times, number of levels fused are associated with increased risk
of complication. Smoking is associated with worse outcome after surgery.

Kalanithi et al. reviewed the Nationwide Inpatient Sample adminstrative data on sixty
six thousand patients undergoing lumbar laminectomy and fusion. They found the
increased age and having 3 or more medical comorbidities was associated with
increased risk of complication.

Carreon et al. reviewed cases of ninety eight patients who underwent laminectomy
and fusion for lumbar degenerative disease. They found that rates of major and minor
complication increased with older age, increased blood loss, longer operative time,
number of levels fused.

Figure A is a AP lumbar spine radiograph. Figure B is a lateral lumbar spine


radiograph showing degenerative spondylolisthesis at L4-L5. Illustration A shows the
postoperative images of the patient treated with an L4-5 fusion with an interbody
device.
Incorrect Answers:
Answer 1, 2, 3, 4: All of the answers presented in 1, 2, and 3 are correct.

(SAE09SN.1) Which is the best initial study for the diagnostic evaluation of
diskogenic low back pain? Review Topic

1 MRI
2 Diskography
3 CT-diskography
4 Radiography
5 CT

PREFERRED RESPONSE 4

Radiography is the best initial study for the evaluation of diskogenic low back pain.
The normal degenerative process can be evaluated. Vacuum phenomenon may be
found within the disk space. Other possible sources for back pain should also be
evaluated. The other tests may be beneficial but represent later imaging options.

(SAE09SN.63) Which of the following is considered the most effective means of


identifying an evolving motor tract injury during cervical spine surgery? Review
Topic

1 Mean arterial blood pressure monitoring


2 SSEP monitoring
3 Free-run electromyography
4 Transcranial motor monitoring (tceMEP)
5 Wake-up test

PREFERRED RESPONSE 4

In a study of 427 patients undergoing cervical spine surgery, 12 patients demonstrated


substantial or complete loss of amplitude of the tceMEPs. Ten of those patients had
complete reversal of the loss following prompt intraoperative intervention. SSEP
monitoring failed to identify any changes in one of the two patients that awoke with a
new motor deficit. SSEP changes lagged behind the tceMEP changes in patients in
which major changes were detected by both modalities. TceMEP monitoring was
100% sensitive and 100% specific. SSEP monitoring was only 25% sensitive and
100% specific.

(SAE12SN.41) A positive straight leg raising sign is when pain radiates distal to the
knee when the affected leg is passively elevated. The increased tension generally
occurs between Review Topic

1 0 degrees to 30 degrees.
2 30 degrees to 70 degrees.
3 45 degrees to 90 degrees.
4 60 degrees to 90 degrees.
5 90 degrees.

PREFERRED RESPONSE 2

A positive straight leg raising sign causes pain radiating distal to the knee when the
nerve is under tension. This increased tension generally occurs between 30 degrees
and 70 degrees of passive elevation.

(SAE10BS.44) Risks associated with vertebral compression fractures in the elderly


population can be defined by which of the following? Review Topic

1 No differences in mortality risk were found between cases and controls.


2 Greater mortality risk was found for men than women with vertebral compression
fractures.
3 Lifetime mortality risk was lower when the fracture occurred at a younger age.
4 Mortality within the case or control groups was not affected by duration of follow
up.
5 Risk of additional fractures was not affected by vertebral fracture status.

PREFERRED RESPONSE 2

In the US, about 1.5 million fractures occur annually. Each year, 250,000 new
vertebral compression fractures are diagnosed, and more than 80% of these are related
to weakened vertebral bodies secondary to osteoporosis. Patients with vertebral
fractures are more likely to fall and are five times more likely to sustain subsequent
fractures than individuals without such a fracture. Analysis of data from Medicare
patients with a vertebral fracture had an overall mortality rate that was approximately
twice that of the matched controls. The survival rates following a fracture diagnosis,
as estimated with the Kaplan-Meier method, were 53.9%, 30.9%, and 10.5% at 3, 5,
and 7 years, respectively which were significantly lower than the rates for the
controls. The mortality risk following a fracture was greater for men than for women.
The mortality risk was greater when the fracture occurred at a younger age. The
adjusted hazard ratio was 1.8 compared to the non-fracture group

(SAE08OS.127) A 62-year-old woman has loss of function of her left shoulder and
dull pain. She denies any history of trauma, and there were no previous surgical
procedures. MRI scans are shown in Figures 42a and 42b. The etiology of this
pathology is most likely confirmed by Review Topic

1 genetic screening.
2 routine serum electrolytes and CBC count.
3 whole body bone scan.
4 MRI of the brain and cervical spine.
5 CT scan of the abdomen and pelvis.

PREFERRED RESPONSE 4

Neuropathic arthropathy of the shoulder is an uncommon diagnosis that often presents


on radiographs with advanced degenerative findings on both sides of the joint, with
relatively minor pain complaints. Function is often significantly compromised due to
the loss of conforming joint surfaces. MRI of the brain and cervical spine will often
reveal a type 2 Arnold-Chiari malformation, a syrinx of the cervical spinal cord, or
both. Recognition of this as a neuropathic problem is imperative to avoid
inappropriate surgical management of the shoulder with rotator cuff repairs or
primary arthroplasty.
(SAE12SN.16) A 56-year-old man has had a 2-year history of slowly progressive
neck pain and bilateral arm aching. Over the past year, he has noticed intermittent,
diffuse numbness in both hands, with decreased grip strength and mild hand
clumsiness. He denies any problems with balance. Examination shows a wide-based
gait, intrinsic wasting, and a postive Hoffman's sign bilaterally. An MRI scan of the
cervical spine is shown in Figure 16. What is the most appropriate treatment? Review
Topic

1 Anterior diskectomy without fusion at C4-C5


2 Epidural injections
3 Anterior diskectomy and fusion at C4-C5 and C5-C6
4 Multilevel laminectomy and fusion
5 Multilevel posterior foraminotomies

PREFERRED RESPONSE 4

The patient has classic symptoms of myelopathy with upper motor neuron signs on
examination. His symptoms have been present for years, and are getting worse. The
cervical spine MRI scan shows spinal stenosis with multilevel spondylosis causing
spinal cord compression at multiple levels. With the longstanding duration of the
patient's signs and symptoms, combined with involvement of multiple levels in the
cervical spine, posterior multilevel laminectomy and fusion is the best treatment
option. Two-level anterior diskectomy and fusion would address the two areas of
most severe narrowing, but it would fail to decompress the other stenotic areas which
also require decompression. Posterior cervical foraminotomies would only address
radicular symptoms, which are not present in this patient, and would not succeed in
decompression of the spinal cord. Cervical epidural injections are not indicated for
myelopathy symptoms, and may in fact place this patient at risk for neurologic
deterioration.
(OBQ14.163) Central cord syndrome would most likely be seen in which of the
following patients? Review Topic

1 67-year-old male following hyperextension cervical injury


2 58-year-old male following penetrating trauma to the neck
3 23-year-old male following hyperflexion cervical injury
4 71-year-old male following surgical aortic aneurysm repair
5 19-year-old male without a traumatic mechanism

PREFERRED RESPONSE 1

Central cord syndrome is an incomplete spinal cord injury most commonly seen in
older patients following hyperextension cervical injury.

Central cord syndrome is most commonly caused by cervical hyperextension in older


patients with underlying cervical spondylosis. It is thought to be due to compression
between anterior osteophytes and posterior infolded ligamentum flavum during
hyperextension. This preferentially affects the motor tracks closest to midline, and
thus motor function is impaired greater than sensation, and upper extremities more
affected than lower extremities (remember upper motor tracks are more central in the
lateral corticospinal tract)

Nowak et al. present a review article on central cord syndrome. They cite the most
common cause of central cord syndrome as a hyperextension mechanism in an elderly
patient with preexisting spondylosis.

Gupta et al. present a review article on the management of spinal cord injuries. They
discuss advances and ongoing study in acute management, surgical techniques,
pharmacoptherapies, and cellular transplantation. They recommend a multi-
disciplinary approach to treating spinal cord injuries.

Dvorak et al. present results of long term follow-up after traumatic central cord
syndrome. They found average improvement of ASIA motor scores (AMS) from 58.7
to 92.3. Bowel and bladder control returned in 81% and independent ambulation
returned in 86%. Variables correlated with final AMS were AMS at injury, formal
education, and presence of spasticity during follow-up.

Illustration A shows the blood supply to the spinal cord. Illustration B shows the area
affected by central cord syndrome to help explain why it produces the described
deficits.

Incorrect answers:
Answer 2: Penetrating trauma to the neck is more likely to cause a Brown-Sequard
Syndrome
Answer 3: Hyperflexion injury in a younger patient leading to central cord syndrome
may be seen in the presence of congenital cervical stenosis. However, this is not the
most common cause.
Answer 4: Aortic aneurysm repair would more commonly lead to anterior cord
syndrome.
Answer 5: Central cord syndrome is not typically seen in a young patient without any
other cause.

(SAE13PE.96) What is a known risk factor for wound infection after spinal fusion for
neuromuscular scoliosis? Review Topic

1 Number of levels fused


2 Blood loss
3 Serum albumin level <3.5 g/dL
4 Preoperative curve magnitude

PREFERRED RESPONSE 3

The risk for wound infection after spinal fusion for neuromuscular scoliosis ranges
from 4% to 14% and is higher than risk after spinal fusion in idiopathic scoliosis. A
recent study of a database of 151 patients with neuromuscular scoliosis found the
presence of ventriculoperitoneal shunt to be associated with an increased risk for
wound infection. Age, preoperative major curve magnitude, number of vertebral
levels fused, length of surgery, blood loss, and transfusion requirements were not
associated with increased risk. A previous study found that poor nutritional status as
measured by serum albumin <3.5 g/dL (reference range, 3.5-5.0 g/dL) or lymphocytes
<1500 cells/µL (reference range, 1000-4800/µL) has been associated with increased
postoperative wound infections.

(SAE11AN.53) Which of the following best characterizes the injury shown in Figure
53? Review Topic
1 Stable tear drop extension injury
2 Facet dislocation
3 Unstable flexion compression injury
4 Stable axial load injury
5 Burst fracture

PREFERRED RESPONSE 3

The injury shown is a flexion compression injury also known as "tear drop" fracture.
It is characterized by the large anteroinferior fragment off the vertebral body and the
retrolisthesis seen in this image. It is considered an unstable injury and should be
distinguished from the more stable and minor extension tear drop avulsion where
there is no vertebral malalignment and the anteroinferior fracture is a small avulsion
of the annulus attachment. Other axial load injuries can be stable but have more of a
compression or even burst pattern with loss of body height rather than the
anteroinferior fragment. The radiograph does not demonstrate facet malalignment that
would be seen with a facet dislocation.

(OBQ15.180) A 69-year-old male presents with acute on chronic neck pain. He denies
trauma, fevers, or chills. He has noticed some clumsiness in his hands recently and
change in his handwriting. He has had progressive deterioration of his gait. Physical
exam shows a positive grip and release test, a positive Hofmann sign, and 3+ patellar
reflexes. MRI image is shown in Figure A. The procedure shown in Figure B is
performed. Which of the following statements is true regarding this treatment option?
Review Topic
1 This procedure is contra-indicated in patients with an ossified posterior longtudinal
ligament.
2 Significant loss of motion can be expected due to fusion of the facet joints
3 The spinal cord is decompressed by increasing the medial to lateral dimension of the
spinal canal
4 The spinal cord is decompressed by increasing the anterior to posterior dimension of
the spinal canal
5 A unilateral approach allows less blood loss than a laminectomy and instrumented
fusion

PREFERRED RESPONSE 4

Figure B shows a multilevel laminoplasty, which can be used in the treatment of


cervical myelopathy. The spinal cord is decompressed by increasing the anterior to
posterior dimension of the spinal canal.

The treatment of cervical myelopathy is dictated by the number of stenotic levels,


sagittal alignment of the spine, and more recently attempt to retain motion. Anterior
cervical discectomy and fusion (ACDF) is used for one or two level disease with
pathology localized to anterior spine. Posterior decompression and fusion is typically
used for disease at more than two levels and is contraindicated with kyphosis greater
than 13 degrees. Lamninoplasty and cervical disc arthroplasty are thought to be
motion-preserving alternatives, although there can be unintended loss of motion with
both techniques. The open door laminoplasty technique involves the use of a suture
anchor or small plate to maintain the opening on one side while the french door
technique involves hinges bilaterally and opening in the midline.

Emery reviewed the diagnosis and management cervical myelopathy. Surgical


management is advocated for patients with functional impairment. Depending on
location and extent of pathology, anterior procedures may require corpectomy and use
of strut grafting.

Gerard et al. reviewed techniques of surgical management of myelopathy. They


describe a minimally invasive posterior decompression using a tubular retractor
system. Patients are carefully selected and should have less than 3 levels of disease,
no evidence of instability, and normal lordosis.
Figure A is a T2 sagittal MRI image showing multilevel cervical stenosis. Figure B is
lateral radiograph of a patient who underwent multilevel laminoplasty. Illustration A
shows the open door technique.

Incorrect Answers:
Answer 1: An ossified posterior longitudinal ligament is not a contra-indication to the
procedure and instead may be helpful in that situation.
Answer 2: Laminoplasty is motion-preserving and does not violate the facet joints
Answer 3: Laminoplasty increases the anterior to posterior dimension, not the medial
to lateral dimension of the spinal canal
Answer 5: While instrumentation is unilateral, the approach for laminoplasty still
involves exposure of the bilateral lamina.

(SAE09SN.91) When comparing the overall outcomes of surgical versus nonsurgical


treatment of stable thoracolumbar burst fractures in patients without neurologic
injury, 5 years following injury, the principle differences lie in Review Topic

1 fracture kyphosis.
2 reduction of retropulsed bone.
3 pain reduction.
4 incidence of complications.
5 return to work.

PREFERRED RESPONSE 4

When patients are compared at 5 years follow-up, there are no statistically significant
differences between the two groups with respect to kyphosis, the degree of
retropulsed bone resorption, pain and function levels, or the ability to return to work.
Nonsurgical management of stable neurologically intact burst fractures has a very low
incidence of complications.

(SAE09SN.8) What is the primary reason for including the ilium in the distal fixation
of long instrumentation constructs in adult scoliosis? Review Topic

1 Better coronal balance


2 Better pelvic balance
3 Reduced fretting and corrosion
4 Improved curve correction
5 Improved fusion success

PREFERRED RESPONSE 5

Studies have shown that when compared with fixation to the sacrum alone, the
success rate of fusion across the lumbosacral junction increases when both the sacrum
and ilium are included in the posterolateral construct. Curve correction, coronal
balance, and pelvic balance are all attended to within the thoracolumbar spine and are
not directly related to the pelvic fixation. Fretting and corrosion are a byproduct of
metal-to-metal connections.

(OBQ14.94) A 32-year-old woman is ejected from her vehicle in a motor vehicle


accident. She has a distant history of pacemaker implantation for an unknown
arrthymia. She complains of neck pain. Examination reveals midline cervical
tenderness but no neurological deficit. CT scans of her cervical spine are shown in
Figures A and B. She requires an emergency laparotomy for a splenic injury.
Regarding her cervical spine, what is the next best step? Review Topic

1 This Type I traumatic spondylolisthesis of the axis requires rigid collar


immobilization.
2 This Type II traumatic spondylolisthesis of the axis requires reduction by axial load
and extension, followed by halo immobilization.
3 This Type IIA traumatic spondylolisthesis of the axis requires reduction by traction
and extension, followed by halo immobilization.
4 This Type III traumatic spondylolisthesis of the axis requires open reduction and
fixation with a posterior approach
5 This Type III traumatic spondylolisthesis of the axis requires open reduction and
fixation with an anterior approach
PREFERRED RESPONSE 1

This patient has a Levine-Edwards Type I hangman's fracture. Rigid cervical collar
immobilization is usually successful.

Traditional hangman's fractures (execution by hanging) are hyperextension-distraction


injuries, severing the spinal cord. Motor vehicle accidents/falls are caused by
hyperextension-compression, and neurologic injury is uncommon because the fracture
fragments separate, decompressing the spinal canal. The Levine-Edwards
classification is most commonly used. Type II fractures have disruption of the
posterior longitudinal ligament, and Type III have bilateral facet joint dislocation.

Pryputniewicz et al. reviewed axis fractures. They advocate rigid collar or halo
immobilization as initial treatment for hangman's fractures, and surgery for fusion
failures or irreducible fractures or repeatedly unstable fractures during initial bracing.
For unstable fractures or failure of external immobilization, surgical options include
C2-C3 ACDF and dorsal C1-C3 fusion.

Jackson et al. reviewed upper cervical spine injuries. They advocate collar
immobilization for Types I and IA fractures, gentle reduction and halo vest
immobilization for 6-8weeks for Types II and IIA fractures, and open reduction and
wiring/plating (depending on the integrity of the facets/lamina) for Type III fractures.

Li et al. performed a systematic review of hangman fractures. They advocate nonrigid


external immobilization for stable Type I and Type II injuries, rigid immobilization
for Type IIa and III fractures, and surgery for Levine-Edwards Type IIA and III
fractures with significant dislocation or possibility for late instability.

Figure A is a composite of sagittal CT scan images through left facet and pars, dens,
and right facet and pars. There is no facet dislocation. Figure B is a composite of
sequential axial CT scan images showing bilateral pars interarticularis fracture.
Illustration A is the Effendi classification. Illustration B is the Levine-Edwards
classification.

Incorrect Answers:
Answer 2: This is a Type I fracture. Type II fractures are treated with reduction by
traction and extension (NOT axial load), followed by halo vest immobilization.
Answer 3: This is a Type I fracture. Type IIA fractures are treated with reduction by
gentle axial load and extension (NOT traction), followed by halo vest immobilization.
Answers 4 and 5: This is a Type I fracture. Type III fractures require open reduction
and fixation through a posterior approach. Anterior facet fixation can be used for
failed posterior fusion or for C1–C2 instability with destruction of the posterior
elements of C1 and C2. An anterior C2-C3 discectomy and fusion is suitable for
hangman's fracture with intervetebral disc injury.
(SAE08OS.197) The most important radiographic predictor of a good clinical
outcome following adult spinal deformity surgery is correction of Review Topic

1 pelvic incidence.
2 listhesis.
3 rotational deformity.
4 sagittal balance.
5 coronal deformity.

PREFERRED RESPONSE 4

Surgery for adult deformity, such as degenerative scoliosis and kyphosis, has gained
popularity in recent years. Improved fixation techniques, such as pedicle screws, and
increased familiarity and comfort with anterior surgery have resulted in greater curve
correction. Multiple studies have demonstrated that correction of sagittal balance is
the most important radiographic predictor of a good clinical outcome. While
correction of coronal deformity is often a surgical goal, it does not appear to be as
important in improving patient outcomes. Correction of listhesis, particularly in the
surgical treatment of adult spondylolisthesis, is controversial because its impact on
clinical outcomes has not been clearly established. Rotational deformities, though
often present with adult scoliosis, are difficult to correct. Pelvic incidence is a fixed
parameter that is unchanged with surgery.

(OBQ13.236) A 39-year-old male falls off his bicycle and complains of neck pain and
tingling in his fingers. Trauma series radiographs are seen in Figures A and B. Which
of the following is likely to be true? Review Topic

1 Examination would likely reveal a short neck, low posterior hairline and limited
neck motion.
2 Serum human leukocyte antigen B27 is likely to be positive.
3 He is likely to be of Japanese descent.
4 The disease is defined by flowing ossification of the anterior longitudinal ligament
at 4 consecutive levels.
5 Rheumatoid factor is likely to be positive.

PREFERRED RESPONSE 2

This patient has ankylosing spondylitis (AS). HLA-B27 is positive in 90% of patients
with this disease. This tends to occur in younger patients (as opposed to DISH, which
happens in older male patients).

Cervical spine fractures are not uncommon in ankylosing spondylitis because of


osteoporosis and the long lever arm from fused vertebrae. They commonly occur
because of hyperextension of the cervical spine (usually C5-7) and have a high rate of
neurologic injury. AS fractures have a higher rate of neurologic injury than DISH
fractures. Posterior decompression and stabilization with long constructs is necessary

Whang et al. reviewed spine injuries in 12 patients with AS and 18 patients with
DISH. Most injuries involved C5-C7. Patients with AS were more likely to have
severe neurologic injury (41% ASIA A) than DISH (44% ASIA E). There was 81%
good-excellent outcome and 4 deaths related to halo vest use.

Caron et al. reviewed spine fractures in patients with ankylosing spine disorders (AS
and DISH). AS patients were younger than DISH patients. Spinal cord injury was
present in 58%. Surgery was performed on 67% and comprised instrumentation 3
levels above/below the injury. Mortality correlated with age. Mortality was 32%.

Westerveld et al. performed a systematic review on spine injuries in ankylosing spinal


disorders. Most patients had sustained low energy trauma (fall from sitting/standing).
In DISH, most fractures were through the vertebral body. In AS, vertebral body
fractures equaled those through the disc. Surgery was performed for neurological
deterioration, unstable fracture and the presence of an epidural hematoma.

Figure A shows a hyperextension injury at C6-7 (Type I, disc or Type IV,anterior


disc, posterior body) cervical spine fracture in ankylosis spondylitis. Visible
radiographic characteristics include osteopenia, bamboo spine, marginal
syndesmophytes and ossfication of the disc space. Figure B is a chest radiograph
showing thoracic syndesmophytes consistent with ankylosing spondylitis. Figure C
shows bilateral sacroilitis and hip joint space narrowing typical of ankylosing
spondylitis. Illustration A shows the Caron classification of spine fractures in
ankylosing spondylitis (Type A, disc injury; Type B, body injury; Type C, anterior
body, posterior disc injury; Type D, anterior disc, posterior body injury). Illustration
B shows the difference between the marginal osteophytes of AS and nonmarginal
osteophytes of DISH in the cervical spine. If you have osteophytes that are building
up, and project out anterior to the anterior cortex of the vertebral bodies, like "flowing
wax" it is DISH.

Incorrect Answers:
Answer 1: This is characteristic of Klippel-Feil syndrome
Answer 3: This is characteristic of ossfication of the posterior longitudinal ligament
(OPLL), which occurs in up to 2% of the Japanese population.
Answer 4: This is characteristic of diffuse idiopathic skeletal hyperostosis (DISH,
Forrestier's disease).
Answer 5: Ankylosing spondylitis is a seronegative spondyloarthropathy and
rheumatoid factor is usually negative.

(SAE11OS.104) Among patients with lumbar degenerative disk disease and low back
pain, what factor is most predictive of clinical outcomes after surgical management?
Review Topic

1 Duration of symptoms
2 Workers' compensation
3 Use of disk arthroplasty
4 Severity of disk degeneration
5 Number of spinal segments treated

PREFERRED RESPONSE 2

The treatment of low back pain ranges from nonsurgical management to surgical
management. Whereas many other treatment modalities have been investigated,
lumbar arthrodesis remains the primary surgical treatment of lumbar diskogenic pain.
Outcomes of surgical management vary but are consistently impacted negatively by
workers' compensation status. Neither the radiographic severity of disease, number of
spinal segments, nor duration of disease has been correlated with clinical outcomes.
While total disk arthroplasty was hoped to be an improvement over fusion, the
evidence available to date has shown no significant differences over arthrodesis.

(SAE12SN.60) Which of the angles depicted in Figure 60 has been found to most
closely correlate with a patient's lumbar lordosis, thoracic kyphosis, and overall
sagittal alignment? Review Topic
1A
2B
3C
4D
5 A and D

PREFERRED RESPONSE 1

Angle A represents pelvic incidence (PI), a constant anatomic relationship between


the pelvis and sacrum. Angle B represents pelvic tilt, and angle C represents sacral
slope. Pelvic tilt and sacral slope can change depending on the rotation of the pelvis.
Pelvic incidence has been found to directly correlate with the magnitude of lumbar
lordosis and thoracic kyphosis because it determines the angle at the base of the spine
(the lumbosacral junction). To obtain sagittal balance, the remainder of the spine
compensates, resulting in the degree of lumbar lordosis and thoracic kyphosis to
maintain an upright posture. Thus, PI must be considered in the evaluation of sagittal
balance and potential reconstructive procedures. Angle D represents the T1 angle.

(SAE12SN.81) Figures 81a through 81c show the MRI scans of a 53-year-old man
who has experienced a long history of progressively worsening right-sided back pain
with radiation to the buttocks and right lower extremity. Examination reveals
weakness and hyperreflexia in the right lower extremity. He reports intermittent
episodes of urinary incontinence. What is the most appropriate surgical approach?
Review Topic
1 Costotransversectomy
2 Laminectomy
3 Transpedicular
4 Lateral extracavitary
5 Transthoracic

PREFERRED RESPONSE 2

The presence of stenosis in the thoracic region with its related clinical manifestations
has only recently been appreciated. The pathogenesis of thoracic stenosis is similar to
that found in the cervical or lumbar spine. Two distinct clinical syndromes of thoracic
stenosis have been identified, the most common being associated with degenerative
changes of the spine. Clinical manifestations include development of unilateral or
bilateral symptoms of pseudoclaudication. Focal radicular pain or paresthesias may
also be present. The neurologic examination initially may be normal, but as the degree
of neural compression progresses, posterior column dysfunction and long tract signs
appear. If allowed to progress untreated, the patient may develop significant difficulty
with gait and bowel/bladder function. Thoracic spinal cord stenosis secondary to
congenital narrowing of the spinal canal is associated with a more abrupt onset of
symptoms. The typical clinical manifestations of myelopathy may commence
following minor or moderate trauma. Radicular symptoms are rare in congenital
thoracic stenosis. One indication for thoracic laminectomy is a patient in whom
imaging has demonstrated evidence of spinal canal stenosis secondary to hypertrophy
of the posterior elements. A laminectomy should not be the primary approach when
stenosis results from a significant ventral epidural osteophyte or herniated disk; these
lesions are more effectively and safely managed by a posterolateral (transpedicular,
transfacetal, or costotransversectomy) or an anterior approach.

(SAE12SN.4) A patient with a grade 2 L5-S1 isthmic spondylolisthesis reports low


back pain and bilateral lower extremity pain. Nonsurgical management has failed to
provide relief, and the patient is now a candidate for surgical intervention. The
surgeon elects to proceed with L5-S1 laminectomy and posterior instrumented fusion
after reduction of the spondylolisthesis. If a postoperative neurologic deficit develops,
what structure has most likely been affected? Review Topic

1 L4 nerve root
2 L5 nerve root
3 S1 nerve root
4 Genitofemoral nerve
5 Cauda equina

PREFERRED RESPONSE 2

The L5 nerve root is especially vulnerable and prone to injury after the reduction of
spondylolisthesis in patients with mid-and high-grade isthmic spondylolisthesis. The
genitofemoral nerve is more commonly injured during anterior retroperitoneal
approaches to the lumbar spine. Injury to the cauda equina often leads to bowel and
bladder dysfunction and lower extremity weakness and is uncommon after reduction
maneuvers.

(SAE12SN.26) A 57-year-old man has had a 2-week history of neck pain. He has no
history of radiating symptoms, and has no complaints of numbness or paresthesias.
There was no trauma associated with the onset of the pain. Figure 26 shows the MRI
scan initially obtained by his family physician. What should the patient be told
regarding the prevalence of the MRI findings in his age group? Review Topic

1 Less than 10%


2 20% to 25%
3 50% to 60%
4 75% to 80%
5 Greater than 95%
PREFERRED RESPONSE 3

The MRI findings reveal age-related degenerative changes in the cervical spine,
which is a very common finding in the adult population. Boden and associates
evaluated cervical spine MRI findings on 63 asymptomatic subjects, and found that
the prevalence of having at least one degenerative disk was approximately 57% in
those older than age 40 years.

(SAE12SN.34) Pedicle subtraction osteotomies (PSO) are commonly performed in


the lumbar spine to treat sagittal imbalance. What is the most common complication
following a PSO in the lumbar spine? Review Topic

1 Pseudarthrosis
2 Nerve root injury
3 Spinal cord injury
4 Aortic injury
5 Dural tear

PREFERRED RESPONSE 1

The rate of pseudarthrosis at 5-year follow-up is 29%, with most occurring at the
thoracolumbar junction cephalad to the site of the PSO. The rate of postoperative
neurologic deficits is 11%, with 2.8% resulting in permanent deficits. Spinal cord
injury is rare because the PSO is typically performed in the lumbar spine below the
conus.

(SAE12SN.61) An 83-year-old man is seen in the emergency department for


evaluation of neck pain after a motor vehicle accident. The patient has no neurologic
deficits. The patient has a history significant for late stage ankylosing spondylitis with
cervicothoracic kyphosis. What is the most appropriate method of immobilization for
the cervical spine while diagnostic testing is performed? Review Topic

1 Soft collar
2 Hard cervical collar with head immobilization to a backboard
3 Halo fixation with the neck in extension
4 Maintenance of flexed positioning of the spine
5 No immobilization necessary
PREFERRED RESPONSE 4

In patients with trauma in the setting of ankylosing spondylitis and cervicothoracic


kyphosis, it is highly recommended that the neck be maintained in the pre-morbid
flexed position until definitive management can be performed. Ankylosing spondylitis
is a chronic inflammatory disease that is characterized by ossification of the spinal
column with an associated progressive kyphotic deformity of the spine. The deformity
therefore becomes the native position for the patient with ankylosing spondylitis.
Extension of the injured spine in a patient with ankylosing spondylitis can lead to
neurologic injury and/or displacement of a previously aligned fracture.

(SBQ12SP.43) The right vertebral artery sustains a complex injury during a


occipitocervical (C3-O) fusion. Bleeding cannot be controlled with local tamponade
and a hemostatic agent. After clamping the vessel a segmental defect is noted in the
vessel. The patient undergoes urgent angiography, which reveals adequate collateral
circulation. What is the next step in the management of this injury? Review Topic

1 Addition of a systemic antifibrolytic solution


2 Direct repair
3 Contralateral vetebral artery anastomosis
4 Acute synthetic grafting
5 Embolization treatment

PREFERRED RESPONSE 5

When (1) vetebral artery [VA] bleeding cannot be controlled with local tamponade
and a local hemostatic agent, and (2) collateral circulation is maintained in the non-
dominant artery (left vertebral artery is usually dominant), the next best step would be
to achieve local control with an intraoperative endovascular procedure such as
embolization, stenting or clipping.

The management of VA injury is difficult. If local tamponade can be achieved, the


next best step would be to consider direct repair of the artery. If tamponade fails to
achieve proper hemostasis, additional procedures may be considered, such as
intraoperative endovascular embolization, or clipping and ligation. When considering
these intraoperative endovascular treatments, the collateral circulation should be
assessed. If collateral circulation is inadequate, direct repair or stenting should be re-
considered.

Peng et al. reviewed the anatomical considerations, management, and preventive


measures of vertebral artery injury in cervical spine surgery. They showed that
ligation-associated morbidities such as cerebellar infarction, cranial nerve palsies, or
hemiplegia can occur in up to 12% of cases. They suggest direct repair should be
considered as first-line treatment when local hemostasis can be controlled.

Lall et al. reviewed the perioperative complications associated with occipitocervical


fusion. The most commonly encountered complications were related to
instrumentation failure after nonunion (6-7%). Other complications included vertebral
artery injury (1.3%-4.1% during placement of C1-C2 transarticular screws, most
commonly in the case of high-riding vertebral artery), dural tears, and wound
infections.

Illustration A shows the treatment algorithm of VA injuries as suggested by Peng et


al.

Incorrect Answers:
Answer 1: The addition of a systemic antifibrolytic solution (tranexamic acid) would
not control large vessel bleeding after injury and achieve long-term hemostasis.
Answer 2: The technical demands of the exposure and microvascular repair of a
complex vetebral injury, in the setting of uncontrolled bleeding, would be too
difficult. Especially when it involves the non-dominant artery and collateral
circulation is maintained.
Answer 3: Again, the technical demands of the exposure and microvascular
anastomosis, in the setting of uncontrolled bleeding, would be too difficult.
Answer 4: Vertebral artery stent graft may be considered in chronic situations, such as
symptomatic vertebrojugular arteriovenous fistula. This would not be a first line
treatment in the setting of uncontrolled bleeding.

(OBQ04.76) Which of the following is true regarding changes in the vascularity of the
adult intervertebral disc with age? Review Topic

1 Neovascularization occurs from the nucleus pulposus


2 Neovascularization occurs from the outer annulus
3 Neovascularization occurs from the endplates
4 There is no change in vascularity
5 There is a decrease in vascularity

PREFERRED RESPONSE 2

As a person ages through adulthood, neovascularization of the intervertebral disc


originates from the outer annulus.

The intervertebral disc is composed of an outer structure called the annulus fibrosis
and an inner structure called the nucleus pulposus. The annulus fibrosis is composed
of type 1 collagen, water, and proteoglycans. The inner nucleus pulposus is composed
of type 2 collagen, water, and proteoglycans. Intervertebral discs are avascular with
capillaries terminating at the end plates. The nucleus pulposus receives nutrition
primarily through diffusion through blood vessels within the endplate.

Roberts et al. review the histology and pathology of the intervertebral disc. They note
that at birth, the cartilagenous end plates have large vascular channels through them as
well as vascular channels through the annulus. Soon after birth, these vascular
channels close with none remaining at the end of the first decade of life. However,
with age, more blood vessels grow into the disc from the outer annulus fibrosis in
response to degenerative changes.

Illustration A is a diagram of the vascular supply in an adult intervertebral disc.

Incorrect Answers:
Answer 1: Blood vessels cannot reach the inner nucleus pulposus.
Answer 3: The vascular channels through the end plates close soon after birth.
Answers 4 and 5: There is an increase in vascularization with age.

(SAE11OS.169) Figures 169a through 169c show the radiograph and MRI scans of a
74-year-old woman who has had back and bilateral leg pain for the past 6 months.
Nonsurgical management has failed to provide relief. What is the best option for
surgical treatment? Review Topic

1 Posterior decompression
2 Posterior interbody arthrodesis
3 Posterior decompression and in situ arthrodesis
4 Posterior decompression and instrumented arthrodesis
5 Anterior and posterior arthrodesis
PREFERRED RESPONSE 4

The patient has symptoms of lumbar spinal stenosis and radiographic evidence of a
grade I degenerative spondylolisthesis at L4-5. Surgical treatment has been shown to
provide better clinical outcomes than nonsurgical management. Treatment for
spondylolisthesis remains somewhat controversial but posterior lumbar instrumented
arthrodesis is best supported in the literature. Decompression alone places the patient
at risk for recurrent stenosis and progression of deformity. Noninstrumented
arthrodesis for this condition results in high rates of nonunion and worsened long-
term outcomes. There is insufficient evidence to support the role for interbody
arthrodesis (either through an anterior or posterior approach) compared with posterior
decompression and arthrodesis.

(SAE08AN.16) If a surgeon inadvertently burrs through the midlateral wall of C5


during a anterior corpectomy, what structure is at greatest risk for injury? Review
Topic

1 C5 root
2 C6 root
3 Internal carotid artery
4 Vertebral artery
5 Vagus nerve

PREFERRED RESPONSE 4

The vertebral artery is contained within the vertebral foramen and thus tethered
alongside the vertebral body, making it vulnerable to injury if a drill penetrates the
lateral wall. The C5 root passes over the C5 pedicle and is not in the vicinity. The C6
root passes under the C5 pedicle but is posterior to the vertebral artery and is only
vulnerable at the very posterior-inferior corner. The carotid artery and the vagus nerve
are both within the carotid sheath and well anterior.

(SBQ12SP.54) Integrity of the posterior ligamentous complex (PLC) is a critical


predictor of spinal fracture stability. Components of the PLC include the supraspinous
ligament, interspinous ligament, ligamentum flavum and: Review Topic

1 Facet joint capsules


2 Facet joint capsules, and facet joints
3 Facet joint capsules, facet joints, and the posterior longitudinal ligament
4 Facet joint capsules, and the posterior longitudinal ligament
5 Posterior longitudinal ligament

PREFERRED RESPONSE 1

Components of the PLC include the supraspinous ligament, interspinous ligament,


ligamentum flavum and facet joint capsules.

Numerous methods have been used to evaluate for PLC injury. Palpation is unreliable
and has low accuracy. Radiographs can show characteristic flexion-distraction
fracture patterns with widening or malaligment of the spinous processes. Computed
tomography (CT) is more reliable than radiographs to provide indirect evidence of
ligament injury. Magnetic resonance image (MRI) can provide direct evidence of soft-
tissue injury, making it the preferred method in diagnosing ligamentous injury.
However, MRI may not always be utilized due to situations involving emergency
operations or contraindications to MRI, such as certain metal implants.

Vaccaro et al. introduced a new classification system for thoracolumbar injuries,


TLICS, based on morphological appearance, integrity of the posterior ligamentous
complex, and neurological status. They advocate use of the system for nonoperative
versus operative decision making and communication between surgeons.

Varccaro et al. sought to determine the accuracy of magnetic resonance imaging


(MRI) in diagnosing injury of the posterior ligamentous complex (PLC) in patients
with thoracolumbar trauma. Forty-two patients with 62 levels of injury were studied.
The sensitivity for the various PLC components ranged from 79% (left facet capsule)
to 90% (interspinous ligament). The specificity ranged from 53% (thoracolumbar
fascia) to 65% (ligamentum flavum). They concluded that the integrity of the PLC as
determined by MRI should not be used in isolation to determine treatment.

Incorrect Answers:
Answers 2-5: Components of the posterior ligamentous complex ONLY include the
supraspinous ligament, interspinous ligament, ligamentum flavum and facet joint
capsules.

(SAE09SN.38) A 79-year-old woman reports a history of left leg pain with walking.
Her pain is exacerbated with walking and stair climbing, and her symptoms are
improved by standing after she stops walking. Lumbar flexion does not provide any
significant improvement of the symptoms and sitting does not significantly change
symptoms. Her leg pain is worse at night and she obtains relief by hanging her leg
over the side of the bed. The neurologic examination is essentially normal.
Examination of the lower extremities demonstrates mild early trophic changes, and
her pulses distally are palpable but are diminished bilaterally. Radiographs are shown
in Figures 15a and 15b. What is the next most appropriate step in management?
Review Topic

1 Decompression and posterior fusion at L4-L5


2 Epidural steroid injection at L4-5
3 Nonsteroidal medications and physical therapy for 6 weeks
4 Measurement of the ankle-brachial index
5 CT myelogram

PREFERRED RESPONSE 4

The patient has symptoms that are more consistent with vascular claudication than
with the pseudoclaudication anticipated from lumbar spinal stenosis. Therefore, the
patient is a candidate for further vascular work-up. The radiographs reveal early
spinal stenosis and spondylolisthesis at L4-5 but also show significant calcification of
the iliac arteries, suggestive of peripheral vascular disease. Vascular claudication is a
manifestation of peripheral vascular disease and presents with crampy leg pain that is
exacerbated by physical exertion. The pain is easily relieved by standing still or
sitting. Unlike pseudoclaudication, a forward-flexed posture and/or sitting does not
improve the symptoms. Night pain is common in vascular claudication due to the
elevation of the extremities and patients often report pain improvement by hanging
their extremities in a dependent position. In evaluation of a patient with suspected
vascular claudication, the five “P’s” of vascular insufficiency should be monitored,
including pulselessness, paralysis, paresthesia, pallor, and pain. While pain and
paresthesias can be common in both vascular claudication and pseudoclaudication, the
presence of any of the remaining symptoms is suggestive of vascular disease.

(OBQ13.190) A 58-year-old man presents to the clinic with 9 months of progressive


right lower extremity pain. Over the past 4 months, he also notes a decreased ability
to walk long distances due to pain, which is relieved by sitting down. Figure A and B
are his T2 sagittal and axial MRI scans, respectively. Which of the following
statements is true regarding this patient's 4-year outcome in regards to surgical and
non-surgical management? Review Topic

1 Surgical management will lead to more improvement in pain but not function
2 Surgical management will have higher 4-year mortality
3 Surgical management will lead to more improvement in pain, function, and
satisfaction
4 Nonsurgical management will lead to more improvement in pain but not function
5 Surgical management will lead more improvement in function, but less
improvement in pain

PREFERRED RESPONSE 3

The clinical presentation is consistent with lumbar stenosis. At 4 years, surgical


management is expected to result in more improvement in pain, function, satisfaction
than nonoperative management.

Surgical management of lumbar stenosis typically consists of wide pedicle to pedicle


decompression at the affected levels. Instrumention and fusion may be indicated if
there is evidence of instability. Nonsurgical management typically involves oral
medications, physical therapy, and corticosteroid injections

Weinstein et al. (2010, 2008), as part of the SPORT trial, found that surgical
management of symptomatic lumbar stenosis with decompressive laminectomy
resulted in greater improvement in pain, function, and satisfaction as compared to
nonsurgical management. These advantages were maintained at both two and four
years of follow up.

Figure A is a T2 sagittal MRI showing lumbar spinal stenosis from L1-L5, most
prominent from L3-L5. Figure B is a T2 axial MRI showing severe stenosis at L4-L5
secondary to ligamentum flavum hypertrophy and facet arthropathy. Illustration A is
an example of a one-level decompressive laminectomy.

Incorrect Answers:
Answer 1,5: Surgical management leads to more improvement in pain, function, and
satisfaction.
Answer 2: Surgical management and nonsurgical management have the same 4-year
mortality.
Answer 4: Nonsurgical management leads to less improvement in symptoms.

(SAE12SN.44) On an axial CT image, which of the following dimensions is


considered to be indicative of a critical amount of lumbar spinal stenosis? Review
Topic

1 AP canal diameter of less than 20 mm


2 AP canal diameter of less than 50 mm
3 Cross-sectional area of less than 100 mm2
4 Cross-sectional area of less than 50 mm2
5 Circumferential area of less than 25 mm

PREFERRED RESPONSE 3

Lumbar central stenosis is defined by an AP canal diameter of less than 10 mm or a


cross-sectional area of less than 100 mm2 as measured on CT.

(SAE12SN.18) A 42-year-old woman has a 3-week history of acute lower back pain
with radiation into the left lower extremity. There is no history of trauma and no
systemic symptoms are noted. Examination reveals a positive straight leg test at 25
degrees on the left side. Motor testing reveals mild weakness of the gluteus maximus
and weakness of the gastrocnemius at 3/5. Sensory examination reveals decreased
sensation along the lateral aspect of the foot. Knee reflex is intact; however, the ankle
reflex is absent. MRI scans show a posterolateral disk herniation. The diagnosis at this
time is consistent with a herniated nucleus pulposus at what level? Review Topic

1 L1-2
2 L2-3
3 L3-4
4 L4-5
5 L5-S1
PREFERRED RESPONSE 5

The patient's history and examination findings are consistent with a lumbar disk
herniation at the L5-S1 level. Weakness of the gastrocnemius and gluteus maximus
are consistent with an S1 lumbar radiculopathy. Nerve root tension signs are also
consistent with a disk herniation at L5-S1, which typically affects the traversing S1
nerve root.

(SAE12SN.2) In the evaluation of somatosensory-evoked potential waveforms for


intraoperative neuromonitoring for spinal surgery, the minimum criteria for
determining potentially significant changes include Review Topic

1 10% decrease in amplitude, 50% decrease in latency.


2 10% decrease in amplitude, 50% increase in latency.
3 0% loss of amplitude, transient increase in latency.
4 50% decrease in amplitude, 10% increase in latency.
5 50% decrease in amplitude, 10% decrease in latency.

PREFERRED RESPONSE 4

The established criteria for interpreting a significant change are 50% decrease in
signal amplitude, 10% latency increase, and/or a complete loss of potential.
Intraoperative spinal cord monitoring during spinal surgery generally consists of a
combination of monitoring modalities. Somatosensory-evoked potentials in
combination with intraoperative electromyography can provide adequate coverage of
sensory and motor components of spinal cord and nerve root function. Significant
changes in evoked potential waveform characteristics can reflect dysfunction of the
ascending somatosensory system.

(SAE09SN.4) A 44-year-old man reports persistent left leg pain following a L5-S1
hemilaminotomy and partial diskectomy. Examination shows a grade 4 weakness of
the left extensor hallucis longus and a positive left straight leg raise. A radiograph is
shown in Figure 1a, and sagittal and axial MRI scans are shown in Figures 1b and 1c.
Nonsurgical management consisting of medication, physical therapy, and injections
has failed to provide relief. Surgical management should consist of Review Topic
1 revision L5-S1 hemilaminotomy.
2 L5-S1 total disk arthroplasty.
3 L5 Gill laminectomy.
4 posterior foraminal decompression and fusion at L5-S1 with instrumentation and
bone graft.
5 stand-alone posterior lumbar interbody fusion.

PREFERRED RESPONSE 4

The patient has a grade I isthmic spondylolisthesis at L5-S1. He has an L5


radiculopathy with foraminal stenosis. Any further treatment needs to include an
arthrodesis and foraminal decompression. Isolated interbody fusion is contraindicated
in patients with spondylolisthesis, as is total disk arthroplasty. Therefore, the best
procedure is a posterior fusion with instrumentation and bone graft along with a
foraminal decompression.

(SAE12SN.50) Which of the following factors is most likely to contribute to


pseudarthrosis in a patient who has undergone a single-level anterior decompression
and fusion procedure for the treatment of cervical radiculopathy? Review Topic

1 Sagittal alignment
2 History of diabetes mellitus and tobacco use
3 Performance of an uninstrumented fusion (ie, no plate and screws)
4 Use of allograft (instead of autograft)
5 Fusion at the C3-C4 level

PREFERRED RESPONSE 2

Various factors affect the pseudarthrosis rate in patients who undergo anterior cervical
decompression and fusion. Patient factors, including history of smoking and history of
diabetes mellitus, have been shown to significantly increase pseudarthrosis rates. The
literature has been mixed with regard to fusion rates for allograft versus autograft,
especially for one-level fusions; in that category, there is minimal, if any, difference.
Similarly, several authors have shown higher rates of fusion with uninstrumented
single-level rather than instrumented anterior cervical decompressions and fusions.
The level (ie, cranial or caudal) of fusion and sagittal alignment have not been
correlated with fusion rates.

(SAE08AN.2) The injection shown in Figures 1a and 1b would most benefit a patient
who reports which of the following symptoms? Review Topic

1 Dorsal foot pain extending into the great toe


2 Foot pain extending along the lateral border of the foot
3 Pain extending into the foot in a stocking distribution
4 Anterior thigh and shin pain ending at the ankle
5 Lateral foot paresthesias

PREFERRED RESPONSE 1

The images demonstrate a L5 selective root block as it exits the L5-S1 foramen. This
root block best helps relieve pain or paresthesias in the L5 distribution, which is the
dorsal first web space and the great toe. The lateral foot is an S1 distribution and
would need to be blocked through the posterior first sacral foramen. The anterior shin
and thigh represent the L4 root which exits a level above this at the L4-5 foramen. A
stocking distribution is nonanatomic and not indicative of a specific root.
(OBQ14.7) A 43-year-old man is currently taking medication for the disease
condition shown in Figure A. His wife is taking the same medication. Her radiograph
is shown in Figure B. What is the medication? Review Topic

1 Denosumab
2 Adalimumab
3 Tofacitinib
4 Imatinib
5 Rituximab

PREFERRED RESPONSE 2

The male patient has ankylosing spondylitis (AS), and is taking the same medication
as his wife, who has rheumatoid arthritis (RA). Adalimumab is approved for both
conditions.

TNF-a inhibitors are biological agents approved for 2nd-line treatment of AS. These
include etanercept, infliximab, adalimumab, golimumab, and certolizumab pegol,
which are all approved for treatment of RA and psoriatic arthritis as well. Additional
biological agents unique to RA are IL1 antagonists (e.g. anakinra), B cell inhibitors
(rituximab), T cell inhibitors (abatacept), IL6 receptor inhibitors (tocilizumab) and
Janus kinase (JAK) pathway inhibitors (tofacitinib).

Kubiak et al. reviewed the orthopaedic management of AS. They support using anti–
TNFa medications when there is: (1) definitive diagnosis of AS, (2) disease lasting >
4 weeks, (3) refractory disease, (4) failure of corticosteroid sacroiliac joint injections,
(5) failure of sulfasalazine for peripheral disease, and (6) no medical contraindications
to the initiation of treatment.

Khalessi et al. reviewed the medical management of AS. They discuss physical
therapy, education, and medications and radiation therapy. Non-biological
medications include NSAIDs, coxibs, corticosteroids and DMARDS (sulfasalazine
and methotrexate). They note that both sulfasalazine and methotrexate do not improve
pain or function from AS spinal disease.

Figure A shows bilateral sacroiliitis and bilateral hip arthritis, which in a young male
patient, is highly suggestive of AS. Figure B shows ulnar drift of the digits and MCPJ
subluxation characteristic of RA.

Incorrect Answers:
Answer 1: Denosumab is a RANKL inhibitor, mimicking the action of
osteoprotegerin. As Prolia, it is used to treat osteoporosis. As Xgeva, it is used for
prevention of skeleton-related events in patients with bone metastases, giant cell
tumors, multiple myeloma with bone metastases, and hypercalcemia of malignancy.
Answer 3: Tofacitinib is an oral Janus kinase (JAK) pathways inhibitor and is second-
line treatment for moderate to severe RA.
Answer 4: Imatinib is a tyrosine-kinase inhibitor used to treat Philadelphia
chromosome-positive (Ph+) chronic myelogenous leukemia (CML).
Answer 5: Rituximab is a CD20+ B cell inhibitor and is used to treat moderately to
severe RA unresponsive to therapy with 1 or more TNF antagonists.

(SAE12SN.95) A 35-year-old rock climber sustains an L1 burst fracture from a 30-


foot fall while climbing. He sustained no other fractures or serious injuries. He is
neurologically intact and has minimal posterior tenderness without increased spinous
process separation on examination. Radiographs reveal kyphosis of 20 degrees
between T12 and L2 with 30% vertebral height loss. A CT scan shows 55% canal
compromise. What is the most appropriate management? Review Topic

1 Bed rest for 6 weeks, followed by a thoracolumbosacral orthosis (TLSO) until the
fracture is healed
2 TLSO and or body cast for 3 to 6 months with mobilization when comfortable
3 Posterior spinal fusion and instrumentation
4 Anterior decompression with spinal fusion and instrumentation
5 Anterior decompression and anterior-posterior spinal fusion and instrumentation

PREFERRED RESPONSE 2

Wood and associates have shown that the use of a TLSO or a body jacket was equally
effective as surgery for the treatment of thoracolumbar burst fractures without
neurologic deficit. The only difference in any of the measured parameters (including
pain, functional outcome, residual canal compromise, and kyphosis) was a decreased
complication rate in the nonsurgical group compared with the surgical group. The
maximum time to mobilization in the nonsurgical group was 5 days.

SAE11AN.92) A 24-year-old man sustained a bilateral C5-6 facet dislocation in a car


accident and was intubated at the scene. He remains sedated in the intensive care unit
so the clinical neurologic examination is limited. What MRI finding would most
likely predict a complete spinal cord injury? Review Topic

1 4-mm rostral caudal cord edema


2 Disruption of the anterior longitudinal ligament
3 Edema in the soft tissue anterior to the spine
4 Diffuse cord edema
5 6-mm cord hematoma

PREFERRED RESPONSE 5

The MRI finding that most consistently corresponds with a complete spinal injury is a
hematoma within the cord. Cord edema can predict a poor prognosis if it is more
extensive but is not considered as consistent a finding. Ligamentous injury about the
neck can indicate musculoskeletal instability but it does not in and of itself indicate
the presence or predict the severity of spinal cord injury. Likewise, soft-tissue edema
anterior to the spine may indicate musculoskeletal injury but does not offer specific
information regarding the presence or absence of cord injury.

(SAE10PE.17) A 14-year-old boy has had a 3-month history of low back pain with no
known trauma. The pain is worse with activity and relieved by rest, although he does
report difficulty with prolonged sitting in school. The patient was on the football team
but stopped participating because of the back pain during football practice. He reports
no history of radicular pain and denies any numbness, tingling, or weakness in the
legs. Neurologic examination is normal. Back examination reveals slight tenderness
over the lower back area but no swelling or skin defects. Strength testing is 5 over 5 in
the lower extremities and the straight leg raise test is negative. Back range of motion
is nearly full, but back extension is painful. The hamstrings are slightly tight. Initial
radiographs, including AP, lateral and oblique views, are negative. What is the best
test to determine the patient's diagnosis? Review Topic

1 Flexion and extension lateral radiographs


2 MRI
3 Myelogram
4 Diskogram
5 Bone scan with SPECT

PREFERRED RESPONSE 5

A bone scan with SPECT is very sensitive and specific for spondylolysis not seen on
initial radiographs. MRI can sometimes visualize spondylolysis, but it is not as
sensitive nor as specific as a bone scan with SPECT. Flexion and extension views
have no role in the evaluation of the patient who presents with classic spondylolysis-
type symptoms. The most sensitive physical examination finding is pain with back
extension. Oblique radiographs can be obtained, but they are not as sensitive or
specific as a bone scan with SPECT. The patient does not have any signs of a disk
problem; therefore, an evaluation of the disk is not helpful.

(SBQ13PE.79) A 17-year-old male American football lineman presents with low back
pain of insidious onset that is somewhat worse with activity. He has no neurologic
complaints, night pain or fevers. His symptoms have been present for a few years but
this is the first time he has sought medical attention. What physical examination
finding is most likely to be found in this clinical scenario? Review Topic

1 Popliteal angle of 5 degrees


2 Heel cord tightness
3 Increased femoral anteversion
4 Pain with lumbar extension in single leg stance
5 Numbness of the skin of the anterolateral calf and dorsum of the foot

PREFERRED RESPONSE 4

The patient demographics and clinical presentation are consistent with lumbar
spondylolysis. Pain with lumbar extension is the most common physical exam
finding.

Office assessment of the patient with spondylolysis should note pertinent negatives
that would signify other causes of back pain. The history is most commonly negative
for neurologic symptoms such as weakness or numbness, although patients will
occasionally have radicular pain. On exam, patients may have localized spasm or
tenderness, step off (if there is spondylolisthesis), hamstring tightness. The most
common finding is pain with lumbar extension.

McCleary et al. review the diagnosis and treatment of spondylolysis in athletes. They
identify three types of patients with spondylolysis: (1) female dancer or gymnast who
is hyperlordotic, with increased motion and flexibility, (2) male weightlifter or
football player undergoing a growth spurt, with decreased motion and flexiblity,
especially of the spinal erectors, and (3) a novice athlete undergoing vigorous
preparation for a new sport, with poor core strength and flexibility.

Incorrect Answers:
Answers 1 and 2: Hamstring (not gastroc-soleus) tightness is characteristic of patients
with spondylolysis, this could result in a large popliteal angle. Although this patient
could have a popliteal angle of 5 degrees, pain with lumbar extension is more likely.
Answer 3: Femoral anteversion is not related to spondylolysis
Answer 5: In the absence of disk herniation, numbness is not typical of patients with
spondylolysis.

(SAE11AN.57) Which of the following radiographic parameters is most predictive of


a poor result following multilevel fusion surgery for adult degenerative scoliosis?
Review Topic

1 An L5-S1 degenerative disk left out of the fusion


2 Coronal imbalance
3 Residual scoliosis of greater than 25 degrees
4 Residual foraminal stenosis
5 Sagittal imbalance

PREFERRED RESPONSE 5

Sagittal imbalance appears to be the greatest predictor of a poor surgical outcome in


multilevel fusions for adult scoliosis. Coronal imbalance is better tolerated as long as
it is not excessive. The amount of residual scoliosis does not seem to play a role as
long as overall balance is achieved. The issue of including the L5-S1 level in long
fusions remains debatable, and some residual foraminal stenosis can be tolerated,
particularly when included within the stabilized/fused segments.

(OBQ13.198) A 56-year-old male presents to your office with a primary complaint of


pain in his lower back that extends down his left leg when he walks. He states he rides
a stationary bike without pain, but he has severe pain walking more than two blocks.
On exam he has 5/5 strength in all major muscle groups, and his sensation is intact to
light touch in all dermatomes. He has no upper motor neuron signs. The pain has been
going on for about a year, and he has had no improvement with physical therapy or
anti-inflammatory medication. Figure A is an upright lateral radiograph of his lumbar
spine. Figures B is his sagittal MRI, and Figure C is an axial image through L4/5.
Assuming this patient is going to undergo surgery, what is most important in ensuring
longterm symptomatic relief? Review Topic
1 A solid L4/5 fusion
2 The addition of an interbody fusion device
3 Not disrupting the L4/5 facet capsules during surgery
4 Using rhBMP-2 in the surgery
5 Removing all of the herniated disc

PREFERRED RESPONSE 1

This patient has neurogenic claudication with a degenerative spondylolisthesis.


Patients with a degenerative spondylolisthesis will have the best longterm outcomes if
a wide decompression is accompanied by a solid fusion.

Degenerative spondylolisthesis occurs when there is anterior translation of one


vertebral body (most commonly L4) on another vertebral body (most commonly L5).
When this occurs, there can be compression of the traversing/caudal (L5) nerve root
in the lateral recess. Because of the instability associated with this diagnosis, a fusion
is needed to preserve longterm outcomes.

Kornblum et al., reported on the outcomes of 47 patients with degenerative


spondylolisthesis at an average follow-up of 7 years and 8 months. Excellent to good
results were reported in 86% of patients with a solid arthrodesis compared to 56% of
patients with a pseudarthrosis.

Weinstein et al., reported the 4-year data from the SPORT study and found that
patients with degenerative spondylolisthesis treated with surgery had statistically
significant improvements in health related quality of life scores compared to those
treated non-operatively.

Figure A is an upright lateral radiograph of the lumbar spine in which a degenerative


spondylolisthesis of L4 on L5 is identified. Figure B is a sagittal T2 MRI re-
demonstrating the spondylolisthesis as well as spinal stenosis. FIgure C is the axial
image through L4/5 demonstrating spinal stenosis; additionally, a left sided facet cyst
can be seen.

Illustration A and B are the postoperative films from the same patient after he
underwent an L4/5 posterior decompression and instrumented fusion.
Incorrect Answers:
Answer 2 is incorrect because the addition of an interbody fusion device has not been
shown to affect clinical outcomes in patients with a degenerative spondylolisthesis.
Answer 3 is incorrect because it is completely acceptable to disrupt the facet capsules
at the level of a fusion. In non-fusion surgery, the facet joints should be protected.
Answer 4 is incorrect because using rhBMP-2 has not been shown to increase the
longterm clinical results of surgery.
Answer 5 is incorrect because the patient does not have a herniated disc, rather the
patient has a degenerative spondylolisthesis causing symptoms.

(SBQ13PE.7) A 45-year-old HIV-positive homeless man presents with increasing low


back pain for the last three weeks. He now reports difficulty ambulating, fever, and
loss of appetite. He denies bowel and bladder symptoms. He denies any symptoms
radiating into his buttock or legs. On physical exam he has in obvious discomfort with
standing which worsens in the forward flexion position. He has a normal motor and
sensory exam in his lower extremities. Blood cultures are performed which come
back negative. What would be the most next appropriate step in treatment? Review
Topic

1 Broad spectrum antibiotics


2 Isoniazid, rifampin, and pyrazinamide therapy
3 CT guided biopsy with cultures
4 Technetium bone scan
5 Anterior corpectomy with a retroperitoneal approach, strut grafting and
instrumentation

PREFERRED RESPONSE 3

The clinical presentation is consistent with spondylodiscitis. Although the patient has
risk factors for spinal tuberculosis, a CT guided biopsy should be performed to
establish a diagnosis.
There is an increasing incidence of TB in United States due to increasing
immunocompromised population from HIV. 15% of patients with TB will have
extrapulmonary involvement. 5% of all TB patients have spine involvement. With any
type of spondylodiscitis the infectious organism must be identified with blood
cultures or a biopsy prior to initiating treatment.

Khoo et al. emphasize with the recent global pandemic of human immunodeficiency
virus, the number of tuberculosis and secondary spondylitis cases is again increasing
at an alarming rate. They report that medical treatment alone remains the cornerstone
of therapy for the majority of Pott disease cases. Surgical intervention should be
limited primarily to cases of severe or progressive deformity and/or neurological
deficit.

Hadjipavlou et al. performed a retrospective study of 101 cases of spondylodiscitis.


They found Staphylococcus aureus was the main organism. Infection elsewhere was
the most common predisposing factor. Leukocyte counts were elevated in 42.6% of
spondylodiscitis cases. The erythrocyte sedimentation rate was elevated in all cases of
epidural abscess.

Figure A shows lumbar radiograph with a radiolucent lesion in L2 with mild loss of
disc height of the L2/3 level. Figure B shows a T2-weighted MRI showing a lesion
involving the L2/3 disc spaced and extending into the L2 vertebral body.

Incorrect Answers:
Answer 1: Broad spectrum antibiotics should not be initiated until a diagnosis has
been made with either blood cultures or a CT guided biopsy.
Answer 2: Isoniazid, rifampin, and pyrazinamide therapy is the mainstay of treatment
for tuberculosis of the spine. However, although suspicion is high for tuberculosis in
this patient, a diagnosis with a biopsy must be made first.
Answer 4: A technetium bone scan would be indicated if there was a high suspicion
for metastatic bone disease, which is not the case in this situation.
Answer 5: This patient has no evidence of spinal instability or neurologic deficits.
Therefore surgery is not indicated at this time.

(SAE09SN.67) A 67-year-old retired steelworker was involved in a motor vehicle


accident and sustained a midcervical spinal cord injury. Radiographs and MRI scans
reveal severe cervical stenosis and spondylosis without fractures or dislocations.
Neurologic examination reveals an ASIA C spinal cord impairment with greater
motor involvement of the upper extremities than the lower extremities. What is the
probability that the patient eventually will become ambulatory? Review Topic

1 2% to 5%
2 15% to 20%
3 35% to 45%
4 60% to 70%
5 Greater than 90%

PREFERRED RESPONSE 3

The patient sustained an incomplete spinal cord injury known as central cord
syndrome. Central cord syndrome characteristically has disproportionate involvement
of the upper extremities with the lower extremities being relatively spared. It is most
commonly seen after cervical injuries in elderly patients with spondylosis and spinal
stenosis, often without fracture. Penrod and associates noted that 23 of 59 patients
with central cord syndrome (ASIA C and D) ultimately walked. The poorest
prognosis, however, was in ASIA C patients older than age 50, in which only 40%
walked.

(SAE12SN.67) A 50-year-old man fell from a height of 10 feet and sustained an axial
loading injury to the cervical spine. He reports neck pain and right upper extremity
weakness and has weakness in the lower extremities. An MRI scan is shown Figure
67. What imaging study should be obtained next to further evaluate this patient?
Review Topic

1 Cervical spine flexion and extension radiographs


2 AP radiograph of the cervical spine
3 CT of the cervical, thoracic, and lumbar spine
4 Electromyography and nerve conduction velocity studies of the upper extremities
5 MRI of the lumbar spine
PREFERRED RESPONSE 3

The MRI scan shows a C7 burst fracture. A CT scan of the cervical spine will allow
for optimal evaluation of this C7 burst fracture. Specifically, it will provide additional
osseous detail and will assist with the detection of additional fractures, including
those of the posterior elements. Additional CT imaging of the thoracic and lumbar
spine is required to rule out concommitant injuries (which may be present in 10% to
15% of patients). Anteroposterior and lateral cervical spine radiographs would be a
good option for further evaluation but are not included in the available choices here.
Cervical spine flexion and extension radiographs should not be obtained in a patient
who is known to have a relatively unstable spine and a neurologic deficit.
Electromyography and nerve conduction velocity studies are best used to evaluate for
cervical radiculopathy secondary to degenerative abnormalities and are usually not
indicated in the acute trauma setting.

(SAE08OS.46) A patient has a large T11-T12 disk herniation that is causing


substantial compression of the spinal cord. The patient reports walking imbalance
over the past few weeks. Examination of the patient's reflexes is likely to show
Review Topic

1 normal reflexes in the upper extremities and hyperreflexia in the lower extremities.
2 hyperreflexia in the upper extremities and normal reflexes in the lower extremities.
3 hyperreflexia in the upper extremities and hyporeflexia in the lower extremities.
4 hyperreflexia in the upper and lower extremities.
5 hyporeflexia in the upper and lower extremities.

PREFERRED RESPONSE 1

The patient has a large thoracic disk herniation that is causing spinal cord
compression. The history of gait imbalance suggests that the patient has thoracic level
myelopathy. Assuming that the patient does not have lumbar stenosis, compression of
the spinal cord at the T11-T12 level will cause upper motor neuron findings distal to
it. Hyperreflexia of the upper extremities would suggest that the patient has cervical
spinal cord compression. In this patient, the upper extremity reflexes should be
normal. Most likely, the patient will exhibit hyperreflexia in the lower extremities,
which is an upper motor tract sign.
(OBQ14.233) A morbidly obese 80-year-old woman presents with back pain for 2
months. Sagittal T2-weighted, sagittal T1-weighted, coronal short tau inversion
recovery (STIR) images are shown in Figures A through C, respectively. Which of the
following statements is true of her diagnosis? Review Topic

1 MR fluid sign suggests that this is an osteoporotic vertebral compression fracture


2 Convex posterior vertebral border suggests that this is an osteoporotic vertebral
compression fracture
3 Replacement of the normal marrow signal suggests that this is a malignant vertebral
compression fracture
4 Retropulsion of a vertebral body fragment suggests that this is a malignant vertebral
compression fracture
5 The band-like low T1 signal suggests that this is a malignant vertebral compression
fracture

PREFERRED RESPONSE 1

The MR fluid sign is highly suggestive of an osteoporotic vertebral compression


fracture (VCF).

The MR signs suggestive of a benign, osteoporotic VCF include band-like low T1


signal, fluid sign, retropulsion of a vertebral bone fragment. Acute fractures show low
signal on T1-weighted and high signal on T2-weighted and STIR images. The MR
signs suggestive of malignant VCF include complete replacement of normal marrow
signal, convex posterior vertebral border, pedicle involvement, focal paraspinal mass
and epidural mass.

Kim et al. performed a literature review on management of the osteoporotic spine.


They concluded that nonsurgical management should focus on pain control and
maximizing function. Surgery remains controversial and should be reserved for those
who fail nonoperative management. Surgical options include vertebroplasty, balloon
tamp reduction, and decompression and instrumented fusion with structural graft and
multiple fixation points.

Spivak et al. discussed percutaneous treatment of VCF. They recommend


vertebroplasty for patients with unremitting pain and little deformity as it is less costly
and does not require general anesthesia. They recommend kyphoplasty for cases with
greater vertebral collapse, especially subacute fractures where partial healing may
hinder postural fracture reduction.

Figure A is a T2-weighted sagittal MR image showing a fluid-filled cleft ("fluid


sign"). Figure B is a T1-weighted sagittal MR image showing loss of intensity (black)
consistent with vertebral body edema. Figure C is a coronal STIR image showing the
fluid-filled vertebral body cleft. An incidental renal cyst is noted. Illustration A shows
different patterns of band-like signal abnormality on T1-weighted imaging. The black
region represents low signal intensity (edema). Illustration B is a sagittal STIR image
showing the MR fluid sign. Illustration C shows the band like signal abnormality on
T1-weighted imaging. Illustration D shows metastatic disease with complete
replacement of normal marrow intensity and diffuse convexity of the posterior
vertebral border.

Incorrect Answers
Answer 2: A convex posterior vertebral border is suggestive of a malignant VCF.
Answer 3: While marrow replacement suggests a malignant process, there is no
marrow replacement seen in this patient. Rather, there is a fluid-filled cleft ("fluid
sign").
Answer 4: Retropulsion of vertebral body fragments suggests a benign VCF.
Answer 5: A band-like low T1 signal is consistent with a benign VCF.

(OBQ14.199) A 68-year-old male presents with gait instability, clumsiness of the


hands, and the MRI images shown in Figure A. You decide to proceed with surgical
decompression. When planning your surgical treatment, it is important to note that
compared to a posterior approach, the anterior procedure has: Review Topic

1 Higher risk of infection


2 Lower risk of C5 radiculopathy
3 Higher over-all complication rate
4 Lower average blood loss
5 Increased rate of numbness to the long finger and wrist flexion weakness
PREFERRED RESPONSE 4

Surgical decompression of cervical myelopathy via an anterior procedure has lower


reported blood loss compared to a posterior procedure.

Cervical myelopathy has a progressive course and therefore if there is evidence of


functional impairment surgical decompression is indicated. Either an anterior
decompression or posterior decompression can be used depending on a variety of
factors including number of levels involved and sagittal alignment of the cervical
spine. In general, a posterior approach is used when three or more levels are involved
and the spine is in neutral or lordotic alignment.

Fehlings et al. did a prospective study on the risks of complications associated with
surgical treatment of cervical myelopathy. They found that combined anterior and
posterior procedures had a significantly higher rate of complication than either
anterior-only or posterior-only procedures. Posterior procedures had a higher rate of
wound infections compared to anterior. They found no statistical difference in the
over-all complication rate, incidence of C5 radiculopathy, or dysphagia between an
anterior-only or posterior-only procedure.

Fehlings et al. did a prospective study on outcomes following surgical treatment of


cervical myelopathy. At one year follow-up they found a significant improvement in
mJOA score, Nurick grade, NDI score, and all SF-36v2 dimensions. With the
exception of mJOA scores, these improvements were not statistically related to
severity of disease.

Liu et al. performed a meta-analysis of outcomes following surgical decompression of


cervical myelopathy. They found outcomes following anterior procedures were better
than those for posterior procedures when there were less than 3 affected levels. With 3
or greater levels, no statistical difference in outcomes could be found between the two
approaches. They note none of their reviewed publications represent high-quality
prospective randomized trials.

Figure A is a sagittal MR image of the cervical spine showing multi-level


degenerative disease with cord compression consistent with cervical myelopathy.

Incorrect Answers:
Answer 1: Incidence of wound infection was found to be higher in patients following
a posterior procedure.
Answer 2: No statistically significant difference was found in the incidence of C5
radiculopathy between the anterior or posterior procedures.
Answer 3: No statistically significant difference was found in the over-all
complication rate between the anterior or posterior procedures.
Answer 5: No statistically significant difference was found in the incidence of C5
radiculopathy between the anterior or posterior procedures.
(SAE13BS.14) A 23-year-old man was a restrained backseat passenger in a head-on
motor vehicle collision. Examination revealed extensive bruising diagonally across
his chest and abdominal areas and extreme pain at the thoracolumbar junction
posteriorly. The lateral chest radiograph showed splaying of the spinous processes and
increased disk height at the level of the injury, but no fracture of the vertebral bodies.
He underwent a posterior spine fusion with instrumentation to stabilize the spine after
a full trauma workup revealed the spine to be his only injury. Where is the center of
rotation for this injury? Review Topic

1 Interspinous ligaments
2 Posterior longitudinal ligament
3 Anterior longitudinal ligament
4 Abdominal viscera

PREFERRED RESPONSE 4

The injury described is termed a "chance fracture" of the spine, and the radiograph
suggests significant soft-tissue disruption of the spine without bony involvement
anteriorly. The question asks the examinee to infer that the injury is an all-soft-tissue
injury, continuing through the disk space, and that the forces acting on the spine are
almost completely distractive and on the same side of the center of rotation. A chance
fracture of the spine is classically regarded as a flexion-distraction injury resulting in
variable bony/soft-tissue injury to the spine, classically associated with a center of
rotation anterior to the spine (frequently being associated with bowel or other
abdominal organ injury).

(OBQ13.27) A 35-year-old physical therapist presents with right-sided back and leg
pain. For the last 4 months, he has taken anti-inflammatory medications and
performed exercises on his own. While his back pain has improved slightly, his leg
pain remain severe and interferes with his ability to sleep and work. Examination
reveals positive ipsilateral and contralateral straight leg raise at 30 degrees. He has
mildly diminished big toe dorsiflexion strength on the right side. There is a small
patch of diminished sensation on the dorsum of the foot. MRI scans are shown in
Figures A and B. What is the most appropriate next step in treatment? Review Topic
1 Continued rest, formal physical therapy and anti-inflammatory medications
2 Targeted chemonucleolysis of the L4-5 disc
3 Discectomy and interbody fusion L4-5
4 Discectomy utilizing a midline approach between the spinous process and
multifidus
5 Discectomy utilizing an intermuscular approach between multifidus and
longissimus

PREFERRED RESPONSE 4

The clinical presentation is consistent with a paracentral disc herniation at L4/5 that
has failed nonoperative treatment and continues to limit is his activities of daily
living. A laminotomy and discectomy (microdisckectomy) with a midline approach
would be the next most appropriate treatment.

For lumbar disc herniation, the first line of treatment is rest, physical therapy and oral
medications (NSAIDs, gabapentin, steroids). The second line of treatment is selective
nerve root corticosteroid injections. The last line in treatment is laminotomy and
discectomy.

Pearson et al. determined which individuals (as opposed to groups) in the SPORT
(Spine Patients Outcomes Research Trial) would benefit from surgery. They found
that disc herniation patients improved more with surgery than without.

Lurie et al. reviewed the 8 year outcomes of the SPORT. In patients with HNP on
imaging and leg symptoms persisting for at least 6 weeks, surgery was superior to
nonoperative treatment in relieving symptoms and improving function.

Figures A and B are sagittal and axial T2-weighted MRI images showing a large
L4/L5 herniated disc causing neural foramina narrowing and impinging on the right
L5 root.

Incorrect Answers:
Answer 1: The patient has already taken anti-inflammatories and performed exercises.
A repeat course of rest and physical exercise is not warranted.
Answer 2: There is no indication for chemonucleolysis. Because of serious side
effects (anaphylaxis, hemorrhage, pain, paralysis), in 2003, the FDA halted the sale
and distribution of chymopapain in the US
Answer 3: There is no instability to warrant interbody fusion.
Answer 5: Laminotomy and discectomy utilizing an intermuscular approach between
multifidus and longissimus would be appropriate for a far lateral disc herniation, not a
paracentral disc herniation.

(SAE09SN.93) Figure 33 shows the MRI scan of a 55-year-old woman who has had a
6-week history of back and leg pain. Which of the following clinical scenarios is most
consistent with the MRI scan findings at L4-L5? Review Topic

1 L4 nerve root radiculopathy


2 L5 nerve root radiculopathy
3 Associated bowel and bladder dysfunction
4 Symptoms associated with arachnoiditis
5 Wide-based gait, left-sided Hoffman’s sign

PREFERRED RESPONSE 1

The MRI scan reveals a L4-L5 foraminal disk herniation originating from the L4-5
disk space that has migrated up into the foramen, compressing the left L4 nerve root.
There is normal distribution of the roots in the cerebrospinal fluid, excluding
arachnoiditis as a diagnosis, and disk herniation in this location would not result in
cauda equina syndrome or myelopathy.

(SAE10PE.70) A patient with Pott's disease, tuberculosis of the spine, is more likely
to have which of the following early findings? Review Topic

1 Acute onset back pain and neurologic dysfunction


2 Preservation of the disk space between two affected adjacent end plates
3 Involvement of the cervical spine and torticollis
4 Elevated WBC count and markedly elevated erythrocyte sedimentation rate
5 Lordotic deformity in late stages of the disease

PREFERRED RESPONSE 2

Tuberculosis of the spine typically has an indolent presentation. Unlike pyogenic


infections of the spine, the disk space is usually preserved. Most commonly, the
thoracic and lumbar spine are affected. Laboratory studies may be nonspecific.
Delayed presentation usually results in neurologic compromise and a kyphotic
deformity. Treatment includes a multidrug regimen. Surgery is indicated for
deformity correction or failure of medical treatment.

(SAE09SN.61) Steroids are thought to prevent neurologic deterioration after


traumatic spinal cord injury by which of the following mechanisms? Review Topic

1 Maintains calcium influx into damaged cells


2 Destabilizes lysosomal membranes in the zone of injury
3 Reduces TNF-alpha expression
4 Increases NF-kB binding capacity
5 Maintains free radical oxidation

PREFERRED RESPONSE 3

The proposed mechanisms by which steroids such as methylprednisolone are thought


to prevent neurologic deterioration by limiting secondary insult, include: decreasing
the area of ischemia in the cord, reducing TNF-alpha expression and NF-kB binding
activity, decreasing free radical oxidation and thus stabilizing cell and lysosomal
membranes, and checking the influx of calcium into the injured cells, thus reducing
cord edema.

(SAE12SN.32) For a patient with a type II odontoid fracture, which of the following
factors best predicts the development of a nonunion with nonsurgical management?
Review Topic

1 Frontal oblique pattern


2 Magnitude of fracture displacement
3 Degree of posterior angulation
4 Age
5 Patient history of diabetes mellitus

PREFERRED RESPONSE 2

All five factors have been found to be associated with nonunion for type II odontoid
fractures. Of these, initial fracture displacement of greater than 6 mm has the greatest
association with the development of fracture nonunion.

(SAE12SN.99) An acute posterolateral disk herniation at the L4-5 level will most
likely affect what nerve root? Review Topic

1 L3
2 L4
3 L5
4 S1
5 S2

PREFERRED RESPONSE 3

Posterolateral disk herniations will generally compress the transversing nerve root.
Therefore, a posterolateral disk herniation at the L4-5 level will typically compromise
the L5 nerve root. Far lateral (extraforaminal) disk herniations generally compromise
the exiting nerve root. Therefore, an extraforaminal herniation at the L4-5 level will
typically compromise the exiting L4 nerve root.

(SAE12SN.6) A 63-year-old man has a feeling of generalized clumsiness in his arms


and hands, difficulty buttoning his shirt, and gradually worsening gait instability.
During examination, his neck is gently passively flexed to end range while he is
seated. The patient describes an electric shock-like sensation that radiates down the
spine and into the extremities. This describes which of the following? Review Topic

1 A positive Lhermitte sign


2 A positive Spurling sign
3 A positive Jackson sign
4 A positive Lasegue sign
5 A positive Hoffmann sign

PREFERRED RESPONSE 1

What is now referred to as Lhermitte sign was first described by Marie and Chatelin
in 1917 to describe "transient 'pins and needles' sensations traveling the spine and
limbs on flexion of the head" in some patients with head injuries. A positive test is
indicated by the presence of electric-like sensation down the spine or extremities. It is
associated with cervical spinal cord pathology from a wide variety of etiologies,
including multiple sclerosis. Recent studies suggest that it has a low sensitivity and
high specificity. Spurling's sign is elicited by axial compression of the tilted head.
Jackson's sign is elicited by hyperextension of the cervical spine. Lasegue sign refers
to the straight leg raise with dorsiflexion of the ankle for lumbar radiculopathy.
Hoffmann sign is a pathologic reflex of the upper extremity, that is felt to be an
indicator of cervical myelopathy.

(SAE12SN.14) What clinical scenario is most consistent with the MR image of the
L4-L5 disk level shown in Figure 14? Review Topic

1 Left L4 nerve root radiculopathy


2 Right L5 nerve root radiculopathy
3 Bowel and bladder dysfunction
4 Arachnoiditis
5 Symptomatic pseudomeningocele

PREFERRED RESPONSE 1

The MRI scan reveals a foraminal disk herniation originating from the L4-L5 disk
space that has migrated into the foramen compressing the left L4 nerve root. There is
no evidence of compression of the right L5 nerve root. Bowel and bladder dysfunction
are not associated with L4-mediated nerve function. There is no evidence of
arachnoiditis or pseudomeningocele.
(SAE08OS.114) A 22-year-old man sustained a cervical fracture-dislocation of the
C5-6 level in a motor vehicle accident along with an associated spinal cord injury. Six
months after his injury, he has 4 out of 5 biceps on the left, with 5 out of 5 biceps on
the right. Deltoid is graded at 5 out of 5 bilaterally. There is 0 strength in the triceps,
wrist flexors, wrist extensors, and digital extensors. He has neurogenic bowel and
bladder with absent perianal sensation and no voluntary motor in the lower
extremities. The patient's neurologic deficit is best categorized as which of the
following? Review Topic

1 Incomplete C5 spinal cord injury


2 Complete C5 spinal cord injury
3 Complete C6 spinal cord injury
4 Central spinal cord injury
5 Brown-STquard syndrome

PREFERRED RESPONSE 2

The patient has a complete spinal cord injury. The level of a spinal cord injury is
determined by the most distal intact (5/5) function. The lowest motor intact level in
this patient is C5 based on the described examination. Central spinal cord injury and
Brown-STquard injuries are both incomplete patterns of spinal cord injury.

(SAE09SN.68) A 20-year-old man involved in a motor vehicle accident is brought to


the emergency department with a C6-7 unilateral facet dislocation. His neurologic
examination reveals a focal left-sided C7 nerve root palsy. He is awake and
cooperative with questioning and has no other obvious traumatic injuries. What is the
most appropriate treatment at this time? Review Topic

1 Further imaging studies, including MRI


2 An awake closed reduction with Gardner-Wells traction with neurologic
examination
3 Immobilization in a halo skeletal fixation for definitive treatment
4 Closed reduction under general anesthesia
5 Immediate open reduction and internal fixation in the surgical suite
PREFERRED RESPONSE 2

In the patient who is neurologically intact or has an incomplete injury from a cervical
facet dislocation, a closed reduction with weighted tong traction is appropriate when
the patient is awake, alert, and cooperative. Although there is a risk that a cervical
facet dislocation could occur with an underlying cervical disk herniation, Vaccaro and
associates have shown that closed reduction can be safely carried out in the awake,
responsive patient. Closed reduction can be performed in the emergency department
with traction with skull tongs or a halo ring. A slow stepwise application of weight is
added until a reduction is achieved. Any worsening of the neurologic status of the
patient requires immediate termination of the closed reduction and further diagnostic
imaging before proceeding with further treatment.

(SAE12SN.9) Of the following, what is the most reliable method of assessing spinal
fusion? Review Topic

1 Radiographs
2 MRI
3 Flexion/extension radiographs
4 CT
5 CT myelography

PREFERRED RESPONSE 4

Despite the ease of attainment, radiographs only accurately diagnose failed


arthrodesis in 60% to 80% of uninstrumented cases and these numbers are even lower
in cases with posterior instrumentation. The role of dynamic radiographs remains
unclear because of the paucity of normative data values after lumbar spine fusion. CT
scans provide excellent bony detail and their images are not affected by metal
components as in MRI. Post-myelogram CT is useful for identifying neurologic
compression.

(SAE08OS.117) The spinal cord terminates as the conus medullaris at what vertebral
level in adults? Review Topic

1 T12
2 L1
3 L2
4 L3
5 L4

PREFERRED RESPONSE 2

The spinal cord anatomy changes at the thoracolumbar junction. The spinal cord
terminates as the conus medullaris at the lower portion of L1 in women and the
pedicle of L1 in men.

(OBQ13.141) A 13-year-old girl presents with back pain for 6 months. Figures A and
B are SPECT scan and CT images taken at the time of presentation. What is the most
likely diagnosis? Review Topic

1 Osteoid osteoma
2 Bone island
3 Spondylolysis
4 Osteoblastic metastases
5 Aneurysmal bone cyst

PREFERRED RESPONSE 3

This patient has spondylolysis.

Spondylolysis is a common cause of back pain in children/adolescents. It is common


in sports with repetitive hyperextension (gymnasts, weightlifters, football linemen). It
is best seen on lateral and oblique radiographs, CT (best study to diagnose and
delineate anatomy), and SPECT.

Saifuddin et al. reviewed the orientation of the pars fracture. They found that only
32% of defects were oriented within 15° of the 45° lateral oblique plane and would be
visible on oblique radiographs. They thus recommend CT scans for spondylolysis.

Cheung et al. reviewed spondylolysis and spondylolisthesis. They advocate pars


repair for symptomatic spondylolysis and low-grade, mobile spondylolisthesis with
pars defects cephalad to L5 and for those with multiple-level defects.

Figure A is a 99mTc-MDP SPECT scan showing increased uptake at the right L5 pars
interarticularis. Figure B is an axial helical CT image showing bilateral spondylolysis
at L5. Illustration A is a corresponding sagittal reconstruction image demonstrating
right pars fracture into the right L5 superior facet. Illustration B shows the appearance
of osteoblastic metastasis (green arrow). Illustration C shows the options for pars
repair.

Incorrect Answers:
Answers 1 and 2: Osteoid osteomas and osteoblastomas have a predilection for the
posterior elements and will be hot on bone scan/SPECT. They are less likely to found
in bilateral pars, and are not usually associated with pars fractures.
Answer 4: There is no history of cancer to suggest blastic metastasis.
Answer 5: The lesion demonstrates extensive sclerosis and is bilateral. This is not
typical of an ABC.

(OBQ12.176) A 12-year-old female is referred to the office by a community


orthopaedic surgeon concerned that her shoulders appear to be at different heights.
With Adam's forward bending, she is noted to have a significant right thoracic rib
prominence. She denies pain. A detailed neurological examination reveals no
abnormalities. Radiographs of her spine show an apex left lumbar curve measuring 32
degrees and an apex right thoracic curve measuring 28 degrees. She had her first
menses last month and her Tanner-Whitehouse staging is consistent with an
adolescent steady state. The most appropriate treatment would be? Review Topic

1 Posterior spinal fusion


2 Spinal manipulations
3 Observation and referral to an endocrinologist
4 Thoracolumbosacral orthosis
5 Halo-gravity traction

PREFERRED RESPONSE 4

A growing child presenting with a curve of 25°–40° or a curve 20- 25° with
documented progression should be treated first with a thoracolumbosacral orthosis.

Assessing a child's skeletal maturity has important clinical implications when treating
patients with idiopathic scoliosis. The Tanner-Whitehouse III method specifically
uses the distal radial and ulnar epiphyses, as well as the metacarpal and phalangeal
epiphyses of the first, third, and fifth digits for determination of skeletal age. A
Tanner-Whitehouse Stage in 'adolescent steady-state' means she is past peak height
velocity. However, as she is before skeletal maturity, and her lumbar and thoracic
curvatures are > 25 deg., these curvatures may continue to progress. Therefore,
bracing would be the best treatment option at this time.

Hasler et al. randomized twenty post-pubertal young women (20°–40° idiopathic


scoliosis to an observation or complementary and alternative osteopathic treatment in
a prospective study. Pre- and post- intervention trunk morphology and spine
flexibility were assessed at 3 month intervals. A regression analysis of repeat
measurements revealed no therapeutic effect on rib hump, lumbar prominence, plumb
line, sagittal profile or global spinal flexibility with either treatment.

Sponsellar et al. examined the literature form the past 25 years to summarize the
current practice of bracing in the treatment of adolescent idiopathic scoliosis. They
found the highest level of existing evidence from a prospective center-randomized
study, which showed that thoracolumbosacral bracing was most effective for single
curves of 25 to 35 degrees in female patients with a starting Risser score of 0 to 2.

Illustration A shows the bones of interest in the hand and wrist for the the Tanner-
Whitehouse III skeletal maturity assessment. Each are divided into epiphysis,
metaphysis and diaphysis to help identify these different ossification centers in the
phalanx proximity as seen in Illustration B. The development of each region is
divided into discrete stages and given a letter (A,B,C,D, . . ., I). These correlate with a
score that are added together to give an overall maturity score on the graph.

Incorrect Answers:
Answer 1: Posterior spinal fusion can be used for all types of idiopathic scoliosis.
However, the indications for posterior fusion in this patient would include, failed non-
operative treatment, rapidly progressing curvatures, symptomatic double curvatures,
cobb angle > 45 degrees.
Answer 2: Spinal manipulation is considered an alternative and complementary
treatment for idiopathic scoliosis. There is little evidence that supports or refutes this
treatment. It is currently not supported.
Answer 3: Observation alone may be warranted in skeletally mature patients with
stable curvatures <25 degrees. As this patient has a double curvature, bracing should
be the initial treatment with close follow-up to assess for progression.
Answer 5: Halo-gravity traction has been used pre-operatively before spinal operative
spinal stabilization for scoliosis. This would not be suggested in this patient as first
line treatment.

(SAE09SN.28) What root is most commonly involved with a segmental root level
palsy after laminoplasty? Review Topic
1 C3
2 C4
3 C5
4 C6
5 C7

PREFERRED RESPONSE 3

The postoperative incidence of C5 root palsy after laminoplasty ranges from 5% to


12%. Other roots also may be affected. The palsies tend to be motor dominant,
although sensory dysfunction and radicular pain are also possible. The palsy may
arise during the immediate postoperative period or up to 20 days later. C5 may be
preferentially involved because it is at the apex of the cervical lordosis. Recovery
usually occurs over weeks to months.

(SBQ12SP.52) A 68-year old woman with known Rheumatoid cervical spondylitis


presents with progressive neck pain, hand clumsiness and gait instability. She has
been treated with methotrexate and etanercept for several years. All of the following
are indications for cervical decompression and surgical intervention EXCEPT?
Review Topic

1 Cervicomedullary angle <135 degrees on MRI


2 3.5 mm of change in the atlanto-dens interval (ADI) between flexion and extension
views
3 Posterior atlanto-dens interval (PADI) <14 mm
4 Progressive myelopathy
5 ADI > 10 mm with no change on flexion/extension views

PREFERRED RESPONSE 2

The patient has been treated with agents for rheumatoid arthritis (RA) and is
developing symptoms concerning for rheumatoid cervical spondylitis. All of the
answers are indications for surgical intervention EXCEPT >3.5 mm change in ADI on
flexion/extension views.

With the introductions of disease-modifying antirheumatic agents (DMARDs), the


incidence of RA patients undergoing cervical spine surgery has decreased
significantly. Basilar invagination, atlantoaxial instability, and subaxial subluxation
are the three most common manifestations of cervical disease. Multiple studies in RA
patients with untreated or poorly controlled disease have led to the development of a
set of measurements that identify patients who require surgical intervention and
predict outcome after surgery. Additionally, progressive neurological compromise and
refractory pain are indications for intervention.

Kim and Hilibrand reviewed management of the rheumatoid cervical spine and
outline parameters for surgical intervention. These include a PADI < 14 mm,
cervicomedullary angle <135 degrees, progressive neurological deficit, refractory
pain, atlantoaxial impaction as determined by migration >5 mm rostral to McGregor's
line, and subaxial canal diameter < 14 mm.

Boden et al. analyzed 73 patients followed for rheumatoid cervical spine disease with
an average follow up of 7 years. They found that the PADI correlated with paralysis.
Patients with PADI less than 10 mm had no recovery, and all patients with PADI
greater than 14 mm had full recovery.

Illustration A demonstrates the measurement of the ADI and PADI. Illustration B


demonstrates how to measure the cervicomedullary angle (as marked by A), which is
typically determined on MRI

Incorrect Answers:
Answer 1, 3-5: all of these parameters are indications for surgical intervention in the
rheumatoid cervical spine.

You might also like